Stats (Janakiram Sir's) PDF

You might also like

Download as pdf or txt
Download as pdf or txt
You are on page 1of 285

: jv

PROBLEMS
IN
BUSINESS STATISTICS
(with detailed notes, Illustrations, Problems from
Previous Question Papers and Hints)

With 13e$t (Sompliments ofl


..A .- A - .A A ..A ..A ...A vV .fj>
i For 3*
i >
B.Com.
i Second Year
>
< *
iy\ ^yk iy» -o^v lyv"
Dr. W. riniSHNA RAO
an- — M.Com.. Ph.O
• «WOE» IN COMMERCE
» fS U T BOYS 0E6REE COtlffiE
SECUNDERABAD 500 003 A P
TWO AUTHORS

SAI SURI PUBLICATIONS


H.No. 1-30-266/2, Plot No. 290,
Telecom Colony, Kanajiguda, Trimulgherry,
Secunderabad - 500 015. Ph : 27791915
First Edition 2009

Copy Right © All Rights Reserved with Aniline.

Copies can be had from

SAI SURI PUBLICATIONS DURGA BOOK DISTRIBUTORS


H.No. 1-30-266/2, Plot No 290, 5-3-655, Old Topkhana,
Telecom Colony, Kanajiguda, Opp. Osman Gunj,
Trimulgherry, Hyderabad.
Secunderabad - 500 015.
Ph : 27791915
1V» aJwsmikjwtOv) i

NEEL KAMAL BOOK AGENCY A-1 BOOK CENTRE


Sultan Bazar, Opp. S.P.G. Church
Hyderabad. Station Road, Secunderabad

RAJKAMAL BOOK CENTRE SONU BOOK AGENCY


Sultan Bazar, Sultan Bazar,
Hyderabad. Hyderabad.
15If*
n \ Jiu 892
SREE VIDYA BOOK DISTRIBUTORS
Koti, Hyderabad

P r ic e R s . 9 6 /-

Printed a t :
Sree Ramana Process Pvt Ltd. S.D. Road, Sec-bad-3. Ph 27811750
CONTENTS
Chapter Page No.

Introduction to Statistics.................................. 1-15

I. Classification of Statistical D ata..................... l-6h

II. . Measures of Central Tendency.......... 7-75

III. Measures of Dispersion................................. 76-113

IV. Skewness..................................................... 114-127

V. Correlation................................................. 128-144
*
VI. Regression................................................. 145-155

VII. Additional Problems ................................. 156-163

VIII. Typical Problems....................................... 164-169


Logarithms............................... 170-171
Anti logarithms........................................... 172-173

IX. Analysis of Time Series....................... A-l to A-36

X. Index Numbers..................................... B-1 to B-44

w
PREFACE
We fe e l immensely happy to place this hook in your hands.
The coverage in BUSINESS STATISTICS, which is expected o f the
students at the B. Com level is intended to groom them fo r further
studies at higher levels viz., M.Com, C.A., I.C.W.A., M.B.A.,
M. C.A., etc. Naturally, the standard books which cater to the needs
o f the students at higher level, leave fo r the B. Com, students very
little to digest. That is, the fru it is too big fo r their appetite.
Therefore, keeping in view the ability and needs o f the B.Com
students, we have endeavoured to present in (his book a balanced
choice o f problems. The problems including those which appeared
at various examinations conducted by Universities/professional
bodies have been supported by illustrations, hints, and above all,
by concise descriptive notes. We fervently hope that with these
features the book will go a long way in helping the students.

We owe a debt o f gratitude to M/s. Katepalli Gangaiah, paper


merchants who supplied paper on time and M/s. Sree Ramana
Process Pvt. Ltd. who have taken due care o f the get up o f the
book.

We hope that both the lecturers and students will find the
book useful and rewarding. C onstructive suggestions fo r
improvement in the book will be gratefully acknowledged.

Authors
SAI SURI PUBLICATIONS
H.No. 1-30-266/2, Plot No. 290,
Telecom Colony, K anajiguda, Trim ulglierry,
Secunderabad - 500 015. Ph : 27791915
INTRODUCTION
Origin and Development of Statistics:
The word 'Statistics’” has been derived from the Latin word ‘Status’ or the
Italian word ‘Statista’ or the German word ‘statistik’ or the French word
‘statislique’ each of which means a political state.
In the ancient times the scope of Statistics was primarily limited in keeping
the records of population in regard to age and sex wise, birth, death, property,
wealth etc., of a country. This knowledge was used as a tool to know the man
power and also to fix taxes and levies. Statistics in the past was a by-product
of administrative activity o f the state. The state collected statistics for
administrative purposes and as such statistics was regarded as ‘The Science
of Kings’. Many people contributed to the development of Statistics.
Sixteenth century saw the applications of statistics for the collections of the
data relating to the movements of stars and planets to foretell their position
and to make forecasts of eclipses. It is on the basis of these statistics that Sir
lsac Newton formulated his famous theory of Gravitation.
Seventeenth century witnessed the origin of vital statistics. Captain John Graunt
of London (1620-1674) known as the father of vital statistics, studied statistics
of births and deaths.
Eighteenth century, Sussmilch, a Purssian clergyman, found out the natural
order of physiocratic school i.e., ‘ratio of births and deaths remains constant’
and gave statistical explanations^ the theory. J.Bernoulli discovered the law
o f large numbers. Laplace published the monumental work on the ‘Theory of
Probability’
During the end o f the eighteenth century in Germany systematic collection of
statistics was made by the state. In India also the systematic collection of
statistics had been an age old tradition. Megasthanese had given an account
of method o f collecting data in respect of revenue and expenditure, births and
deaths, military, land etc., during the Chandragupta’s regime.
Francis Galton( 1822-1921) pioneered the study of ‘Regression Analysis’. Karl
Pearson (1857-1936) pioneered the study of Correlation Analysis. R. A. Fisher
is another great scholar who propounded many concepts which are useful to
draw conclusions from the statistical data.
In India, Prof. P.C.Mahalanobis, Dr.V.K.R.V.Rao and R.C.Desai are notable
among the economists who have made considerable
contribution to the field of statistics.
Till the second half of nineteenth century, the use of statistics was limited
either to the requirements of the governments or to social problems. But now
there is hardly any field or branch of knowledge which does not make use of
statistical methods. This is because of the increasing data needs of the business,
governments and science.
1
Meaning of Statistics:
The word ‘statistics’ has many meanings. It may mean any one of the
following:
a. Numerical statement of facts or simply data.
b. Statistical methods to help analysis and interpretation of data.
c. A measure based on sample observations.
The illustrations on the three possible uses are as follow.
a. Statistics as a product refers to the statistical data It is tlu- aggregate of
numerical statement of facts. For example, we talk about statistics of national
income, production, sales, births, deaths etc., In this context, the term ‘statistics’
is understood in a plural sense.
b. Statistics as a process refers to statistical method Statistical methods are
tools and techniques which aid the statistical investigation. The main phases
of a statistical investigation are collection, organization, presentation, analysis
and interpretation of data. Here statistics is undet stood in a singular sense.
c. Statistics derived from a small representative gump taken from the whole
lot used for drawing inferences about the charactei r.tics of the whole. Here
statistics represents measures based on sample obsei \ it ions.
,
Definition of Statistics:
Different authors have framed different definitions ol Statistics from time-
to-time. While some ofthem defined Statistics as ‘Sinitsin nl I )ata’(in a plural
sense), others defined it as ‘Statistical methods’(in a singulat sense). Some of
the important definitions are given below:
A. Statistics defined as ‘Statistical data’: Bowles defined Statistics as
‘Numerical statements o f facts in any department ol enquiry placed in relation
to each other’
According to Yule and Kendall, ‘Statistics are quantitative data affected
to a marked extent by a multiplicity of causes’.
The definition given by Prof. Horace Secrist is considered to be the
comprehensive one. According to him ‘Statistics arc aggo-gate of facts affected
to a marked extent by m ultiplicity o f causes, immencally expressed,
enumerated or estimated according to reasonable standard o f accuracy,
collected in a systematic manner, for a pre-determined put pose and placed in
relation to each other’.
A close examination of this definition reveals the following characteristics
that statistics should possess:
a. Statistics are aggregates of facts: Aggregates o f facts relating to a
phenomenon which are comparable with other related aspects can only be
called statistics.

2
Singled and isolated figures relating to production, purchases, sales etc.,
are not statistics.
b. Statistics are affected to a marked extent by multiplicity of causes:
( ieiu'i ally speaking, facts and figures are affected to a considerable extent
by a number o f forces operating together. For example, statistics of
production of wheat are affected by the rainfall, quality o f soil, seeds and
manure, methods o f cultivation etc., So, statistical facts are not attributable
to a single factor.
c. Statistics are numerically expressed: All statistics are numerical statement
of facts i.e., expressed in numbers. Quantitative statements such as ‘India
is a developing country’ or ‘the population of India is rapidly increasing’
do not constitute statistics. Because such statements are vague facts and
one cannot make out anything from them.
d. Statistics are enumerated or estimated according to reasonable
standards of accuracy: Statistics about a given phenomenon can be
collected in two ways namely, enumeration and estimation. Enumeration
refers to the method of survey according to which data are collected for
each and every unit of the universe by actual counting. Estimation refers
to the prediction of values for a universe on the basis of general surveys.
Estimates are not as precise and accurate as enumeration studies. The
degree of accuracy desired depends upon the nature and purpose of study.
Reasonable standards of accuracy must, however be attained, otherwise
data may be altogether misleading.
e. Statistics are collected in dksystematic manner: Before collecting
statistics a suitable plan of data collection should be prepared and the
work carried out in a systematic manner. Data collected in a hasty and
haphazard manner would very likely to lead to misleading conclusions.
f. Statistics are collected for a pre-determined purpose: The purpose of
collecting data must be decided in advance. The purpose should be well
defined and specific. If the data are collected without any aim, much of
the collected data will not be useful. Money, time and energy spent on the
collection o f such data go waste.
g. Statistics are placed in relation to each other: Statistical data are mostly
collected for the purpose of comparison. In order to make valid comparison,
the data should be homogeneous, i.e. they should relate to the same
phenomenon or subject. For example, heights of the boys in a class are to
be compared with the corresponding heights of boys in another class. But
it would be meaningless to compare the heights of students with the heights
of trees.
Thus the definition of statistics given by Prof. Horace Secrist is regarded
as the best as it is exhaustive. It brings out all the characteristics which
statistical data should possess.

3
B. Statistics defined as statistical methods:
According to Bowley ‘Statistics is the science o f the measurements of
social organism regarded as a whole in all its manifestations’.
According to Croxton and Cowden statistics may be defined as the
collection presentation, analysis and interpretation of numerical data.
According to Ya-Lun-chou, ‘Statistics is a method ol decision-making in
the face of uncertainty on the basis of numerical data and calculated risks.
Statistics can also be defined as the science of collection, organization,
presentation, analysis and interpretation of numerical data. According to this
definition, statistical methods include five stages:
i. Collection of data: The first step of an investigation is the collection of
data. Before proceeding for the collection of data, certain aspects like
purpose of the study, sources of data, methods of collection and the degree
of accuracy desired will have to be planned and specified. After this, data
are to be collected systematically. There are different methods of collecting
data (census, sampling, primary, secondary etc.,). If the collected data are
faulty, results will also be faulty. Therefore, the investigator must take
special care in collection of the data.
ii. Organization of data: Data collected from published sources are generally
in an organized form. Mass data collected from a survey is usually in an
unorganized form and requires organization. The first step in the
organization o f data is editing. After the editing, data must be classified
according to same common characteristics of the items. After completing
the editing and classification, data are to be tabulated. In this stage data
are arranged in suitable columns and rows. The main object of tabulation
is to ensure clarity in the data presented.
iii. Presentation of data: The collected data are to be presented in tabular or
diagrammatic or graphic form. The data presented in a systematic order
will facilitate further analysis.
iv. Analysis of data: After the presentation of data, the next step is to analyze
the presented data. Analysis includes condensation, summarization,
conclusion, etc., through the means o f measures of central tendency,
measures o f dispersion, skewness, correlation, regression analysis etc.,
v. Interpretation of data: The last stage in the statistical investigation is
interpretation o f data. Figures do not speak for themselves. The duty of
the statistician is not complete with mere collection and analysis of data.
But, valid conclusions must be drawn on the basis of analysis. A high
degree of skill and experience is necessary for the interpretation. Correct
interpretation leads to valid conclusion.
Importance of statistics:
Statistics affects every one and touches life at many points. In the modern
age figures have become the basis of rational decisions and events are proving

4
dial dec is ions based on figures give better results. As Prof. Bowley said ‘A
know ledge of statistics is like the knowledge of foreign language or of algebra,
ii may prove of use at any time under any circumstances’. Some of the practical
.ipplii .it ions of statistical methods are mentioned below:
statistics in States: In olden days, statistics was regarded as ‘the science
o! kings’. Different policies of the state are based on statistics. Statistics is of
"icat help in promoting human welfare. The main object of every state is to
promote the welfare of the people. Before adopting any welfare scheme, the
state refers the statistics for the proper decision. The state may accept or
reject a policy on the basis of statistics. Periodical collection of data relating
to population, agriculture, national income, education, taxes, exports and
imports etc., are the main guidelines to the governm ent for a good
administration.
Statistics in Economics: Statistical data and methods o f statistical analysis
render valuable assistance in the proper understanding of various problems
such as balance of trade, wage structure, inflation, output of industries,
unemployment, saving and investment, national income etc., and in the
formulation of economic policies. All laws on consumption, production,
exchange and distribution are formulated with the help of statistics.
Statistics in Planning: Statistics is indispensable in planning. The modern
age is termed as ‘the age of planning’ and almost all organizations in the
government or business are resorting to planning for efficient working and
for formulating policy decisions. To achieve this end, the statistical data relating
to production, consumption, investment, prices, income, expenditure and so
on and the advanced statistical techniques such as index numbers, time series
analysis and forecasting techniques for handling such data are of paramount
importance.
Statistics in Business and Industry: Statistical information is required
right from the time business or industry is started. Decision regarding location,
size, production planning, quality control, marketing, sales and output all need
statistical data. To eliminate over production or under production proper
budgeting and costing is required which involves use of statistical methods.
Business runs on estimates and probabilities. An entrepreneur estimates
demand for his product, taking into account various factors such as taste,
habit, income, seasonal changes and adjust his output. His success depends
on correct decisions which in turn depend on reliable statistics.
Statistics to Bankers, Brokers and Insurance companies: Statistical
techniques like percentage, ratios, dispersion etc., are profusely used by the
bankers for evaluation of the credit worthiness of their customers. Knowledge
of probability will be useful to the brokers who are engaged in the purchase
and sale of shares. With the help of statistical methods, insurance companies
calculate financial risk on current life policies which would mature on future
dates.

5
Statistics in other Sciences: Statistics is being used more and more in
biological sciences as an aid of experiments Meteorology uses statistics to
forecast weather. Astronomy uses to predict eclipses.
Statistical methods are of wide almost universal applicability. They have
become very useful tools in the world of affairs
Limitations of Statistics:
The scope of statistics is very wide and it has great utility, but these are
restricted by its limitations. Following are the important limitations of statistics:
i. Statistics deals with only qu an titativ e phenom enon: Statistics is
numerical statement of facts. Statistics deals with only the quantitative
I I data. For example, per capita income, population growth etc., can be studied
by statistics but qualitative aspect such as honesty, poverty, intelligence,
blindness etc., cannot be studied directly.
ii. Statistics does not deal with individuals: Since statistics are aggregates
of facts, the study of an individual fact lies out side the scope of statistics.
For example, the per capita income is obtained by dividing the total income
by the total population. The per capita income does not reveal the poverty
o f individuals. Statistics proves inadequate, where one wants to study
individual cases. Thus it fails to reveal the true position.
iii. Statistical laws are true only on averages: Laws of physical sciences
are perfect. But statistical laws are not so perfect as the laws of physical
sciences. Statistical results are true only on the average. Statistical laws
are based on the theory of probability.
iv. Statistics is only one of the methods of studying problem: Statistical
tools do not provide best solution under all circumstances. Very often, it is
necessary to consider a problem in the light of a country’s culture, religion
and philosophy. Statistics cannot be of much help in studying such
problems. Hence statistical conclusions should be supplemented by other
evidences.
v. Statistics can be misused: The greatest limitation of statistics is that they
are liable to be misused. In this context, Prof.W.I.King pointed out, ‘one
of the short comings of statistics is that they do not bear on their face the
label of their quality’. So, it requires experiences and skill to draw sensible
conclusions from the data, otherwise, there is every likelihood of wrong
interpretations.
Collection of data:
Collection of data implies a systematic and meaningful assembling of
information for the accomplishment o f the objectives o f a statistical
investigation. It refers to the purpose of gathering of relevant information to
the subject matter of the study from the units under investigation. Collection
of data is the most important stage in the process of statistical investigation.

6
Source of data:
I lie tl.ua for the purpose of statistical investigation may be collected from
in imiiiy and secondary sources.
I i.u.i c.cncrated from a primary source is called primary data and data
i n il.. led Irom a secondary source may be referred to as secondary data. In
in oilier words, data originally collected in the process of investigation is
Known as primary data and that collected by the other person is known as
secondary data.
Methods of collecting prim ary data: Primary data may be collected by the
following methods:
i. Direct personal interviews
ii. Indirect oral interviews
iii. Information from agencies
iv. Questionnaires
v. Schedule
i. Direct personal interviews: Under this method, the data are collected by
the investigator personally. The investigator must be keen observer, tactful
and polite in behavior. He asks or cross-examines the informat and collects
necessary information. The enquiry is intensive, rather than extensive.
ii. Indirect oral interviews: Under this method, the investigator conducts
interviews with several third party informats who are supposed to have
the knowledge about the problem under investigation. Generally this
method is employed by different enquiry committees and commissions.
The police department generally adopts this method to get clues o f thefts,
riots, murders etc. ^
iii. In fo rm a tio n from agencies: In this method, persons known as
correspondents or local agents are appointed at different places to collect
data. These correspondents gather information regularly and send the same
to their central office for further processing and analysis.
iv. Q uestionnaires: In this method, a questionnaires consisting of a list of
questions pertaining to the enquiry is prepared. There are blank spaces for
answers. These questionnaires are sent through the post to the respondents
with a request to fill them up and return them to the investigator. This
method is adopted by research workers Private individuals, State and
Central Governments.
v. Schedule: Under this method a group of enumerators are asked to collect
data through schedules. A schedule is a form containing specific questions,
relating to the problem under investigation. The schedules are personally
served on the respondents and their answers are recorded by the
enumerators.

7
CONSTRUCTION OF A QUESTIONNAIRE OR SCHEDULE:
Questionnaire or Schedule occupy an important place in the collection of
primary data, since the success of the collection of relevant, adequate and
accurate data depends upon the careful and cautious preparation o f the
questionnaire or schedules. Although no hard and last rules exists for the
designing of the questionnaire the following essential general principles can
be followed.
a. The questionnaire should be brief. The number o f questions in the
investigations must be minimum and at the same time, they must fully
cover the object and purpose of the investigation
b. The questions should be simple to understand, questions should be prepared
in simple, clear and straight terms, so that, the meaning may be understood
in its proper perspective.
c. Questions should be arranged logically.
d. Questionnaire should consists of simple alternate questions and multiple
choice questions.
e. Questions of a sensitive and personal nature should be avoided.
f. Questions related to mathematical calculations should not be asked.
g. Questions should be capable of objective answer.
h. The questionnaire should look attractive and impressive.
i . , Necessary instructions for filing up the questionnaire must be given to
guide the respondents.
j. A covering letter along with the questionnaire should be sent by the
organizers to the respondents informing the purpose o f enquiry and
requesting them to give their co-operation. They should be assured that
the answers given by them will be kept confidential.
k. Pre-testing the questionnaire must be done before posting it. The
investigator can find out the defects of the questionnaire and suitable
changes can be made before it is send to a large number o f respondents.
l. The accuracy of the questionnaire must be judged. After pre-testing the
questionnaire, we can judge the accuracy of the questionnaire. Thus we
can know the accurate picture of information.
Sources of secondary data: Secondary data are those data which have been
already collected and analysed by same earlier agency for its own use and
later the same data are used by a different agency. The various sources of
secondary data are
1. Published sources
2. Unpublished sources
1. Published Sources: published sources o f secondary data can be obtained
from the following

8
it) G overnm ent Publications: Departm ents o f the union and state
I'.nvi-i nmcnts regularly publish reports on a number of subjects. They gather
additional information. Some ofthe important publications are :The R.B.I.
Bulletin, Census of India, Statistics Abstracts of States etc.
i>) l“nl>li«ilions of Semi-Government Organizations: Semi government
institutions like Municipal corporations, District Boards, Panchayat etc.,
Publish reports.
c) Publications of Research Institutions: Various research institutes publish
reports on the projects undertaken by them. Indian statistical institutions,
Institute of Labour Research, Various Universities etc are some of the
examples of research institutes bring out publications
d) Other Institutional Sources: Various public and private, commercial and
financial institutions such as the Institute of Chartered Accounts of India,
Stock Exchanges, State Financial Corporations, Banks etc, also publish
reports on aspects related to their activities
e) Newspapers and Journals: Current and Important materials on statistics
and socio-economic problems can be obtained from newspapers and
journals like. The Economic times, The Financial Express, Commerce,
Capital, Indian Finance etc.
f) International publications: International agencies and international
bodies publishes regular and occasional reports on economic and statistical
matters. They are The I.M.F, The I.B.R.D., The U.N.O. etc.
2. Unpublished Sources: Unpublished sources constitute the internal records
o f private organizations, results%)f research carried out by individual
researchers etc. The investigator has to visit their offices and take down such
information from documents and records.
SAMPLING: Statistical data can be collected either by census method or by
sampling method. If it is needed to study all the objects of a universe, census
method is followed. On the other hand, if it is proposed to study only a few
objects of the universe and draw conclusion concerning the entire universe,
sampling method is followed.
CENSUS METHOD: In a census method, information is collected for each
and every unit ofthe population or universe. The group of individuals or unit
under study is known as population or universe. For example, if a study
regarding wage pattern of workers in the Tata Iron and Steel Company includes
all the workers of the organizations such number is known as population.
Merits & Limitations of Census Method:
Merits
i. As the information is collected from every unit of the population the results
tend to be more accurate and reliable
ii. Intensive study is possible
iii. The data collected may be used for various surveys, analyses, etc.
9
Limitations
i. It requires large number of trained personnel and it is very costly method.
ii. It requires more money, labour, time, energy etc.
iii. It is not possible in some circumstances where the universe is infinite.
SAMPLE METHOD: This technique studies a part o f the total elements of
the universe and the results obtained on the basis of study are applied to the
universe from which the sample is drawn.
M erits & Limitations of Sample method:
M erits
i. As the data is collected from a part of the universe, it is economical.
ii. It saves time, because fewer items are collected and processed. When the
results are urgently required, this method is very helpful.
iii. If the population under study is infinite, sampling method is the only method
to be used.
iv. Often, the sampling method is used to test the accuracy of the results
obtained by the Census Method.
Limitations
i. A sample enquiry if not carefully planned and executed, the conclusions
may be inaccurate and misleading.
ii. As there is lack of trained personnel to plan and conduct a sample survey,
Its execution and analysis, the results o f the sample survey are not
satisfactory and trustworthy.
iii. There is organizational problem in sample investigation.
iv. The selection of sample and determination of sample size pose serious
problems.
THEO RIES OF SAMPLING: In statistics, where complete enumeration is
not possible, conclusions are drawn by studying the sample on the assumption
that the sample units are representative o f the population. This assumption is
based on the following two theories of sampling.
(i). Law of statistical regularity and
(ii) . Law of inertia of large numbers
LAW OF STATISTICAL REGULARITY:
According to W.l.KING, “The Law of Statistical Regularity lays down that a
moderately large number of items chosen at random from a very large group
are almost sure on the average to have the characteristics of the large group"
LAW OF INERTIA O F LARGE NUMBERS:
This law is also known as ‘Principle of stability of mass data’ is developed
from the law of statistical regularity. Th is law is based on the assumption that
the larger the size of data, the lesser the fluctuations, because of the cancellation
effect. It means that the moderately large number of items would balance the
variations of small observations. Thus, the law o f inertia of large numbers
10
- ii. ill it hy increasing the size of the sample, the results can be made more
i'' intilt*
I I '. ii U S OF SAMPLING:
fli.l ei|i.i in r of sampling is to draw inferences about the parent population,
din h mlriences would be true, if the sample has the following essentials.
. 11 Representativeness: Representativeness implies that the sample should
possess the characteristics of the population from which it is drawn.
(ii) . Homogeneity: Selected samples from the universe should have the similar
nature and should not have any difference, when compared with the
universe.
(iii) . Independent: While selectingthe sample, every unitmust be independent
of each other in order to be included in the sample.
(iv) . Adequacy: The size of the sample must be adequate to yield accurate
results. If a large number of items are included in the sample, they will
truly represent the universe.
METHODS OF SAMPLING:
There are many methods of sampling. The choice of method will be determined
by the purpose of sampling. The various methods can be grouped under two
groups:
1. Random Sampling:
(A) . Simple or unrestricted random sampling
(B) . Restricted Random sampling
2. Non-Random Sampling:
1. Random Sampling: A random sample is one where each item in the
universe has an equal chance o f known opportunity of being selected.
A. Simple Random Sampling
It is a technique in which sample is so drawn that each and every unit in
the population has an equal and independent chance of being included in
the sample. Several methods have been adopted for random selection of
the sample. They are:
(i) Lottery method. This is the most popular and simplest method; all
the items of the universe are numbered on separate slips of paper of
same size, shape and colour. They are folded and mixed up in a drum
or container. A blindfold selection is made. The selection of items
thus depends on chance.
(ii) Table of Random Numbers. As the lottery method cannot be used,
when the population is infinite, the alternative method is that of using
the table of random numbers. There are several standard tables of
random numbers. But the credit for this technique goes to prof.
L.H.C.Tippett (1927). The random number table (taken from the

II
British census Report) consists of 10,400 four-figure numbers giving
in all 10,400x4=41,600.There are various other random numbers.
They are Fisher and Yates (1938) comprising o f 15,000 digits arranged
in twos, Kendall and B.B.Smith (1939) consisting of 1,00.000 digits
grouped in 25,000 sets of 4 digited random numbers etc.
B. Restricted Random Sampling:
(i) Stratified sampling: When the population is heterogeneous with respect
to the variable under study, then it is stratified.
First the population is divided into a number of sub-groups or strata.
Each stratum is homogeneous. A sample is drawn from each stratum at
random.
There are two types of stratified random sampling. They are proportional
and non-proportional. In the proportional sam pling, equal and
proportionate representation is given to sub-groups. If the number of items
is large in the population, the same will have a higher size and vice versa.
In non-proportionate sample, equal representation is given to all the sub­
strata regardless o f their existence in the population.
(ii) Systematic sampling: A systematic sample is selected at random
sampling. When a complete list of the population is available, this method
is used. We arrange the items in numerical, alphabetical, geographical or
any other order. If we want to select a sample of 100 students from 1,000
students, under this method kth item is picked up from the sample frame
and k is the sample interval.
k =N/n k=Samp!ing interval N=Size o f universe n=Sample size
(iii) Cluster sampling: It refers to a sampling procedure, which is carried out
in several stages. The whole population is divided into sampling units,
and these units are again divided into sub-units. This process will continue
when we reach a least number.
2. Non-random sampling:
(a) Judgement sampling: The investigator has the power to select or reject
any item in an investigation. The choice of sample items depends on the
judgement of the investigator. He has the important role to play in collecting
the information.
(b) Quota sampling: This sampling is similar to stratified sampling. To collect
data, the universe is divided into quota according to some characteristics.
Each enumerator is then told to interview a certain number of persons
who are in his quota. The selection of sample items depends on personal
judgement.
(c) Convenience or Chunk sampling: Chunk is a convenient slice of a
population which is commonly referred to as a sample. It is obtained by
selecting convenient population units.

12
A .ample obtained from automobile registration, telephone directories, etc.,
is a convenience sample
I ABDICATION: Tabulation is the process of systematically arranging the
dal.i m columns and rows. Rows are horizontal arrangement where as columns
.in- vertical ones. A statistical table may be defined as follows “A statistical
table in the logical listing of related quantitative data in vertical columns and
horizontal rows of numbers, with sufficient explanatory and qualifying words,
phrases and statements in the form of titles, heading and notes to make clear
the full meaning of the data and their origin.”
Thus the statistical table is a condensed summary of the original data. It is
an orderly arrangement designed to give maximum possible information
leaving out unimportant and irrelevant figures.
General rules for tabulation: It is difficult to lay down any hard and fast
rules for tabulating because much depends upon the given data and
requirements o f the survey. Infact, constructing a good table is an art, and
therefore a practical experience is of immense help. However, the following
general consideration may be kept in view while tabulating data:
1. Every table should bear a suitable title. The title should be clear and
explanatory. Every table should be numbered so that it can be easily
identified.
2. In all tables the captions and stubs should be arranged in some systematic
order.
3. There should be proper ruling and spacing to be attractive.
4. The table should suit the size of the paper.
5. If certain figures are to be emphasized they should be in, distinctive type
or in a box or between thick lines.
6. The table should not be overloaded with details.
7. A miscellaneous column should be added for data which do not fit in the
classification made.
8. The arrangement of the table should be logical.
9. Percentages and ratios which are meant for comparison should be placed
adjacent to one another.
10. The sources o f data should be mentioned.

13
SPECIMEN FORM OF A TABLE
Table Number
Title Head Notes
Captions
Stub Column heading Column heading
Heading
x caption y caption x caption y caption
Stub A
Stub B Bo dy
Stub C

Total
Foot note Sources of the data
Parts of a table:
1. Table number: Every table should be numbered. The mat be given either
in the centre at the top above the title or in the side of the top or in the
bottom of the table on the left hand-side.
2. Title of the table: Every table must have a suitable title. The title should
be clear, brief and self-explanatory. Long titles cannot be read as promptly
as short titles. It should be in the form of a series of phrases rather than
complete sentences.
3. Captions: Captions refers to the column heading. Under a column heading
there may be a sub-heads. The caption should be clearly defined and placed
at the middle of the column. If the different columns are expressed in
different units, the units should be mentioned with the captions. As
compared with the main part of the table the caption should be shown in
the smaller letters.
4. Stub: Stubs are at the extreme left and perform the same functions for the
horizontal rows of the numbers in the table as the columns headings do for
the vertical columns of numbers. The stubs are usually wider than column
heading but should be kept as narrow as possible without sacrificing
precision and clarity of the statements.
5. Body: The body o f the table contains the numerical information. This is
in fact main part of the table. Data presented in the body are arranged
according to description or classification of the captions and stubs.
6. Head Note: It is a brief explanatory statement applying to all or major
part of the material in the table, and is placed below the title centered and
enclosed in brackets. It is used to explain certain points relating to the
whole table have not been included in the title, captions or stubs.
7. Foot Note: A foot note is a phrase or statement which classifies some
specific items or explains the omission thereof. It is placed on the bottom
of the table.

14
QUESTIONS
i I n-.tir.s die meaning and scope of statistics.
■ VVhut are Statistics? Describe their characteristics.
i '.late a few of the definitions of statistics and explain the one which you
think to be the best.
I Define ‘Statistics’ and discuss its importance and limitations.
5. Discuss briefly the importance of statistics in the following disciplines:
(i)Economics (ii) Planning (iii) Business and Industry (iv) Bankers, Brokers
and Insurance companies (v) Other Sciences.
6. Discuss the utility of statistics to the state.
7. Write a critical note on the origin and development of the science of
statistics.
8. Statistics may be described as the science of collection, organization,
presentation, analysis and interpretation of numerical data- Discuss.
9. Differentiate between primary and secondary data and discuss the various
methods of collecting Primary data.
10. What is a Secondary Source? What are the chief sources of Secondary
data.
11. What do you mean by a Questionnaire? Discuss the essentials of a good
Questionnaire. ^
12. Distinguish between census and sampling methods of collecting data and
compare their merits and demerits.
13. Define clearly the Law of Statistical Regularity and Law of Inertia of Large
Numbers.
14. What are the essentials of Sampling.
15. Discuss the various methods of random sampling.
16. Enumerate the various methods of non-random sampling.
17. What do you mean by “Classification of Data”? Explain the various bases
of classification.
18. What do you mean by seriation of data? Explain in detail the various types
of series.
19. Distinguish between cumulative frequency by ‘Less than method’ and ‘more
than method’.
20. Explain Tabulation? What are the essentials of a good Statistical Table?

15
CHAPTER I

Classification of Statistical Data


In any statistical investigation, after collection and editing of
data the first step towards further processing the same is classifica­
tion. Classification refers to grouping of data into homogeneous
class and categories.

Broadly, the data can be classified on the following four basis:

1. Geographical, i. e., area-wise.


2. Chronological, i.e., on the basis of time.
3. Qualitative, i.e., on the basis of some attributes.
4. Quantitative, i.e., in terms of magnitudes.

Statistical Series :
According to H. Secrist "A series, as used statistically, may
be defined as things or attributes of things arranged according to
some logical order."

Statistical series can be classified into three classes from one


point of view namely. Series of Individual Observations, Discrete
Series and Continuous Series.

(1) Individual Observations : A series of individual o


vations is a series where items are listed singly after observation,
as distinguishing from listing them in groups. If marks of twenty
students are individually given in a series, it will constitute a series
of individual observations. The following is an example of a aeries
of individual observations

Maiks obtained by five students


Roll Nos. Marks
1 20
2 40
3 50
4 40
5 45
2
(2) Discrete Series : When items are arranged in group
allowing definite breaks from one point to another, and when they
mu oxaetly measurable, they constitute a discrete series.

The following procedure must be followed for formation of


a Discrete Frequency Distribution.
(a) In the first column, place all possible values of variables
from the lowest to the highest, or highest to the lowest.
(b) In the second column (Tallies column), put a bar (vertical
line) opposite the particular value to which it relates, to facilitate
Counting, blocks of five bars are prepared and some space is left in
between each block.
(c) In the third column (frequency column), we finally count
the number of bars corresponding to each value of the variable and
place It in this column.

Illustration 1 : In a survey of 30 families in a village, the


number of children per family was recorded and the following data
obtained :
1 0 2 3 4 5 6 7 2 3 4 0 2 5 4
5 6 3 ^ 7 6 5 3 3 7 7 4 5 4 3

Represent the data in the form of a discrete frequency distri­


bution.
Solution :
Frequency Distribution of the Number of Children :
No. of Children Tallies Frequency
0 I I 2
1 . I 1
2 I I 11 4
3 mji 6
4 rw ■ 5
5 mi 5
6 111 3
7 m 11 4
Total 30
3

(3) Continuous Series : When items are arranged


groups or classes because they are not exactly measurable, they form
a continuous series. This type of classification is most popular in
practice. The follow ing technical terms are important when a conti- f
nuous frequency distribution is formed.

(a) Class In te rv a l: A large number of observations varyin


in a wide range are usually classified in several groups according to
the size of value. Each of these groups defined by an interval is
called class interval or simply class. For example, in the class
10-20, the interval is 10 (i.e., 10-20).

An important decision while constructing a freqency dis­


tribution is about .the width of the class interval, i.e., whether it
should be 10, 20, 50, 100 etc The decision would depend upon a
number of factors. A simple formula to obtain the estimate of app­
ropriate class interval i.e., i is

L -S

where L — Largest item,

S — Smallest item,
.-in i;>• '■ r.
K «=- the number of classes.

(b) Class Lim its : The class limits are the lowest and th
highest values that can be included in the class. For example, take
the class 1 0-20. The lowest value of the class is 10 and the
highest 20. The tw o boundaries of a class are known as the lower
limit and the upper lim it of the class. The lower limit of a class is
the value below which there can be no item in the class. The upper
limit of a class is the value above which no item can belong to that
class-. The way in which class limits are stated depends upon the
nature of data. In statistical calculations, lower limits are represen­
ted b y " /1" and the upper limits by " / a".

There are different ways in which limits of class intervals are


shown. We may represent class limits in any of the following forms :
4

I II III IV
(Exclusive type) (Inclusive type) (open end classes)

10 nnd under 20 10-20 10-19 Below 20

20 and under 30 20-30 20-29 20-30

30 and under 40 30-40 30-39 30-40

40 and under 50 40-50 40-49 40-50

50 and under 60 50-60 50-59 50-60

The definition of class limits in (1) is very clear. The assump­


tion in (II), i.e., exclusive type is that a value equal to the lower limit
should be included in the class interval and a value equal to the upper
limit should be excluded. In (III), i.e., inclusive type, the overlapping
of class intervals is avoided both the lower and upper class limits are
included in the class intervals. In(IV), i.e., open end classes, a class
limit is missing either at the lower end of the first class interval or at
the upper end of the last class interval or at both the ends.

(c) Class Frequency: The number of observations corres­


ponding to a particular class is known as the frequency of that class.

(d) Class marks or (mid-valuers) : It is the value lying half­


way between the lower and upper class limits of a class interval.

Lower class lim it-f Upper class limit


Mid-Point of a class *=
2 ’

Illustration 2 : Prepare a frequency table given the follow­


ing data of wages earned by 30 workers.

87 87 94 146 106 112 106 69 125 119


TOO 137 150 87 131 119 100 75 106 137
100 80 75 125 112 125 94 131 112 144

Solution : The lowest value is 69 and the highest value 150.


The difference in the highest and the lowest value is 81. If we take
■1
5

a class interval of 10, nine classes would be formed. The first class
should be taken as 6 5 -7 5 .

Frequency distribution of the wages of 30 workers.

Wages Tally Bars Frequency

6 5 -7 5 I 1
7 5 -8 5 II1 3
8 5 -9 5 fHI 5
9 5 -1 0 5 11 1 3
1 0 5 -1 1 5 m 6
1 1 5 -1 2 5 11 2
1 2 5 -1 3 5 fHI ■ 5
1 3 5 -1 4 5 111 o
1 4 5 -1 5 5 11 2
Total 30

Illustration 3 : Prepare a frequency table taking class


imervals 1 5 -1 9 , 2 0 -2 4 and so on from the following data (inclusive
method).

30 54 15 42 30 36 37 31 36 31 16 21 33
42 40 17 25 27 28 54 36 22 19 42 22 41
30 48 51 41 42 26 44 40 30 48 32 40 21

Frequency Distribution

Variable Tally Bars Frequency

1 5 -1 9 III1 4
2 0 -2 4 1 1 11 4
2 5 -2 9 111 1 4
3 0 -3 4 IH1 I I I 8
3 5 -3 9 m i 4
4 0 -4 4 mi mi 10
4 5 -4 9 11 2
5 0 -5 4 111 3
Total 39
6
Preparation o f c o n tin u o u s F req u en cy D istrib u tio n : If
any in s tru c tio n s are given in th e problem , th e n p rep are th e
table accordingly. In th e a b s e n c e of a n y in s tru c tio n s , th e
following g u id e lin e s can be followed :

(ij F ind o u t th e m in im u m an d th e m axim um v alu es in the


d a ta to be classified. T his will h elp in draw ing u p th e
c la ss e s to acco m m o d ate all th e v alu es.

(ii) D ecide th e n u m b e r of c la ss-in te rv a ls. The n u m b e r of


c la ss in te rv a ls s h o u ld n e ith e r be too large n o r too
sm all. The n u m b e r m ay vary b etw een 5 arid 20.

(iii) The c la ss -in te rv a ls s h o u ld be of e ith e r 5 or m u ltip les


of 5.

(iv) U n less o th erw ise s ta te d th e d a ta is to be classified


u n d e r E xclusive M ethod.

(v) In the table place th e class-in terv als in the first colum n,
tally m a rk s in th e seco n d co lu m n , an d co rresp o n d in g
freq u en cy rin th e th ird colum n.

(Note : Refer Illu s tra tio n s 2 a n d 3)

C u m u la tiv e F r e q u e n c y D is tr ib u tio n : A F re q u e n c y
d is trib u tio n sh o w s how th e freq u en cy of a p a rtic u la r value
(variable or class) is occu rrin g . However, if we w ant to know
th e to tal n u m b e r of o b serv atio n s g ettin g a v alue Less th a n '
or More th a n ' a p a rtic u la r v alu e of th e variab le or class,
th is freq u en cy tab le fails to fu rn is h th e in fo rm atio n as
su c h . T his in fo rm atio n can be o b ta in ed very easily from th e
"Cum ulative F req u en cy D istrib u tio n ' w hich is a m odification
of th e given freq u en cy d is trib u tio n a n d is o b tain ed on
su ccessiv ely ad d in g th e freq u en cies of th e v a lu e s of th e
v ariable or class acco rd in g to a c ertain law. The freq u en cies
so o b ta in e d a re k n o w n a s th e cu m u la tiv e fre q u e n c ie s
ab b rev iated a s c.f. The cu m u lativ e freq u en cy d is trib u tio n is
of two types.
6a
(a) Less th a n cu m u lativ e freq u en cy d is trib u tio n .
(b) More th a n cu m u lativ e freq u en cy d is trib u tio n .
In case of Less th a n ' m e th o d , we s ta r t w ith u p p e r lim its
of th e c la sse s an d go on ad d in g th e freq u en cies. In case of
More th a n ' m ethod, we s ta rt w ith lower lim its of th e classes.
Total freq u en cy is w ritten a g a in s t th e first n u m b e r, th e n
th e frequency of th e first class is su b tra c te d a n d th e b alan ce
is w ritten ag ain st th e next class, th e n th e p ro cess is repeated
for th e rem a in in g classes.

Illu stration 4 : Convert th e following freq u en cy d istrib u tio n


into cu m u lativ e freq u en cy d is trib u tio n by (a) Less th a n '
m eth o d a n d (b) More th a n ' m e th o d

Marks No. o f s tu d e n ts
10-20 4
20-30 6
30-40 10
40-50 20
5 0 -6 0 ' 18
60-70 2

S olu tion (a) : F orm ation of a Less th a n cum ulative frequency


d istrib u tio n .
Marks (f) c.f.

Less th a n 20 4 4
Less th a n 30 4+6 10
Less th a n 40 10+10 20
Less th a n 50 20+20 40
Less th a n 60 40+18 58
Less th a n 70 58+2 60
6b
(h) F o rm a tio n of a m o re th a n C u m u lativ e F req u en cy
D istribution:
Marks (f) c.f.
More th a n 10 (Table freq u en cy 60) 60
More th a n 20 (60-4) 56
More th a n 30 (56-6) 50
M ore th a n 40 (50-10) 40
More th a n 50 (40-20) 20
More th a n 60 (20-18) 2
More th a n 70 0

Illu stra tio n 5: C onvert th e following cu m u lativ e frequency


d is trib u tio n in to sim ple freq u en cy d istrib u tio n .
W eekly w ages No. o f Workers
le s s th a n (in Rs.)
20 41
40 92
^ 60 156
80 194
100 201

S o lu tio n : C onversio n of cu m u lativ e freq u en cy d is trib u tio n


in to sim ple freq u en cy d is trib u tio n
W eekly w a g es (in Rs.) No. o f w orkers (f)

0-20 41
20-40 92-41=51
40-60 156-92=64
60-80 194-156=38
80-100 201-194=7
201
6c
•. •i ArM
Illu stra tio n 6 : C onvert th e following cu m u lativ e frequency
into sim ple freq u en cy table.

Marks No. o f s tu d e n ts
1
More th a n 30 70
More th a n 35 65
More th a n 40 55
More th a n 45 40
More th a n 50 10
More th a n 55 5
More th a n 60 0

S o lu tio n : C onversion of cu m u lativ e freq u en cy in to sim ple


freq u en cy table.

Marks F requ en cy

30-35 70-65=5
35-40 65-55=10
40-45 55-40=15
45-50 40-10=30
50-55 10-5=5
55-60 5-0=5
70
...
PROBLEMS

P r e s e n t th e d a t a giv en in t h e fo rm of a d is c r e te
fre q u e n c y d i s t r ib u t io n

8 0 2 0 9 3 10 2 4 10 6 5

3 2 5 1 5 0 3 8 7 4 2 3 2

The follow ing d a ta sh o w s th e p re v io u s te a c h in g


experience of te ach ers in a school. P repare a Frequency
d istrib u tio n tab le in D iscrete form.

2, 3. 4. 10, 6, 3, 5, 5, 7, 3, 3, 2, 12, 8, 9, 2, 10, 5, 2


11, 8, 12, 4, 2, 12, 3, 5, 7. 5, 9.

C o u n t th e n u m b e r of le tte rs in eac h w ord of th e


p ara g ra p h given below (ignoring p u n c tu a tio n m arks)
an d p rep are a d iscrete freq u en cy d is trib u tio n .

"A Frecfuency d is trib u tio n refers to d a ta classified on


the b a s is of som e variab le th a t can be m e a su re d s u c h
as prices, w ages, age, n u m b e r of u n its p ro d u ced or
c o n s u m e d . A v a ria b le m ay be e ith e r d is c re te or
continuous". (Ans : N = 36)

C o u n t th e n u m b e r of le tte rs in each w ord of th e


p ara g ra p h given below (ignoring p u n c tu a tio n m arks)
an d p rep are a d iscrete freq u en cy d is trib u tio n .

"When th e clock strik e s, one h o u r is s h o rte n e d from


o u r life-span. W ith th e p a ssin g of each m in u te, we are
m oving clo ser to d eath . We do n o t know w h en th e
telegram of d e a th will grab u s : - acc id en t, d isease or
old-age? H ence, w h en th e body is fit an d h ealth y ,
before th e s e n s e s lose th e ir n a tu ra l p ro p e n sitie s a n d
before d e a th o v ertak es life, striv e to a tta in God.
6c
Illu stra tio n 6 : C onvert th e following cu m u lativ e frequency
into sim ple freq u en cy table.

Marks No. o f s tu d e n ts
(j
More th a n 30 70
More th a n 35 65
More th a n 40 55
More th a n 45 40
More th a n 50 10
More th a n 55 5
More th a n 60 0

S o lu tio n : C onversion of cu m u lativ e freq u en cy in to sim ple


freq u en cy table.

' Marks F requ en cy

30-35 70-65=5
35-40 65-55=10
40-45 55-40=15
45-50 40-10=30
50-55 10-5=5
55-60 5-0=5

j pi >,i ■■ -i
i . . 70
PROBLEMS

P r e s e n t th e d a t a giv en in t h e f o rm of a d is c r e te
fre q u e n c y d i s t r ib u t io n

8 0 2 0 9 3 10 2 4 10 6 5

3 2 5 1 5 0 3 8 7 4 2 3 2

T he follow ing d a ta s h o w s th e p r e v io u s te a c h in g
experience of te ach ers in a school. P repare a F requency
d is trib u tio n tab le in D iscrete form.

2, 3, 4, 10. 6. 3, 5. 5. 7, 3, 3, 2. 12. 8. 9, 2, 10. 5, 2


11, 8, 12, 4, 2. 12. 3. 5. 7. 5, 9.

C o u n t th e n u m b e r of le tte rs in eac h w ord of th e


p a ra g ra p h given below (ignoring p u n c tu a tio n m arks)
a n d p rep are a d iscrete freq u en cy d is trib u tio n .

"A F req u en cy d is trib u tio n refers to d a ta classified on


the b a s is of som e v ariab le th a t can be m e a su re d s u c h
as prices, w ages, age, n u m b e r of u n its p ro d u ced or
c o n s u m e d . A v a ria b le m ay be e ith e r d is c re te or
continuous". (Ans : N = 36)

C o u n t th e n u m b e r of le tte rs in each w ord of th e


p a ra g ra p h given below (ignoring p u n c tu a tio n m arks)
an d p rep are a d iscrete freq u en cy d is trib u tio n .

"When th e clock strik e s, one h o u r is s h o rte n e d from


o u r life-span. W ith th e p a ssin g of each m in u te, we are
m oving clo ser to d eath . We do n o t know w h en th e
telegram of d e a th will g rab u s : - acc id en t, d ise a se or
old-age? H ence, w h en th e body is fit a n d h ealth y ,
before th e s e n s e s lose th e ir n a tu ra l p ro p e n s itie s a n d
before d e a th ov ertak es life, striv e to a tta in God.
Is th ere any g u aran tee of obtaining' th is precious h u m an
b irth ag ain ? No! o p p o rtu n ity lo st will n o t re tu rn to u s
again. H ence, live every m in u te a n d do y o u r d u ty
correctly". (Ans : N = 92)

From th e following o b serv atio n s, p rep are a frequency


d istrib u tio n table in ascen d in g o rder sta rtin g w ith 100-
110 (exclusive m ethod)
125 108 112 126 110 132 136 125
120 130 136 138 125 111 119 132
147 137 145 160 142 135 137
155 148 154 149 140 165 130

From th e following o b se rv a tio n s p rep are a classified


freq u en cy d is trib u tio n :
110 108 126 132 149 136 125 112
138 155 125 138 136 130 120 148
140 125 119 111 154 147 165 137
145 132 150 137 142 135 125 126

M arks o b tain ed by 50 s tu d e n ts of a class are a s under:


- f.r ufji’i i
34 54 10 21 51 52 12 43 48 30
48 22 39 26 34 19 10 17 47 43
13 30 30 60 59 15 7 18 40 25
40 51 55 32 41 22 30 35 53 42
14 18 19 40 43 4 17 45 25 34

C o n s tru c t a freq u en cy ta b le w ith class in terv al 0-9,


10-19, 2 0 -2 9 etc.
6f
H. From a freq u en cy d is trib u tio n from th e follow ing d a ta
by inclusive m e th o d ta k in g 4 a s th e m a g n itu d e of
c lass in terv als.
10 17 15 22 11 16 19 24 29 18 25 26
23 27 30 12 15 18 15 21 28 33 38 34
13 10 16 20 22 29 19 23 31 14 32 14
17 20 24 36

9. The following d a ta p e rta in s to a p o s t office.


N o.of p ack ages No. o f tim e s receiv ed
received in a m o n th
0-10 8
10-20 3
20-30 4
30-40 s* 5
40-50 8
50-60 2

P repare cu m u lativ e freq u en cy tab le acco rd in g to (i)


Less th a n m e th o d (ii) More th a n m eth o d .

10. From th e following freq u en cy tab le p rep are cu m u lativ e


frequency tab le acco rd in g to (a) Less th a n m e th o d (b)
More th a n m eth o d
Marks No. o f s tu d e n ts
0-10 8
10-20 10
20-30 12
30-40 40
40-50 56
50-60 44
60-70 26
70-80 4
11. C o n v e rt th e fo llow ing in to sim p le fre q u e n c y ta b le s
No. o f D ays a b sen t No. of stu d e n ts

Less th a n 5 29
Less th a n 10 224
Less th a n 15 465
Less th a n 20 582
Less th a n 25 634
Less th a n 30 644
Less th a n 35 650
Less th a n 40 653
Less th a n 45 655

12. C onvert th e following' in to sim ple frequency tab le an d


m ore th a n cu m u lativ e freq u en cy table.

S ize F requency
B elow 10 5
Below 20 20
Below 30 50
Below 40 105
Below 50 190
Below 60 250
Below 70 295
-1
1■ •
Below 80 320 '
6h
l '• C onvert th e following' c u m u la tiv e fre q u e n c y d is trib u tio n
into sim ple freq u en cy d istrib u tio n .
Marks above No. o f s tu d e n ts
o ' . 80
10 77
20 72
30 65
40 55
50 43
60 28
70 16
80 10
90 8
100 0

14. C onvert th e following in to sim ple freq u en cy tab le an d


Less th a n cu m u lativ e freq u en cy table.
^ Marks No. o f s tu d e n ts
\ More th a n 20 200
More th a n 30 195
jMore th a n 40 177
$
jMore th a n 50 157
iiMore th a n 60 142
Jviore th a n 70 124
More th a n 80 84
;More th a n 90 16
----------- L------- -- -------------------
i
f

I
f
1
•f

j
C H A P T E R il

treasures of Central Tendency


i
One of the most important objectives of statistical analysis
is to get one single value that describes the chief features of the
entire mass of unwieldly data. Such a value is known as the central
value or an 'average.
1
The meaning of average is nicely gi ven in the following defi­
nitions :
"A n average is a typical value in the sense that it is some
times employed to represent all the individual values in ? series
or of a variable." — Ya-Lun-Chou
"A measure of centra! tendency is typical value around which
other figures congregate." —Simpson and Kafka
It is clear from the above definitions that, an average is a
single value that represents a group of values.
,
The following are the important types of averages used in j
statistical analysis :
1. Arithmetic Mean : (i) simple, and (ii) weighted.
2. Median
3. Mode
4. Geometric Mean
5. Harmonic Mean

1. Arithmetic Mean
Its value is obtained by adding together all the items and by
dividing this total by the number of items. Arithmetic mean mo/
either be
• (i) Simple arithmetic Mean or
(ii) Weighted arithmetic mean
- , - .i; ■ /]
( i) Calculation o f Simple Arithm etic Mean ;
The simple Arithmetic Mean can be computed by making use
of tw o methods namely (a) Direct Method and (b) Short cut Method ;
8
<n Indirect Method in the case of three series namely (A) Series of
individual observations (B) Discrete series and (C) Continuous series-

(A) Series of Individual observations :


(a) Direct Method The formula for calculating mean is

tf
X== N

sum : The formula involves tw o steps in calculating mean :
f'i) Add together all the values of the variable X and obtain
the total, i.e., 2 X .
(ii) Divide this total by the number of observations, i.e., N.
Where
X = Arithmetic Mean
2X ■= Sum of all the values of v a r i a b l e X.
N •=» Number' of observations.
(b) Short-cut Method The formula for calculating mean is
j: d
A +
N

Steps : The following steps are required


(i) Take an assumed mean.
(ii) Take the deviation of items from the assumed mean and
d e n o te these deviations b y "d'
(Mi) obtain the sum of these deviations, i.e.,’ 2 d -

flv) Apply the formula : X = A 2d


N
Where
X — Arithmetic Mean :d=X-A
A — Assumed Mean
2 d =* Sum to ta l of deviations of items from an assu­
med mean.
IV =■ Number of observations.
9
Illustration 1. From th 8 following marks obtained by 6
students compute the Arithmetic Mean by (a) Direct Method and (b)
Indirect Method.

56, 75, 34, 64, 48 and 22

Solution : (a) Direct Method.

Calculation of Arithmetic Mean

Roll Numbers Marks

1 56
2 75
3 ‘34
4 64
5 48
6 22

N 6 2 X = 299

v 2 X
X -~ N
2 X -2 9 9 , N - 6

Hence X - — = 49.8 marks

(b) Indirect Method


Calculation of Arithmetic Mean

Roll Numbers Marks (X — 48)


X d

1 56 -f* 8
2 75 + 27
3 34 — 14
4 64 + 16
5 48 0

6 22 — 26

/V= 6 2 d - + 11
10

X “ A +
N
A 48 2 cf = 1 1 N = 6

48 4- = 48 + 1.8 = 49.8 marks

(B) Discrete Series i


(a) Direct Method : The formula for calculating mean is

Steps : (i) Multiply the frequency of each row with the variable
and obtain the total ^ fX .

(ii) Divide the total obtained by step (i) by the number


of observations, i.e., total frequency.

Where f — Frequency.
X = The variable in question.

y jX - Total of the products of the frequencies and


the variable X.

N — Total number of observations, i,e.( total


frequency.

(b) Short-Cut Method : The formula for calculating


mean is

X - A -f

Steps : (i) Select an assumed mean A.

(ii) Take the deviations of the variable X from the


assumed mean and denote the deviations by d

(iij) Multiply these deviations with the respective


frequency and obtain the total J W .

(iv) Divide the sum of the products ~2.fd by the total


frequency, to obtain the correction factor.
11

(V) Add the correction factor to the assumed m9an


to obtain exact mean of the data.

Where A Assumed mean

d « (X -A )

~2.fd — Sum of the products of the deviations from


the assumed mean and the frequencies.

N =- Total number of observations, i.e., total


frequency. __

Illustration 2 From the following data of the marks


obtained by 60 students of a class compute the arithmetic
mean by (a) Direct method and (b) Short-cut method.

Marks 20 30 40 50 60 85
No. of students : 8 12 20 10 6 4

Solution : (a) Direct method.

Calculation of Arithmetic Mean

Marks No. of Students


X f IX

20 r: i t• >-f' •8;* ' -•..«?*»•; 160


30 12 360
40 20 800
50 10 500
60 6 360
85 4 340
'tyth .p.:fw \v
N = 60 2 /X - 2,520

T JX _ 2 ,5 2 0
N 60
Hence the average marks - 42

"
12

(b) Short-cut method :

Calculation of Arithmetic Mean

Marks No. of Students (X - 40)


X. .; f d fd

2. 0 8 — 20 — 160
30 12 — 10 - 120
40 20 0 0
50 10 4 10 + 100
60 6 - 20 4- 120
85 4 +- 45 -f 180
s:

+
to

T fd - 120
II

Y ,fd 120
A + 40 + 40 -f 2 = 42
N 1 60

(C) Continuous Series :


(a) Direct Method : The formula for calculating mean is
:Z fm
X =
N

Steps (i) Obtain the mid-point of each class and denote k


by m.

(ii) Multiply these mid-points by the respective frequency


of each class and obtain the total ^ fm .
(iii) Divide the total obtained in step (ii) by the sum
of the frequency i.e., N to obtain the exact mean.

Where m « mid-point of various classes

..... . . Lower limit + Upper limit )

f *» The frequency of each class.


■ . . i
"Zfm - Sum of the products of the freqpencies and
the mid-points.
N = The total frequency.
13
(b) Short-cut method : The formula for calculating mean is
•Zfd
X- A+
N
Steps : (i) Select an assumed mean A.
(ii) Take the deviation of each class mid-point from an assu­
med mean and denote the deviations by d.
(iii) Multiply these deviations with the respective frequencies
and obtain the total ~2.fd.

(iv) Apply the formula : X = A +■ - ~ -

Where A — Assumed mean


d = (m -A )
~2,fd = Sum of the products of the deviations from the
assumed mean and the frequencies,
N — The total frequency.

Step Deviation Method : The short-cut method discussed


above may further be simplified by step deviation method. In this
method the only additional point is that in order to simplify calcu­
lations we take a common factor from the data and multiply the
result by the common factor. The formula is.
-Zfdx
X A + N
X C

Where A — Assumed mean


f *» frequency
dx ■■ d/c
C = Common factor
S W x ■■ Total of the products of each class frequency
with the step deviation of the respective class.
N — The total frequency.

lllustration3. Find the mean from the following frequency


distribution by (a) Direst method, and (b) Short - cut method.
Marks : 0 - 1 0 1 0 -2 0 20-30 3 0 -4 0 4 0 -5 0 5 0 -0 0
No. of students : 1 0 20 50 60 40 20
14

S olution: (a) Direct Method

Calculation of Arithemetic Mean

Marks No., of Students Mid-points


X / m fm
0-10 10 5 50
10-20 20 15 300
20-30 50 25 1,250
30-40 60 35 2,100

40-50 40 45 1,800
50-60 20 55 1 ,100

N = 200 •Zfm = 6,600

„ 'E.fm 6,600
X N 200

(b) Short-cut method

Calculation of Arithmetic Mean

Marks Mid points No. of Students (m -35)


m f d fd

0-10 5 10 — 30 — 300
10-20 15 20 — 20 — 400
20-30 25 50 —10 — 500
30-40 35 60 0 0

40-50 45 40 + 10 + 400
50-60 55 20 + 20 + 400

N - 200 S t t = — 400

A - 35

200
15

Illustration 4 ; Compute the arithmetic mean by step


deviation method.
1 i, ) " ai j> ,
Marks: 0-10 10-20 20-30 30-40 40-50 50-60 50-70
No. of students: 5 14 29 21 25 21 10
Marks: 70-80 80-90 90-100
No. of students : 7 15 3
Solution ; Calculation of Arithmetic Mean

Marks Mid-points No. of students (m -45) dx~ die


m f d c =* 10 fdx
0 -1 0 5 5 ' —40 —4 — 20
10-20 15 14 -3 0 —3 —42
2 0-30 25 29 — 20 —2 — 58
3 0-40 35 21 — 10 —1 — 21
4 0-50 45 2.5 0 0 0
5 0-60 55 21 + 10 + 1 + 21
60-70 65 10 + 20 + 2 + 20
70-80 75 7 + 30 +3 + 21
80-90 85 15 4 40 + 4 + 60
9 0-10 0 95 3 + 50 + 5 + 15
/V - 150 HJdx = —4

A - 45
2/c/x
'Z fd x ^ —4 X ~ A + — jg - X C
N = 150
C =10
—4
45 X 10
150
45—0.27
44.73
Combined Mean : If we have the arithmetic mean and
number of items of tw o or more than tw o related groups, we can
compute combined average of these groups by applying the following
formula :
V /V i* i+ ^ X a
X' 9 ~ +
10

Where
X 3a - Combined mean of the tw o groups.
X x = arithmetic mean of first group.
X a •=* arithmetic mean of second group.
Nx — number of items in the first group.
N % «-» number of items in the second group.

Note : if we have to find out the combined mean of three


groups, the above formula can be extended as follows :

NxXi + /VaXa + /V3X 3


N x + /Va + N a

Illustration 5 : The mean wage of 60 male workers in


a factory is Rs. 40 and the mean wage of 40 female workers in the
same factory is Rs. 35. Find the combined mean wage of 100
workers in the factory.

A/.Xj + WaXffi
Solution :
Nr+Na

^ - 6 0 , X i- 4 0 , A/a= 4 0 , X a= 35

6 0 x 4 0 4 -4 0 x 3 5 2,400 + 1,400
Xl* ” 00 + 40 ” 100

3800
- 38.
100

(ii) Calculation of weighted Arithmetic Mean :

One of the limitations of the S i m p 1 e arithmetic mean is that it gives


equal importance to all the items. But there are cases where the
relative importance of the different items is not the same. When
this is so, we compute weighted arithmetic mean. The term
'w eight' stands for the relative importance of the different items *
The formula for computing weighted arithmetic mean is given by :

v*>W _~~ J S
<
2 W
( 2) 17

Steps : (i) Multiply the weights by the variable x and obtain the
total £ w x .
(ii) Divide this total by the sum of the weights, i.e., 2 vv
to obtain the exact weighted arithmetic mean.
Where
Xw = Weighted arithmetic mean
x = Variable
w = Weight
2 W = Total of the product of wx
2 w = Total of weights.

Illustration 6. Calculate weighted arithmetic average from


the following data :

Designation Monthly Salary (in Rs.) Strength of the cadre

Class I officers 1,500 10


Class II officers 800 20

Subordinate staff 500 70


Clerical staff 250 100

Lower staff 100 150

Solution t
Calculation of Weighted Arithmetic Mean

Designation Monthly Salary Strength of the wx


in Rs. (x) cadre ( \m)

Class 1 officers 1,500 10 1 5,000


Class II officers 800 20 16.000
Subordinate staff 500 70 35,000
Clerical staff 250 100 25,000
Lower staff 100 150 15,000

2 w = 350 Z w x - 1,06.000

2 ;wx 1,06,000
Xw — - e 302.86
2 W 350
18

2. Median, Quartiles, Deciles and


Percentiles
Ca'culation of Median
The median, as the name suggests, is the middle value of
« series arranged in ascending or descending order of magnitude,
it is called a positional average.
(A) Series of Individual observations
Steps (i) Arrange the data in ascending order of magnitude*
then
( 11) Median will be calculated by making use of the following
formula.
/V+ 1
M = size of — 2 —th item.
Where M =* Median
N — Total number of items.
lllustiat'iQn 7. From the following data of the weekly wages
of 7 workers compute the median wage :
Wages (in Rs.): 200 300 160 180 320 400 280

Solution:
Calculation of Median

S. No. Wages arranged in


ascending order

1 160
2 180
3 200
4 280
5 300
6 320
7 400

A/+1
M s size of th item
2
19

7+ 1
4th item.

Size of 4th item = 280


Hence median wage =*» Rs. 280:

Illu s tra tio n 8. Obtain the value of median from the fo llo w in g
data :
12, 15, 40, 23, 20, 17, 89, 75.
: +■>},> + ski** l ■
S olution :
Calculation of Median

S,No. Data arranged in


ascending order

1 12
v
2 15
3 17
4 20
5 23
6 40
7 69
8 75

N+ 1
M size of —y~ th item

8 r 1
« 4.5th item

4th item + 5th item


size of 4.5th item

20+23 43
- 21.5

(B) Discrete Series


Steps (i) Arrange the series in ascending or descending order of
magnitude.
(ii) Find out the cumulative frequencies
/V+1
(iii) Apply the formula : M *» size o f — t h item
20

(iv ) Now lo o k a t th e c u m u la tiv e f r e q u e n c y 1


column and f i n d o u t t h a t t o t a l w hich i s e ith e r
e q u a l t o N + 1 th ite m o r n e x t h ig h e r th a n t h a t
2
and d e te r m in e th e v a lu e o f th e v a r ia b le
c o r r e s p o n d in g t o t h i s . That g i v e s th e v a lu e o f
m e d ia n .
Il/u s tra tih n 9 ! From the fo llo w in g data find the value of
median.
Income (Rs) 120 160 90 220 260 190
No. of persons 24 26 16 20 6 30

S olution :
■•■ •i 'tiki Calculation of Median ..

Income arranged in No. of persons c. f


ascending order /

90 16 ■ 16
120 24 40
160 26 66
190 30 96
220 20 116
260 6 122

N — 122
M “ *• size of —— ! t h item

122 4-1
■ » ---- y - - —6 1 .5th item

Size of 61.5th ite m = R s . 160.

(C.) Continuous Series :


Steps : (i) Determine the particular class in which the
N
yaiue of median lies. In this series we have to use —^ as the rank
of the median.

(ii) A fte r ascertaining the class in which median lies, the


follow ing formula is used for determining the exact value of median.

M = rL J + (m -c )
*1
21
Where
M = Median
L1 <=* Lower lihriit of the median class
L t — Upper limit of the median class
- Frequency of the median class
m - N/2
C =* Cumulative frequency of the class preceding the
median class^

Illustration 10 : Calculate the median of the following


distribution:

Length in inches No. of units

0 — 20 1

20— 40 14
40 — 60 35
60— 80 85
80 — 100 90
100— 120 45
120— 140 18
140— 160 2

(C.A.)

S olution:
Calculation of Median

Length in inches f c.f


0 — 20 1 1

20— 40 14 15
40 — 60 35 50
60— 80 85 135
80 — 100 90 225
100— 120 45 270
120— 140 18 288
140— 160 2 290

N
M = size of th item
22
290
«■* 145th item
2
Median lies in the class 80 — 100

M <=LX + — 1 (m — c)
'x
100—80
-8 0 + -^ q- - (1 4 5 -1 3 5 )
20
-.8 0 + ^ X10
— 80 + 2 .22

—< 8 2 .2 2 inches.

Illustration 1 1 : The following table gives you the distri-


bution of marks secured by some students in an examination.

Marks No. of students


0 — 20 42
21— 30 38
3 1 — 40 120
#1— 50 84
51— 60 48
61— 70 36
7 1 — 80 31

Find Median marks (C.A.


\
-
Solution :
Calculation of Median

Marks No. of students


f c. f.

0 — 20 42 42
21— 30 38 80
31— 40 120 200
41— 50 84 284
51— 60 48 332
51— 70 36 368
71— 80 31 399
23

N
Median «=* size of -^ -th item

399
= — 2 — — 199.5 th item

Median lies in the class 3 1 -4 0

But the real limit of this class is 3 0 .5 -4 0 .5

M = jL1+ (m -c)
h

= 30.5 + — ®2 q— (199.5-8C&
4 i*f)i tr0 y I
- 30.5 + X 2 0 ' X 1 1 9 -5

= 30.5 + 9.96
— 40.46
Hence Median marks = 40.46

Computation of Quartiies, Deciles and Percentiles


Quarti/es The quartiies divide the distribution in four
parts. There are three quartiies. The second quartile divides the
distribution into tw o halves and therefore is the same as the median,
The first (lower) quartile (Q J marks off the first one fourth, the
third, (upper) quartile (Q3) marks off the three-fourth.

Deciles The deciles divide the total number o f observations


into 10 equal parts, There are 2 deciles DX, D %, etc. These
are known as first decile, second decile, etc.
•••)h;i ' 1
Percentiles The percentile values divide the distribution
into 100 parts each containing 1 percent of the cases.

The procedure for computing quart'les, deciles and percentiles


is the same as the median. While computing these values in
individual and discrete series w ; add 1 to N Whereas in continuous
series we do not add 1. Thus in individual and discrete series
24

N +-1
Qj - Size of — ^— th item

3 (7V+1;
Qo = Size of th item

D 3 = Size of -A i-^ -LL th item

, 30 ( N + l )
Size o f------— —th item
100
In Continuous Series
N
<?^ = Size of -^ -th item

3N
Q j = Size of —-^ -th item

3N
aj Size of th item
10
30 N
30 =» Size of th item
100

illustration 12. From the follow ing data compute the values
of Q j, <2S, D„, and P2a.
Marks: 0-10 10-20 2 0 -3 0 3 0 -4 0 4 0-50 5 0 -6 0 6 0 -7 0 7 0-80
No. of
students: 5 25 40 70 90 40 20 10
Solution :
Calculation of Qu Qs, Ds, and Pta.

Marks f c.f

0 -1 0 5 5
1 0-20 25 30
2 0-30 40 • 70
3 0 -4 0 70 140
4 0-50 90 230
5 0 -6 0 40 270
6 0-70 20 290
70-80 10 300

N =>300
25

N
Size of—2-t h item
4
300
Size of 75th item

lies in the class 30— 4 0 .

Qx - c,

30 + — 7Q— (7 5 -7 0 )

10
= 30 X5
70
Qx - 30 + 0.71 = 30.71
3N
<?3 r Size o f—^—th item

Size of 3- i 3— «, 225 th hem


i
4
Qa lies in the class 4 0 -5 0

= 4 0 f - 5° ~ ^ ° (2 2 5 - 14° )

- 40+ - s r x 85
Qa - 40 + 9.44 = 49.44

dg - Size of ^ th item

«- Size of —-^ q*— m 180th item

d - lies in the class 40— 50


6
D. - + (\ d L- c )

- 40 + -5 °a y - (1 8 0 -1 4 0 )

- 4 0 + - § r * 40
Dt = 40 + 4.44 - 44.44
26

P ao - Size of th item

20x300
Size of 60th item
100
Pa# lies in the class
>s 2 0 -3 0

L ~ Lx (p-c)
fi
3 0 — 20
-20 + (6 0 — 30)
40

2°+4^ x 30
Pflo = 20 + 7

Thus Q i = 30.71
Q 3 = 49.44

D. = 4 4 .4 4
P 20 = 27.5

3. Mode
The Mode refers to that value in a distribution which occurs
most frequently.

Calculation of Mode
(A) Series of individual observations
in a series of individual observations, Mode is the most
frequently occuring item and hence can be located by inspection.

Illustraiion 13. The following are the size of shoe as worn


by 10 persons. Compute the modal size.
Size of ihoe.
4, 5, 4.5, 5, 5.5, 4.5, 6 , 5, 4, 5.
27

Solution. Calculation of Mode

Size of Shoe No. of times it occurs

4 2
4.5 2

5 4
5.5 1

6 1

It is clear from above that the size 5 occurs the maximum


number of times, i.e., 4. Hence Modal size of shoe is 5.
,■ | • ' t ' '
When there are tw o or more values having the same maximum
frequency, one cannot say which is the modal value and hence mode
is said to be ill-defined. Such a series is also known as bi-modal.

(B) Discrete Series /


In a discrete series it is not always possible to locate the
Mode by inspection because mode is affected by the values of the
neighbouring items. Hence we have to first of all locate the Mode
by mere inspection and then confirm it by means of "grouping and
analysis tables."

The following is the procedure for grouping


A grouping table has six columns.
(1) In column 1, we write down the frequency against the
respective items.
(2) In column 2, frequencies are grouped in tw o ’s starting
from the top.
(3) In column 3, leave the first frequency and then group
the remaining in tw o ’s.
(4) In column 4, frequencies are grouped in three's.
(5) In column 5, leave the first frequency and then group
the remaining in three's.
( 6 ) In column 6 , leave the first two frequencies and then
group the remaining in three's.

In each of these cases take the maximum total and mark it in


a circle
28

In order to find out the item which repeats largest number of


times, grouping is analysed in analysis table. White preparing this
table put column number, on the left-hand side and the various
probable values of mode on the right-hand side. The values against
which frequencies are the highest are marked in the grouping table
and then entered by means of a bar in the relevant box corresponding
to the values they represent.

Illustration 14. Following is the distribution of certain farms


selected at random from a district determine the mode :

Central size of the farm in acres : 10 20 30 40 50 60 70


No. of farms : 7 12 17 29 31 5 3
(B. Com. Osmania)>
Solution ;
Location of Mode by grouping

Central Frequency
size (1 ) (2 ) (3) (4) (5) (6 )
10 7 *1 'V
> 19 1
20 12 I 36 ^
J 'I >
> 29 j 1
30 17 ] > 58 >
40 29 > 46 1 1 J[ 1
> 77
J
50 31 ■S J■ 6 0 > 65
■; f
11
j J
60 5 39
/ 36 1 J I1>
70 3 i 8 Ji

Analysis Table

Central size of the farm in acres


Column No. 10 20 30 40 50 60 70

1
2
3
4
5
6
Total 1 3 5 4 1
29
40 size is the mode as it occurred the maximum number
of times.

<C) Continuos Series ;


The following steps must be followed for computing mode
in the case of continuous series :—
(i) By inspection or by preparing grouping table and
analysis table ascertain the modal class.
(ii) The value of mode will be determined by applying the
following formula :

Where
Mo =* Mode
L *» Lower Umit of the modal class.
A i = The difference between the frequency of the modal
class and the frequency of the pre-modal class, i.e.
preceding class (ignoring signs).
A ? = The difference between the frequency of the modal
class and the frequency of the post-nr.odal class i.e.,
succeeding class (ignoring signs).
/ = The class-interval of the modal class
Note : While calculating the value of mode it should be seen
that the class intervals of the different classes are equal otherwise
the above formula will not give correct results.
Another important point to be noted is that when mode is ill-
defined the above formula is not applicable. In that case the value
of mode is indirectly obtained by applying the following formula.
Mode •=» 3 Median— 2 Mean
This measure is called an empirical mode.

I lustration 15 : The monthly profits in rupees of 100 shops


ere distributed as follows :
Profits per shop : 0 -1 0 0 100-200 200-300 3 0 0 -4 0 0 400-500
No. of shops : 10 18 27 20 16
Profits per shop : 5 0 0 -600
No. of shops : 9
30

Compute mode from the above data

..ihitlon : By inspection mode lies in the class 2 0 0 — 300

L + — A jl -X /
Ai+A a
£ - 2 0 0 , A .* = (2 7 — 18) —9, (2 7 — 20) = 7, /= 1 0 0

M „=200 + 5 - x 100
y+ /

-200 + _ A _ x 100

- 200 + 56.25
- 256.25

Mode when class intervals are Unequal

For determining the value of mode in a frequency distribution


it is necessary that the class intervals are equal. If the class
intervals are unequal, then we must make them equal before we
start computing the value of mode. The following illustration
explains this point.

Illustration 16 : Calculate the modal value from the follow ­


ing data :

Class Frequency Class Frequency

0— 3 9 15— 18 15
3— 6 3 18— 20 25
6 — 8 5 20 — 24 13
8 — 10 5 24 — 28 25
1 0 — 12 14 23 — 30 8

12— 15 21 3 0 — 36 6

Solution : Since the class intervals are unequal, we must


adjust the frequencies and make the class intervals equal and then
compute Mode.
31

Class Frequency

0 — 6 12

6— 12 24
12— 18 36
18— 24 38
24 — 30 33
3 0 — 36 6

By inspection the mode lies in the class 18— 24.

M 0—L +
Ai X /
A i+ A a
£.=■18, A i = 3 8 — 36 = 2, A , ~ 3 8 — 33 =5, / = 6

=18 + X 6
2+ 5

—18+ - y - X 6

=18 + 1.71
= 19.71

Illustration 77 i Calculate mode from the following data.

Weight No. of students Weight No. of students

90 — 100 4 140— 150 19


100— 110 2 150— 160 10

110— 120 18 160— 170 3


120— 130 22 170— 180 2

130— 140 21 , '-y i; . '■ ;i' , :£ j ; ; :g -

Solution : By observation it <s difficult to say which is the


modal class. Hence we prepare a grouping table and analysis table.
' ■ " V■• '
32
Grouping Table

Frequency
Weight 1 2 3 4 5 6

9 0 -1 0 0
100-110
1 1 0 -1 2 0
1 2 0 -1 3 0

1 3 0 -1 4 0

140-150

1 5 0 -1 6 0
1 6 0 -1 7 0

1 7 0 -1 8 0

Analysis Table
Class in which mode is expected to lie

Column No. 110-120 120-130 130-140 140-150 150-160

Total 2 5 5 3 1

This is a bi-modal series. Hence mode has to be determined


indirectly by applying the following formula.
Mode = 3 Median— 2 Mean.
<3 ) 33

Calculation of Mean and Median

(m -A ) die
Weight f m d dx fdx c.f

9 0 — 100 4 95 -4 0 —4 — 16 4
1 0 0 - 1 1 0 2 105 — 30 -3 — 6 •
1 1 0 — 120 18 115 — 20 — 2 — 36 24
1 2 0 — 130 22 125 —10 —1 — 22 46
1 3 0 — 140 21 135 0 0 0 67
1 4 0 — 150 19 145 + 1 0 + 1 + 19 86
1 5 0 — 160 10 155 + 20 + 2 + 20 96
1 6 0 — 170 3 165 -+30 + 3 + 9 99
1 7 0 — 180 2 175 +40 +4 + 8 101

N - 101 2 fdx = — 24

„ , 2 fdx w „
A + —7— X C

A ■= 135, C — 10, c.f. — Cumulative frequency

135 + X 10
101
240
135 —
101
— 135 — 2.38
— 132.62
NV
/
Median item — ~ 2 “th item

JI01
50.5th item
2
it lies in the class 130— 140

M - J-! + 7 — (m -c )
•i
140 — 130
130 + (5 0 .5 -4 6 )
21
34

- 1 3 0 + 4 t x 4 -s
= 130 + 2.14 = 132.14
Mode = 3 Median — 2 Mean
= 3 (1 3 2.14) — 2 (1 3 2.62)
= 396.42 — 2 6 5 .2 4
= 131.18

Illustration 18.
(a) In a moderately asymmetrical distribution the value of
median is 42.8 and the value of mode is 40. Find the value of
mean.

(b) In a moderately asymmetrical distribution the values of


mode and mean are 32.1 and 35.4 respectively. Find the median
value.

(c) In a bi-modal series the value of median is 141 and the


value of mean is 140. Find the value of mode.

Solution :

Mode = 3 Median — 2 Mean


Hence given any tw o things, we can determine the third

(a) We are given Median «■ 42.8, Mode ■= 40


Substituting the values in the formula.

40 = 3 X 42.8 — 2 mean
2 mean = 1 28-4— 40
2 Mean = 88.4
Mean = 44.2
(b) We are given Mode — 32.1
Mean — 35.4
Substituting the values in the formula
Mode — 3 Median — 2 Mean
32.1 = 3 M edian— 2 x 3 5 .4
32.1 =• 3 Median — 70.8
35
—3 Median — — 70.8—32.1
,■ —3 Median = —■ . 102.9 • r . ,yj
Median — 34.3

(c) We are given Median = 141


Mean - 1 4 0
Substituting the values in the formula
Mode = 3Median — 2 Mean
= 3x141 — 2x140
= 423 — 280
Mode = 1 43
i ' . f . '1? ■
*. T i l 'i 1 ) ■' ■ i C-• i' :: -6 . if
■i i .. ■'

4. Geometric Mean
:
The geometric mean of a series containing’ N observation
is the A/th root of the product of the values.

Calculation of Geometric Mean '(G .M .)


■ ■ , - m
( A) Individual observations
The following steps must be followed :
(i) Take the logarithms of the variable X and obtasn the total
of all logs i.e., 2 L o g X.
(ii) Divide X log X by N and take the antilog of the value s®
obtained. This gives thfe value of geometric mean.

(iii) Formula :

G.M. = Antilog
N

Illustration : 19. Calculate the geometric mean of the


following series

5, 10, 125, 75, 475, 130, 45, 150, 500, 2574

■o
36

Solution : Calculation of Geometric Mean


X Log X
5 0.6990
10 1.0000
125 2.0969
75 1.8751
475 2.6767
130 2.1139
45 1.6532
150 2.1761
500 2.6990
2574 3.4106 ■
N = 10 2 log X - 20.4005

- AL 2.04005
= 109,6
Discrete Series
The following steps must be followed :
(i) Find the logarithms of the variable X
(ii) Multiply these logarithms with the respective frequencies
and obtain the total 2 f log X
(iii) Divide 2 f log X by the total frequency and take the
antilog of the value so obtained. This gives the value of
geometric Mean.
<iv) Formula :
G.M. « A n t il o g ^ - - —?—

Illustration : 20. Find out geometric mean from the following


data
x : 10 20 30 40 50 60
f: 12 15 25 20 6 2
37

Solution : Calculation of Geometric Mean

X f Log X f log X

1 C 12 1 .0 0 0 0 1 2 .0 0 0 0
20 15 1 .3 0 1 0 1 9 .5 1 5 0
30 25 1.4771 3 6 .9 2 7 5
40 20 1.6021 3 2 .0 4 2 0
50 6 1 .6 9 9 0 1 0 .1 9 4 0
60 2 1 .7 7 8 2 3 .5 5 6 4

80 f 2 Log X — 1 1 4 .2 3 4 9

S f log X
G .M . = Ant i log =
N
114.2349
= AL
80
= AL 1.4279
= 26.79

(C ) Continuous Series:

The following steps must be followed :

(i) Find out the mid-points of the classes and take their
logarithms.
(ii) Multiply these logarithms with the respective frequencies
of each class and obtain the total log m.

(iii) Divide the total log m by the total frequency and take
the antilog of the value so obtained. This gives the
value of geometric mean.
(ivj 'l'he f o r m u l a is: C. M. = A.L.HfLoijm
N
/lustration 21. Calculate Geometric mean for the following
cats.
Value: 0 -1 0 1 0 -2 0 2 0 -3 0 3 0 -4 0 4 0 -5 0
Frequency : 8 12 20 6 4
(B Com., Madras)
96

Solution : Calculation of Geometric Mean.

m.p.
Marks m f Log m f log m
0 -1 0 5 8 0.69S0 5 5320
1 0 -2 0 15 12 1.1761 14.1132
20-30 25 20 1.3979 27.9580
3 0 -4 0 35 6 1.5441 9.2646
4 0 -5 0 45 4 1.6532 6.6128
/V=» 50 "Z f log m = 63.5406

6 .M . - AL M lg
N
6 3.540 6
AL AL 1.2708 ~ 18.65.
50

S. Harmonic Mean (H.M.)


Harmonic Mean is defined as the reciprocal of the arithmetic
«»o*n of the reciprocal of the individual observations. Thus by
definition.

H^M. = N
1 1 1 , 1
Xa + + x; +
Computation of Harmonic Mean ; -
We apply the given formula depending upon the series in
computation of H.M.
(A) Individual Series
N
H .M .
2 (1 IX )
iB) Discrete Series:
N
H .M . =
2 (fIX )
(CJ Continuous Series
N
H .M .
2 fflm )
39

Illustration 22 : Calculate Harmonic Mean from the given fl


data :
125, 130, 75, 10, 45, 0 .5 ,. 0 .4 , 500, 150, 5

Solution t Calculation of Harmonic Mean

1
X
X

125 0.0080
130 0 .0 0 7 7
75 0.0 1 3 3
1 0 0.1000
45 0.0222
5 0.2000
0 .5 2.0000
0.4 2.5000
500 0.0020
150 0.0067
o
T—

2 ( 1 / x ) - 4.8599
II

N 10
H. M ~ - -------------------------------------
k > _ --------------------------------------- 2.0 6
2 ( 1 /X ) 4.8599

Illustration i 23. Calculate H.M. frpm the given data :

Size : 3 5 7 9
Frequency : 20 40 30 10

Solution : Calculate of Harmonic Mean.

Size Frequency m )
X f

3 20 6.6667
5 40 8.0 0 00
7 30 4.2857
9 10 1.1111

A /- 100 2 (f/X ) - 20.0635


40

N 100
= --------------- - = ----------------- - = 4.98
2 ( f /X ) 2 0.063 5

Hhistration 24. Find the Harmonic Mean from the following


‘k«tnbutton.

Class : 4 0 -5 0 5 0 -6 0 6 0 -7 0 7 0 -8 0 8 0 -9 0 9 0 -1 0 0
Frequency: 19 25 36 72 51 43

Solution i Calculation of Harmonic Mean

Class Frequency Midpoints flm


f m

4 0 -5 0 19 45 0 .4 2 2 2
5 0 -6 0 25 55 0.4545
6 0 -7 0 36 65 0.5 5 3 8
7 0 -8 0 72 75 0 .9 6 0 0
8 0 -9 0 51 85 0.6000
9 0 -1 0 0 43 95 0.4 5 2 6

N - 246 2 (f \m) = 3.4431

N 246
Harmonic Mean = ----- ---------- HZ — ---- ~=71.45
2 (flm ) 3.4431

Important Illustrations
Illustrations 2 5 : From the following data of calculation of
arithmetic mean, find the missing item :

House Rent : 110 112 113 117 ? 125 128 130


No. of Houses : 25 17 13 15 14 8 6 2

Mean Rent = Rs. 115.86


(B.Com., Osmania)
41

S o lu tio n i Let the Missing Figure be denoted by Z


Calculation of Missing value.
House Rent No. ot Houses
X f fx

110 25 2,750
112 17 1,904
113 13 1,469
117 15 1,755
Z 14 14Z
125 8 1,000
128 6 768
130 2 260
N = 100 2 f x = 9906 I-1 4Z

X=
N
9906 + 14Z
1 1 5 -86 - — Too—
11586 = 9 9 0 6 + 14Z
— 14Z = — 11586 + 9906
— 14Z = — 1680
1680
120
14
Hence missing Value is 120.
Illu s t r a t io n 26. A market with 187 operating firms has
the following distribution of avarage numbar of workers in various
income groups •
Income - groups : 150-300 300-500 500 -8 0 0 8 0 0 -1 2 0 0
No. of firms : 40 32 39 34
Average No. of
workers : 8 12 5 7
Income-groups 1,200-1,800
No. of firms 42
Average No. of
workers
Find the average salary paid in the whole market.
42

S o lu tio n : In this problem frequencies are not given. The


i ii quencies can be obtained by multiplying the number of firm s w ith
1iu> average number of workers.
Calculation of Average Salary paid
No. of Average Total No. m .p fm
Income firms No. of of (4 X 5 )
group workers workers
(2 X 3 )
(1) (2) (3) (4 ) f (5 )/7 l i (6)
150— 300 40 8 320 225 72,000
300— 500 32 12 384 400 1,53,600
500— 800 39 5 195 650 1,26,750
800— 1,200 34 7 238 1,000 2,38,000
1,200— 1.800 42 4 168 1,500 2,52,000
N = 1 ,3 0 5 2 f m = 8,42,350
8,42,350
= 6 4 5 .4 8
1,305
Illu s t r a t io n 27. Find mean for the following data :
Class interval : 5 0 -5 9 4 0 -4 9 3 0 -3 9 2 0 -2 9 1 0-19 0 -9
Frequency ‘ 1 3 8 10 15 3

S o lu tio n : For Calculating arithmetic mean it is not necessary


to change inclusive series into exclusive series by adjusting the
lim its of the class-intervals because the mid-points remain the same
whether or net the adjustment is made. However, in case of median
and mode adjustment is necessary. Also there is no need to put the
series in reverse order, i.e., the lowest class first, the next higher
class, and so on.
Calculation of Arithmetic Mean

Class-interval M id-points Frequency (m -34.5) d x = d lc


m f d c=19 fd x

5 0 - -59 54.5 1 + 20 +2 + 2
4 0 - -49 44.5 3 + 10 + 1 +3
3 0 - -39 34.5 8 0 0 0
2 0 - -29 24.5 10 — 10 —1 -1 0
1 0 --19 14.5 15 — 20 — 2 —30
0-- 9 4.5 3 —30 —3 — 9
N = 40 2 /c fx = — 44
43

A = 34.5
SfoTx = — 44 X = A + C
/V = 40
C = 10 = 34.5 + ^ ^ X 1 0 = 3 4 .5 — 11 = 2 3 .5

Ill u s t r a t io n 2 8 . From the follow ing data of monthly wages


of 40 workers employed in a certain factory, construct frequency
table w ith class intervals of Rs 10.00 each, and using this table
calculate the average wage paid to a worker

Wages in Rupees
100, 90, 99, 70, 76, 75, 89, 66, 109, 93, 97, 92 95, 67,
108, 86, 112, 68, 81, 77, 88, 105, 80, 85, 61, 72, 69, 118,
82, 78, 69, 71, 60, 63, 73, 65, 79, 64, 78, 84.
(B.Com.)
S o lu tio n :
Formation of frequency distribution and calculation of mean

Wages Tally Bars No. of workers mid-points (m -85)


f m d fd

60— 70 mj mi 10 65 —20 — 200


70— 80 m i mj 10 75 — 1 0 — 100
80— 90 m i 111 8 85 0 0
90— 100 m i i 6 95 + 1 0 + 60
100— 110 m i 4 105 + 2 0 + 80
110— 120 11 2 115 +30+ 60
o

£ / c / = — 100
I!

4=85
Zfd
X=A +
N
( - 100 )
=85+
40
= 8 5 — 2.5
= 8 2 .5
44

Illustration 29. The mean of a group of 75 observations


ik 17. It was later discovered, that one observation was wrongly
i nod as 43 Instead of the correct value 53. Find the correct mean.
'solution :
We are given N = 75. X = 27
2X
Since X = N
2 X = /V X = 7 5 X 2 7 = 2 ,0 2 5
But this is not correct 5.X.
Correct 2 X = incorrect 2 X— wrong item -f correct Item
= 2 ,0 2 5 — 4 3 + 5 3
= 2,035
Correct 2 X 2,035
Correct X = -------- /y------ ** = ~ 7 5 = 2 7 .1 3
Hence the correct average = 27.13

Illustration 30. The mean wage of 100 workers in a factory


running two shifts of 60 and 40 workers respectively is Rs. 38.
The mean wage of 40 workers working in the evening shift is Rs. 3 5 .
Find the mean wage of 60 workers working in the morning shift.
Solution : We are given
X i a = 3 8 . N x = 60, /Va= 40, X - = 35
Xx = ?
v _ N xX x+ N t X 2
13 ~ N i+ N ,
o0 _ 6 OXX1 + 40X35
~ 60+40
•ko 60Xx + 1400

3,800 = 6 0 X i + 1,400
—60 X i = — 3 ,8 0 0 + 1 ,4 0 0
—60 Xx = — 2,400
_ 2,400
Al _ 60
X a = 40
45
Illustration : 3 1 . The mean yearly salary of employees of
company was Rs. 2 0,000 . The mean yearly salaries of male anc
female employees were Rs. 20,800 and Rs. 16,800 respectively
Find out the percentage of males and females employed by the
company. (B. Com

Solution : Let A/x represent percentage of males and N 2 percentage


of females, so that /Vx + /Va= 100. We are given X i a ~ 20,000
X i - 20,800, X a~ 16,800

Substituting the values in the formula.


/VxX, + / v , x 2
X i. ■
n x+ n2
N i (20,800) 4- /V, (16,800)
20,000 -
100
2 0 .0 0
. 0 0 0 = 20,800 N x+ [ ( 1 0 0 — /Vx) (1 6 ,8 0 0 )]
20.00. 000 - 20,800 A/x + 16 ,80;000— 16,800 /Vx
4,000 N x — 3 ,20,00 0
3 ,2 0 ,0 0 0
Nx 80
4000
/Va - 100—/Vx
Z= 100— 80
* = 20
•, • «j"i4'\ i: ■ . s
Thus the percentage of males and females employed is 80
and 20 respectively.

Illu stration i 3 2 . Calculate median from the following data .

Value Frequency

Less than 10 4
20 16
30 40
40 76
50 96
60 112
70 120
46
■olution ! • i- K- * v t.J.I „ !• f > i; "l

Since cumulative frequencies are given, first we have to find


the simple frequencies and then compute median.

Calculation of Median

Value Frequency c f.

0 — 10 4 4
10— 20 12 16
20— 30 24 40
30— 40 36 76
40— 50 20 96
50— 60 16 112

60— 70 8 120

70— 80 5 125

N - 125

Median = Size of = - th item


2
1 25
= J j f — = 62.5th item
2
Median lies in the class 3 0 -4 0

M = L 1 - f — ~ - J- (m -c)
'1

= 3° + 4° ^ - 3-°- (62.5 — 40)

= 30 + X 22 5

= 30 - f 6.25
= 36.25
1 'r: ■■ ; ,. cfc '
Illustration : 33. Find the missing frequency in the following
distribution if N is 1 70 and median is 35.
Variable : 0 - 1 0 1 0 -2 0 2 0 -3 0 3 0 -4 0 4 0 -5 0 5 0 -6 0 6 0 -7 0
Frequency: 10 20 ? 40 ? 25 15
47

Solution Let the missing frequency of the class 2 0 -3 0 be denoted


by f 1 and that of 4 0 -5 0 by +

Calculation of Missing Frequencies

Variable f c.f

0 -1 0 10 10

1 0 -2 0 20 30
2 0 -3 0 + 30+ +
3 0 -4 0 40 70++
4 0 -5 0 + 7 0 + / , + ,+
5 0 -6 0 25 95 + + + +
6 0 -7 0 15 110 + ++ +

N
M = Size of -^ -th item

170
— ■
— 2 — — 35th item

Since Median is 3 5 r it lies in the class 30 -4 0 .


M = Li + % 1 (m c)
4 0 _3 0
35 = 30 + ■ ^ Q.- [8 5 -(3 0 + + )]

35 = 30 + -1 2 .(5 5 -^ )

35 - 30 = —

20 = 55— f t
f T = 55— 20
f, =35

The total frequency = 1 70

The frequencies of the classes other than the missing ones are
(10 + 2 0 + 4 0 + 2 5 + 1 5 ) = 1 1 0 .
48

1 1 0 4 -^ + ^ = 1 7 0
Hence A ,+ fa= 1 7 0 — 1 1 0 = 6 0
Since 71+ / 1= 6 0
/ „ = 6 0 — 35
/ a= 25
Thus missing frequencies are ^ = 3 5 , f %— 25.

Illustration : 34. Find the median of the following :


Mid-points ; 5 15 25 35 45
Frequency 3 9 8 5 3
(B. Com. Bangalore)

Solution : Here we are given the mid-values of the class-


intervals of a continuous series. The difference between two mid­
values is 10, hence 1 0 /2 = 5 is deducted from each mid-value to
find the lower limit and the same is added to find the upper limit of
a class. These classes are thus 0 — 1 0 , 1 0 — 20, 20— 30, 30— 40,
and 40— 50.

Calculation of Median

Class-Intervals frequency c.f.


f
0— 10 3 3
10— 20 9 12
20— 30 8 20
30 — 40 5 25
40— 50 3 28

Median = Size o f - y t h item

28
= 14th item

Median lies in the class 20— 30.


L *-L t
M = L, (m -c )
f,
= 20+— 8 — (1 4 -1 2 )
49

= 20 + ~ X 2 = 2 0 + 2 .5

M = 2 2 .5
Illustration : 3 5 Calculate the Mean, Median and Mode from
the following data of marks obtained by 10 students :

S.No. Marks obtained S.No. Marks obtained


1 43 6 60
2 48 7 37
3 65 8 48
4 57 9 78
5 31 10 59

Solution .
Calculation of Mean, Median and Mode
S.No. Marks obtained (Marks arranged in ascending order)
, ii. i in .
1 31
2 37
3 43
4 48
5 48 "in^ 1

6 57
7 59
8 60
9 65
10 78 v?V \
3S

<r—
0

2X=526
II

A/4-1
(ii) Median=Size of - 2 th ltem

= 5.5th item
5th ite m + 6 th item
Size of 5.5 th item = ^ -----------
4 8 + 5 7 _ 2.05 _
52 5
60

(111) Mode = Since the item 48 occurs the maximum number of


times i.e., 2, hence the modal marks are 48.

Illustration: 3 6 Calculate Mean, Median and Mode for the


following frequency distribution,
Marks: S 10 15 20 25 30 35 40 45 50
No. of students : 20 43 75 67 72 45 39 9 8 6
(B.Com. Madras)
Solution : Calculation of Mean, Median and Mode
X

Marks No. of students die


X f dx fdx c.f.
5 20 — 20 „4 — 80 20
10 43 — 15 —3 — 129 63
15 75 — 10 —2 — 150 138
20 67 — 5 —1 — 67 205
25 72 0 0 0 277
30 45 + 5 + 1 + 45 322
35 39 + 10 + 2 + 78 361
40 9 + 15 4-3 + 27 370
45 8 + 20 +4 + 32 378
50 6 + 25 + 5 + 30 384
W =384 "2-fdx.= — 214

i4 = 25
C = 5
Calculation of Mean :~
2/cf'x
X=A X C
N
-21 4
25+ X 5
384
= 25— 2.79
= 22.21
Calculation of Median
A /+ 1
Medians Size of —J-—th item

334+1
192.5
2
51

Size of 192.5th item is 20.


Hence median marks = 20.

Calculation of Mode
Note: Pr epare Grouping and a n a l y s i s Table
Marks 20 i s th e Mode as i t o cc ur ed th e Maximum numbe
o f times.
Illustration 37 Calculate Mean, Median and Mode from the
following data :
Age : 2 0 -2 5 25 -3 0 30-35 35-40 40-45
No. of persons 40 60 100 200 130
Age : 45-50 50-55 55-60
No. of persons : 170 60 40

Solution : Calculation of Mean, Median and Mode

Mark 5 mp (m -A ) d jc
m f d dx fdx C-f

2 0 — 25 22.5 40 — 15 —3 — 120 40
2 5 — 30 27.5 60 — 10 —2 — 1 20 100
3 0 —35 32.5 100 — 5 —1 — 100 200
3 5 —40 37.5 200 0 0 0 400
4 0 — 45 42.5 130 + 5 + 1 + 130 530
45 —50 47.5 170 + 10 + 2 + 340 700
50 —55 52.5 60 + 15 + 3 + 180 760
55— 60 57.5 40 + 20 + 3 + 160 800

N = 800 2 fdx = + 4 7 0
4 = 3 7 .5
C-5
(i) Mean
X = A +
470
37 5 + ~800” X5

= 3 7 .5 + 2 .9 4
= 40.44
52

(It) Median
N
Median = Size of -y-th item

— ? ^ - = 400th item

Median lies in the class 3 5 — 40

Median = Ll + — — (m — c)
'1
o N
*? o
in

o
o

o
o
cs
=35+
1
o

=35+ 5 X 200
200 A
=35 + 5
= 40

Mode

By Inspection mode lies in the class 35— 40

M ode= L + Ai X i
A i+ A *
= 35+ 100 ~ 5
100+70 A 3
JOO
= 35+
170
= 3 5 + 2 .9 4
= 3 7 .9 4
LIST OF FORMULAE
Individual Series Discrete Series Continuous Series

1. Arithmetic Mean : Direct Method Direct Method

TT _ S fX . . S fm
Direct Method ;
X ~ "TV- X N
Short cut Method
A N Short cut Method
* - a + h t
Short cut Method :
Step deviation Method

X - * + c

2. Median : Size of - y t h item


/V4-1
/V+ 1 M = Size of—— th 'tem
M = Size of —2 ~ th item
M - Lx (m -c )

3. Mode : M„Br= By inspection or-by


grouping determine that value
M , = Most CommoruValue. M- = t + A f + A , X i
around which most of the
frequencies are concentrated.

N ot e : When t h e Mode i s i l l - d e f i n e d , t h e f o l l o w i n g form ula w i l l


be a p p l i e d i n c a l c u l a t i o n o f Mode: Mode=3 Median-2 Mean.

I n d i v i d u a l S e r ie s D is c r e t e S e r ie s I C o n tin u o u s S e r ie s

4. Geometric M ean :
s .m .= al a.M, . * ( * £ # £ )

AL — Antitog

5. Harmonic M e a n :

HM-= S m H'M ' = d w


H'M ' = W t F )
8

6. Combined A rithm etic M ean :


Xia

7, W eighted A rithm etic M ean t


•5

PROBLEMS

1. The monthly income of ten families (in rupees) in a ce


locality are given below :

Family : A B C D E F G H 1 J
wH# Income : 30 70 10 75 500 8 42 250 40 36
Calculate the arithmetic average by (a) Direct method anj
(b) Short-cut method.
. (B.Com. Kashmir)
(Ans : X = 1 0 6 .1 >
•t

2 The following table gives the monthly income of twelve


families in a district, calculate the arithmetic mean.

H.No. Monthly income (in Rs.) H.No. Monthly income


(in Rs.)
f '■
1 260 7 70
2 200 8 94
3 90 9 105
■IJ
4 104 10 70
5 109 !.. P 11 550
6 70 12 250

. (Ans : X = 164 33)

3. The following are the prices of shares of a compsn frr m


Monday to Saturday :

Days Price (Rs) Days Fries (Rs .)


Monday 200 Thursday 160
Tuesday 210 Friday 220
i'|
Wednesday 208 Saturday 250
Compute the average value by short-cut method

(B.A Madras)
(Ans : X = 208)
56

4. Calculate the mean of the following data


Size Frequency
11 1
12 2
13 4
14 7
15 10
16 11
17 8
18 4
19 2
20 1
(fi4 : fc = 1 5 .5 4 )

From the following data find the missing frequency when


msari is 15.38.
Marks : 10 12 14 16 18 20
Frequency : 3 7 12 '20 ? 5
(Ans : Missing freq u en cy= 8)
6. The mean of the following series is 30.5 marks. Find the
missing figure.
Marks : 10 20 30 ? 50
Frequency : 8 10 20 15 7
(Ans : Missing figure = 40)
7. Calculate the arithmetic average from the given data :
Miles travelled : 2 4 6 8 10 12 14 16 18
No. of Villagers : 38 104 140 78 48 42 28 24 16
Miles travelled : 20
No. of Villagers : 2
(B.Com. Kakatiya)
(Ans : X = 7 .7 8 )
8. Calculate arithmetic average of the following data :
Marks No. of students Marks No. of students
0— 10 33 50— 60 322
10— 20 53 6 0 — 70 439
20—30 108 70 — 80 526
30—40 221 80 — 90 495
4 0 —50 153 9 0 — 100 50
(B. Com. Osmania)
(Ans. X = 6 2 .3 3 )
y distribution, calculate the

Monthly wag as No. of


(in Rs.) workers

37.5— 42.5 4
42.5— 47.5 6
47.5— 52.5 1
52.5— 57.5 1

(I C W .A )
(Ans: X = 28.25)

10. The marks scored by 60 students in an examination in


statistics are given below. Form a frequency distribution with class
intervals of ten and calculate the arithmatic mean.

6 10 58 56 0 25 32 35 35 9
78 17 60 50 35 38 30 10 48 5
68 48 35 30 31 21 23 23 50 72
19 25 35 40 46 42 45 25 60 41
35 36 38 35 33 46 28 31 35 42
46 38 39 45 48 50 28 29 31 55

(B.Com., Mysore)
(Ans : X = 3 7 )

11. Calculate arithmetic mean from the following data :

Value Frequency Value Frequency

0— 4 328 30— 39 598


5— 9 350 40— 49 524
10— 19 720 5 0 — 59 378
20 — 29 664 60— 69 244

(B. Com, Kerala)


(Ans: X = 2 8 .9 1 )

Hint : Combine the first and second classes and then com­
pute arithmetic mean.
®i

}2 Calculate the aw«r«au» m m k* Of the students from ths


•"Mowing data :
Marks No. of students
Below 10 15
11 20 35
$i 30 00
At 40 84
tt 50 98
it 60 127
70 198
ii 80 250
(B.Com., Bangalore)
(Ans: X = 5 0 .4 )

H a t -.- This is a cumulative frequency distribution. So first


convert it to a simple frequency distribution and then compute
Mean.

13. Calculate the average weight from the following data :


Weight (in lbs) : 100 110 120 130 140 150
No of persons : 100 130 70 20 30 50

(Ans: X — 7T ? *5'

14. The following table gives the number of persons with


different incomes in the U.S.A. during the year 1929.
Income in thousands No. of persons
of dollars in lakhs.
Under 1 13
1— 2 90
2— 3 81
3— 5 117
5— 10 66
10— 25 27
25- 50 6
50— 100 2
to o — 1000 2
Calculate the average income per head.
(B Com. Lucknow)
(Ans: X = 8 .0 6 thousand dollars
69

H i n t For computing mean, class intervale need not bt


uniform. As the classes in this problem are irregular step deviations
are not possible.

15. Find the average marks of a student from the following


table.
Marks No. of students Marks No. of students
Below 80 240 Below 40 75
70 190 ., 30 60
,/ 60 125 „ 20 40
50 95 „ 10 25
(B.Com. Banaras)
(Ars : X = 49.58)
H int This is a cumulative frequency distribution. So first
convert it to a simple frequency distribution and then compute Mean.
Need not put the series in reverse order.

16. From the following data compute the mean marks of all
the students of 50 schools in a city.
Marks obtained No. of schools Average No. of
students in a school
More than 35 7 200
3 0 — 35 10 250
2 5 —30 15 300
20 — 25 9 200
15— 20 5 150
Less than 15 4 100
(B.Com. Rajasthan)
(Ans : X = 27.85)

Hint In this problem frequencies are not given. The


frequencies can be obtained by multiplying the no. of schools with
the average no. of students.
1 7. Find the missing frequency from the following data :
Marks : 0 -5 5 -1 0 1 0 -1 5 1 5 -2 0 2 0 -2 5 2 5 -3 0 3 0 -3 5
Frequency ' 1 0 12 16 ? 14 10 8

The average mark is 16.82


(B.Com. Gujarat)
(Ans: Missing frequency^! 8)
60

18. Calculate the mean for the data A


• U»n interval : 0 -4 5 -9 1 0 -1 4 1 5 -1 9 2 0 —2 4 2 5 -2 9
i M><|tinncy 20 40 52 64 37 8
(B.A., Kerala)
(Ans: X = 1 3 .8 6 )

19. There are three sections in B.A. final year in a certain


•llinjo The nbmber of students in each section and the average
■Mmva obtained by them in the paper of Economics in the annual
mmination »re as follows :

• " tion Average marks in economics Me. of students


A 55 50
II 60 60
C 75 50
Find the combined me a~..
( Ans : X ia3 = 63.13).

2 0 . The mean wage of 100 workers working in a factory


■mining two shifts of 70 ana 30 workers respectively is Rs. 8 4 .
i lie mean wage of 30 workers working in the evening shift is Rs.
/0 Find the mean wage of 70 workers working in the morning
•hift.
Arts : Mean wage of 70 workers or)
X 1= R s .9 0 )

21. The mean wages paid to 1.000 employees of an organi­


sation was found to be Rs 180.4. Later on it was discovered that
Uio wages of tw o employees were wrongly taken as 397 and 265
instead of 297 and 2 8 5 . Find the correct mean.
(Correct mean = 180.32 )

22. The mean age of a combined group of men and women


i 30 years. If the mean age of the group of men is 32 and that of
the group of women is 27, find out the percentage of men and
woman in the group.
(C.A.).
(B.Com., Osmania, May 1984)
(Ans : Percentage of man in the group is 60 and the
percentage of women in the group is 40).
61

23. The mean of 50 items is 100. However tw o of the fi


items have been w rongly taken as 102 and 89 instead o f -1 £2 ani
7 8 . Find the correct mean.

(B.Com ., Osmania, Nov. 1985

(Arts : Correct M ean=100.18)

2 4. The mean height of 25 male workers in a factory is 61


cm s., and the mean height of 35 female workers in the same factory
is 58 cms. Find the combined mean height of 60 workers inthe
factory.

(M .A ., Economics, Punjab) il

{A n s : X , 9 = 5 9 .2 5 cm s.)!

25. The mean annual salaries paid to all employees of a


company was Rs. 500 . The mean annual salaries paid to male and
female employees were Rs. 520 and Rs. 420. Determine the percerw
tage of males and females employed by the company. *

(M .B.A., Delhi)

(A rts ' The percentage of males and females employed is


80 and 2 0 ).

2 6 . The mean weight of a group of 25 adults was calcu.


lated to be 78.4 kg. It was later discovered that one weight was
misread as 69 kg., instead of the correct value 96 kg. Calculate
the correct mean.
(B.Com., Utkal)
(A n s : Correct M ean=79.48)

27. The mean weight of 150 students in a certain class is


60 kgs. The mean weight of the boys in the class is 70 kgs., and
that of the girls is 55 kgs. Find the number of girls in the class.

{Ams: Mo. of Girls = ioo) B.Com ., Osmania, May 1985)


63

28. The mean of 20 observations is 5.1. 8/ mistake one


observation is taken + 7 instead of— 7. Find the correct mean.
(B .C o m . Gujarat)
(Ans : Correct M ean = 4.4)

29. In a class of 50 students, 10 have failed and their


nverace of marks is 2.5. The total marks secured by the entire class
wore 281. Find the average marks of those who have passed.
{B.Com., Bangalore)
(Ans : Average marks of those who have passed=6.4)

30. Calculate the simple and weighted average from the


following and account for the difference between the two :
Price per tonne (Rs.) : 45.60 50.70 42.45
Tonnes purchased 135 40 25
(C.A., Inter)
(Ans: X —46-25, X w —46.23)

31. Calculate the weighted arithmetic mean price per tonne


of coal purchased by an industry for the half year.
Month Price per tonne (Rs.) Tonnes purchased
July 42.50 25
August 51.25 30
September 50.00 40
October 52 00 50
November 44.25 10
December 54.00 45
(Ans : X W = 5 0 .3 6 )

32. Find the combined mean from the following data :


Group 1 2 3
Number 200 250 300
Mean 25 10 15
(B.Com. Sardar paUl University)
(Ans: X - , 3 - 1' 5 )

33. Find the combined mean of the given data :


X!=4210 N \ - 50
X 150 N -1 0 0
(Ans: X ia = 170)
S3

3 4 . ' Determine RWdian ffiQBi *be following data :


35, 30, 15, 45, 18, 9, 11. 3 8 , 13
(Ans : M = 18)
35. The following table gives the monthly incomas of 12
families in a town :
S. No. : 1 2 3 4 5 6 7 8 9 10
Monthly income: 280 180 '96 98 104 75 80 94 100 75
S. No. : 11 12
Monthly income : 600 200
Calculate the median of the above.incomes
(M.A. Lucknow)
(Ans : M = 9 9 )
36. Calculation median from the given data :
22, 28, 16, 30, 18, 10, 35, 45, 12, 32
(Ans : M = 2 5 )
3I f Determine the Median from the following items
25, 15. 23*. 40, 2-7, 25, 23, 25, 20
(M.Com. Agra)
(Ans : M = 2 5 )
38. Compute the Median of the following distribution.
Height of men in inches No of persons
60 27
61 146
62 435
63 398
64 210
65 128
66 98
(Ans : M =63)
39. Calculate median from the given data :
Marks ; 20 30 40 50 60 70
No, of students: 8 12 20 10 6 ^4
( An s : M =40)
40. From the following frequency distribution, find out
value of Median
Size : 0 -1 0 1 0 -2 0 2 0 -3 0 3 0 -4 0 4 0 -5 0
Frequency : 3 5 8 4 2
(B. Com. Bangalore)
(Ans : M = 23.75)
64

41. Calculate median from the following data :


Value Frequency Value Frequency
0 -4 328 30 — 39 598
6— 9 350 40 — 49 524
10— 19 720 5 0 -5 9 378
20—29 664 60 — 69 244
(B.Com. Andhra)
(Ans: M = 2 7 .1 1 )

42. The following table shows the age distribution of


persons in a particular region :
Age No. of persons Age No. of persons
(Years) (in thousands) (Years) (in thousands)
Below 10 2 Below 50 14
„ 20 5 .. 60 15
„ 30 9 70 15.5
„ 40 12 70 and over 15.6
Find the median age.
i
(Ans : M =27)
43. Amend the following table and locate the median from
the amended data :
Size Frequency Size Frequency
10 — 15 10 3 0 — 35 28
16 — 17.5 15 35 — 40 30
17.5— 20 17 40— 50 40
20 — 30 25
(B.Com. Nagpur)
(A n s: M = 3 2 .6 7 )

Hint: The data can be rearranged as follows :


10— 20 20— 30 30 — 40 40 — 50
42 25 58 40

44. Find the median of the following data :


Wages (in Rs) 6 0 -7 0 5 0 - 60 4 0 -5 0 3 0 -4 0 2 0 -3 0
No. of labourers : 7 21 11 6 5
(B.Com. Calicut)
(Ans : M = 5 1 .4 2 )

Hint : Rearrange the classes in the ascending order and then


compute median.
<5) 65

4 5 . Find the missing frequency in the following distribution


if N is 100 and median is 30.
Marks No, of students Marks No. of students
0 -1 0 10 3 0 -4 0 30
1 0 -2 0 ? 4 0 -5 0 ?
2 0 -3 0 25 5 0 -6 0 10
(M.Com.)
(Ans : / a— 15 and / a= 1 0 )

46. Compute median from the following data :


Mid-value : 115 125 135 145 155 165 175 185
Frequency : 6 25 48 72 116 60 38 22
Mid-value : 195
Frequency : 3
(B.Com. Rajasthan)
(Arts: M = 1 53.79
47. You are given the following incomplete frequency
distribution. It is known that the total frequency is 1,000 and that
the median is 4 13.11 . Find the missing frequencies and calculate
the value of the mode.
Value : 3 00-325 3 2 5 -3 5 0 350-375 3 7 5 -400
f : 5 17 80 ?
Value : 4 00-425 425-450 450-475 475-500
f : 326 ? 88 99
( Ans: f 1= 2 2 7 , f 2 =158) (B.Com. Osmania Nov, 1985)

48. Find the median from the data given below :


Size Frequency
Below 10 5
n 20 20
ft 30 50
*t 40 105
ft 50 190
it 60 250
t* 70 295
tt 80 320
(B.Cbm. Madirtai)
(A n s : M = 4 6 .4 7 )
66

49. Calculate the value of Median from the following data


w.ujiis (in Rs.) No. of workmen Wages (in Rs) No. of workmen
20-21 8 2 5 -2 6 25
2 1 -2 2 10 2 6 -2 7 16
2 2 -2 3 11 2 7 -2 8 9
2 3 -2 4 16 2 8 -2 9 6
2 4 -2 5 10
(B.Com. Osmania)
(Ans: M = 2 5, 0 2 )

50. From the following table given below, find the mean and
in median :
Marks No. of students Marks No. of students
1— 5 7 21— 25 24
6— 10 10 26 —30 18
11 — 15 60 31 — 35 10
16— 20 32 3 6 — 40 5
41— 45 1
(B.Com. Osmania)
i.'fy'v! ' ''• ••»Mf'- (Ans : X = 1 8.42 M = 16 .5 2 )

51. Compute Median and Quartiles from the given data :


Marks : 0 -1 0 1 0 -2 0 2 0 -3 0 3 0 -4 0 4 0 -5 0 5 0 -6 0
Frequency : 3 9 15 30 18 5
(B.Com.)
(Ars : M = 3 4 .3 3
Q l = 2 5 .3 3
Q3= 4 1 .67)
. . . (' , ‘J:.\ i'J ‘; ;;
52. Compute 7th Decile and 85th percentile from the given
data :
Deposit Balance 0 -1 0 0 1 0 0 -2 5 0 2 5 0 -4 0 0 4 0 0 -5 0 0
No. of deposits 25 100 175 74
Deposit Balance 5 0 0 -5 5 0 5 5 0 -6 0 0 6 0 0 -8 0 0 8 0 0 -9 0 0
No. of deposits 66 35 5 18
Deposit Balance 9 0 0 -1 0 0 0
No. of deposits 2
(A n s : D ,= 4 6 7 .5 7
P „ = 5 3 8 .6 4 )
67

53. Calculate mode from the following data :


Roll No. : 1 2 3 4 5 6 7 8 9 10
Marks : 43 48 65 56 31 60 37 48 62 78
(B.Com. Madurai)
(Ans : Mu—48)

54. Calculate mode for th e following frequency distribution:


Marks : 5 10 15 20 25 30 35 40 45 50
No. of students: 20 43 75 6/ 72 45 39 9 8 6
(Ans : M0= 20)

55. Find out mode the following data :


Profis per shop No.of shops
(in'000 rupees)
0 — 100 15
1 0 0 —200 20
200— 300 28
3 0 0 — 400 32
4 0 0 — 500 18
500— 600 12
6 0 0 — 700 5
(BCom.Osmania)
fims :M „ = 3 2 2 .2 2 )

56. From the following information, Calculate mode:


Lower limit of the modal class ... 2 00
Upper limit of the modal class ... 2 20
Frequency of the modal class ... 40
Frequency of the class succeedingthe modal class ... 30
Frequency of the class preceding the modal cless ... 28
(Ans:M =210.91)

57. Calulate mode for the following data :


Size : 0— 5 5-10 10-15 15-20 20-25 25-30 3 0 -3 5
Frequency : 51 32 49 56 51 23 17
Size 35-40
Frequency : 7
(Ans: Mq=17.92.
68

58. Calculate the mode from the following data :

Variate value Frequency Variate value Frequency


0— 2 3 18— 24 10
2— 6 6 2 4 — 30 8
6— 12 17 3 0 — 34 5
12— 17 17 3 4 — 36 2
17— 18 10
(B.Com, Osmania)
(Ans 4.22)

Hint : First make the class-intervals uniform and then


xjmpute Mode.

59. Find out the mode in the following series :


Size Frequencies
0— 5 1
5— 10 2
10— 15 TO
15— 20 4
20— 25 TO
25— 30 9
30— 35 2 (B.Com, Osmania)
(Ans. M0= 2 4 .2 9 )

60. Calculate the modal value from the following frequ-


«icy distribution :

Marks obtained No. of candidates


0— 9 6
10— 19 29
20— 29 87
30— 39 181
40— 49 247
50— 59 263
60— 69 133
70— 79 43
80— 89 9
90— 99 2
(B.Com. Calcutta)
(Ans. M 0= 4 7 .5 4 >
69

H int By inspection it is difficult to say which is tkt


modal class. So prepare a .grouping table and analysis table to
find out the modal class.

6 1 . From the following data of the weight of 122 person*


determine the modal weight :
Weight No. of persons
100— 110 4
110— 120 6
1 2 0 — 130 20 m
130— 140 32
140— 150 33
150— 160 17
1 6 0 — 170 8
1 7 0 — 180 2 tii ip Pit i}•'< . PP . ' 1
(Ans : X = 139.51
M = 339 59
M 0 = 1 4 0 .0 5 )
H int : This is a bi-modal series. Hence mode has to b *
determined indirectly.

62. From the following data of the weight of 100 student


of a class calculate the modal weight.
Weight No. of students Weight No. of studentt
90 — 100 3 140— 150 19
100— 110 2 150— 160 10
110— 120 18 160— 170 3
120— 130 22 170— 180 2
730— 140 21 (A ns: X=133
M =132.3 8
M = 1 3 1 .1 4 }
(J
H in t: This is a bi-modal series. Hence mode has to E«
determined indirectly.

63. Calculate modal value from the following data.


Marks : 6 0 -7 0 5 0 -6 0 4 0 -5 0 3 0 -4 0 2 0 -3 0 10-1®
Frequency : 14 14 32 20 20 IS
Marks : 0 -1 0
Frequency : 5
(1 ^ = 4 *
70

Hint Rearrange the data in ascending order and then com-


,..ito the mode.
64. Calculate mode from the following data :
Marks No. of students Marks No. of students
Above 0 80 Above 60 28
, 10 77 70 16
„ 20 72 „ 80 10
„ 30 65 90 8
40 55 „ 100 0
„ 50 43 (B.Com)
(Ans : M 0= 5 5 )
Hint This is a cumulative frequency distribution, we must
tirst convert it into a simple frequency distribution and then compute
Mode.
65. Calculate mean, median and mode from the following

S.No. : 1 2 3 4 5 6 7 8 9
Marks : 54 71 57 52 49 45 72 57 47
(Ans : X = 5 6 ; M = 5 4 ; M u= 5 7 )
66. Calc ulate mean, median and mode from the following
frequency distribution of marks at a test in
statistics : ■,sr
Marks 5 10 15 20 25 30 35 40 45 50
No. of students : 20 43 75 76 72 45 39 9 8 6
(B.Com. Punjab)
(Ans : X = 2 2 .1 6
M =20
M 0= 20)
67. Calculate mean, median and mode for the following
frequency :
Monthly income No. of lecturers
30 0 — 325 5
3 25— 350 17
3 50— 375 80
375— 400 227
4 0 0 — 425 326
4 2 5 — 450 248
4 50— 475 88
475— 500 9
(B.Com. Nagpur)
(Ans : X=41 2.33
M„ = 413.98)
71

68. Find the mean, median and mode from the following :|
Below Marks No. of students
10 5
20 9
30 17
40 29
50 45
60 60
70 70
80 78
90 83
100 85
(M.A.
(Ans : X = 4 8 .4 1
M = 4 8 .4 4
M 0= 4 8 )
69. From the figures given below find the mode, median
quartiles
Age : 20-25 25-30 3 0 -3 5 35-40 40-45 45-5
No. of persons : 50 70 100 180 150 120
Age : 50-55 55-60
No. oF persons : 70 50
(B.Com. Agr,
(Ans: M 0= 3 8 .6 4
M = 3 9 .8 6
Qx= 3 3 .8 9
Qs = 4,6.80)

70. (a) In a moderately skewed distribution the values of mod


and median are 64.2 and 68.6 respectively. Find the valui
of the mean.
(B.Com. Osmania
(Ans: X = 70.8

(b) In a .moderately skewed distribution, the Mean is 10 unit


and tne m#de is 7 units. Find the median.
(B.A. Delhi)
(Ans : M = 9 )
72

fc) In a moderately asymmetrical series the values of mean


and median are 26.8 and 27.9 respectively Find the value
of mode.
(B.Com. Mysore)
(Ans : M „= 30.1>

71. In an examination of 675 candidates, the examiner


•implied the following information :

Marks obtained No. of Candidates

Less than 10% 7


„ 20 % 39
„ 30% 95
„ 40% 201
„ 50% 381
„ 50% 545
„ 70% 631
w* 80% 675

Calculate the mean, median and mode.

Hint : Class intervals are 0 -1 0 , 1 0 -2 0 , 2 0 -3 0 etc.,


(B.A. Delhi)
(Ans : X = 4 6 .8 7
M = 4 7 .5 8
M a= 4 8 .2 2 )

72. Find the mean, median and mode of the following distri­
bution.

Class : 2 0 -2 4 2 5 -2 9 3 0 -3 4 3 5 -3 9 4 0 -4 4 4 5 -4 9
Frequency 10 20 12

Class : 5 0 -5 4 5 5 -5 9
Frequency : 3 1
(B. Com. Gujarat)
(Ans : X = 3 7 67
M = 3 7 .5
M 0= 3 7 ,3 )
73

73. From the following table find the first quartile, t


quartile and first decile :

Size of item : 0 -1 0 1 0 -2 0 2 0 -3 0 3 0 -4 0 4 0 -5 0 5 0 -6 0
Frequency 2 18 30 45 35 20
11JM
Size of item 6 0 -7 0 7 0 -8 0
Frequency 6 3
(B.Com. Nagpur)
(Ans : Q1= 2 6 .5 8
Q3= 4 6 .9 3
D l = 1 7 .7 2 )

74. Monthly incomes of families are given helpw in Rs.

2 000, 36, 4 0 0 , 15, 40, 1500, 300, 6, 90, 250,


20, 12, 450, 10, 150, 8, 25, 30, 1200, 60.

Calculate geometric mean of the series


(B.Com. Osmania)
(Ans: G .M = 7 8 .1 1)

75. Find out geometric m^an from the following data :

X : 10 20 30 40 50 60
f : 12 15 25 10 6 2

(A n s : G .M = 2 5 .3 0 )

76. Compute the geometric mean of the following data :

Marks : 0 -1 0 1 0 -2 0 2 0 -3 0 3 0 -4 0 4 0 - 5 0
No. of students : 5 7 15 25 8

(Ans: G M = 2 5 .6 3 )

77. Compute geometric Mean from the following data :

1 0 0 -1 1 0 1 1 0 -1 2 0 1 2 0 -1 3 0 1 3 0 -1 4 0
No. of persons 14 16 30 20
Weight 1 4 0 -1 5 0 1 5 0 -1 6 0
No. of persons 15 5
(Ans: G.M = 126.3)
74

78. Calculate Harmonic Mean from the following data :

Family : A B C D E F G H I J
i .come (Rs): 85 70 10 75 500 8 42 250 40 36

(B.Com. Mysore)
(,Ans : H.M.= 2 8 .8 )

79 Calculate the Harmonic Mean from the following data :

X : 10 12 14 16 18 20
f : 5 1« 20 10 6 1

(A n s : H .M = 1 3.5)

80. Calculate the H.M. from the given data :

Class : 5 -1 0 1 0 -1 5 1 5 -2 0 2 0 -2 5 2 5 -3 0 3 0 -3 5
Frequency : 2 9 29 54 11 5
(A n s: H.M = 1 9 . 6 5 )

81. Following is the distribution of marks in law obtained


by 50 students :
Marks more than : 0 10 20 30 40 50
No. of students : 50 46 40 20 10 3
Calculate the Median
(B.Com, Osmania, May 1983)
(A n s : M = 2 7 .5 )

82. If the mean and median of a moderately asymmetrical


series are 12.9 inches and 12.1 inches respectively. What would
be the value of mode ?
(B.Com Osmania, May 1983)
(Ans : Mode= 1 0 .5 ^

83. Given the following : /Vt = 112, X i = 114, A/a= 140.


X a~-114. Find the value of X la .
(B.Com. Osmania, May 1983)
(A ns: X ia = 1 1 4 )
I
1
75 C H A P TER ill

84. If the mode and mean of a moderately asymmetrical


series are respectively 16 inches and 15.6 inches, what would b9 Measures of Dispersion
its most probable median ?

(B.Com., Osmania Nov., 1983) According to Simpson and Kafka "The measurement of the-
(Ans : Median = 1 5.73) ,, uttemess of the mass of figures in a series about an average is
niiod measure of variation or dispersion."
85. Calculate the Median annual income of a group of
employees from the data given below : The following are the important methods of studying,
ll Annual Income in Rupees •iitpersion :
No. of employees
([iff: Under — 2000 15 1. The Range
,i 2 0 0 0 — 2999 32
300 0 — 3999 2. The Quartile Deviation
65
4000 — 4999 79 3. The Mean Deviation or Average Deviation
'§ 5 0 0 0 — 5999 90
6000 — 6999 A. The Standard Deviation and
i 57
# 7000 — 7999 36 5. The Lorenz curve (This one is graphic method).
/ 800 0 and above 14
Measures of dispersion may be either absolute or relative.
l l (B.Com., Osmania, May 1984)
Absolute measures of dispersion are expressed in the same statistics f
■ (Ans : M = 5 0 3 2 .8 3 )
unit in which the original data are given such as rupees, cms.,
86. Given below is the distribution of profits (in '000 Rs 1 lonnes etc. These values may be used to compare the variation in
b earned by 94 book depots in a certain territory. Find the mo da i wo distributions provided the variables are expressed in the same
1 value. units and of the same average size. A measure of relative dispersion
L tj* it the ratio of a measure of absolute dispersion to an appropriate
Profit No. of book depots average and is also sometimes known as a coefficient o*- dispersion.
\ Below 20 5 Ii is a pure number and is independent of units of measurement.
By, ft. „ 30 14 Therefore, for comparison between two or more distributions;
„ 40 27 employing different units of measurement, say rupees and tonnes,
. „ 50 48 M only a relative measure of variability can be useful.
1] „ 60 68
70 83
„ 80
1. Range
91
90 94 Range is defined as the difference between the highest
value and the lowest value included in the distribution. Range
(A n s: M o d e = 5 0 .0 4 ) (B.Com., Osmania, May 1985)
77
and c o e ffic ie n t of range w ill be computed by applying the follow
ing formulae.

Range = L—S
Where 1 = Largest value, and
S = Smallest value

Coefficient of Range =
L+ S
Illustration 1. The following are the wages of 8 workers
of a factory. Find the range of variation and also compute the
coefficient of range.

Wages in Rs.
125, 175, 230, 80. 95, 195, 275, 150

Solution :

Range = L—S
L - 275
S = 80
Range = 2 7 5 — 80 = Rs. 195
L— S
Coefficient of Range =
L+ S
275 — 80 195
0.549
275 + 80 355

Illustration 2. Calculate range and its Coefficient from the


follow ing data :

Marks 10-20 2 0 -3 0 3 0 -4 0 4 0 -5 0 5 0 -6 0
No. of students 16 20 24 16 8
Solution f
Range — L — S
L = 60
S = 10
Range = 60 — 10 = 50
L— S 6 0 — 10 50
Coefficient of Range = = 0.71
L+ S ~ 6 0 + 10 70
78

2. The Quartiie Deviation


Quartile deviation is a measure o( dispersion based or. the
third quartile (Q3) and the f i r ' t quartile (Q ). Third quartile and
first quartile were explained clearly with illustrations .n chapter II.
The quarti le deviation and coefficient of quartile deviation will be
calculated by applying the following formulae.

Quartile Deviation or Q.D Qs— Qi

Qt Qi
Coefficient of Quartile Deviation
Q , + Qy
Calculation of Quartile Deviation :
(A) Individual Series
H lustration3. Find out the value of quarti le deviati on and
its coefficient from the following data :
Roll No. : 1 2 3 4 5 6 7
Marks : 30 42 60 18 45 24 75
Solution : Calculation of Quartile Deviation
Marks are arranged in ascending order : 18 24 30 42 45
60 75
A/4-1
Q. = Size of — \— th item
1 4
7+1
=S ize of th item = 2 nd item

Size cf 2nd item is 24. Thus 0 = 2 4 .


/ N+1 \
Q, = Size of 3 ( — 4 — 1th item

3X8
Size of 3 (■ 7+ ) th item 6th item

Size of 6th item is 60. Thus Q3 = 60


Qg-- Qj
Q.D. =
2
6 0 — 24
2
Coefficient of QD.
Q d -Q i
_ 6 0 — 24
_ 60 + 2 4
36

(B) Discrete Series


Illustration 4 Compute the value of quartile deviation and
its coefficient from the following date :
Central Size of item : 1 2 3 4 5 6 7 8 9 10
Frequency : 2 9 11 14 20 24 20 16 5 2
(B.Com, Rajasthan)
Solution :
Calculation of Quartile Deviation and its coefficient

Central size Frequency c f.


of item
1 2 2
2 9 11
3 11 22
4 14 36
5 20 56
6 24 80
7 20 100
8 16 116
9 5 121
10 2 123

Q, = Size of — th item = ■= 31st item


4 4
size of 31st item is 4. Thus Q1= 4

Q.i = Size of 3 ^ th item — ^ X | ^ * ° 9 3 r d item

Size of 93rd item is 7. ThusQ9^= 7

Q.D. = °1 _ 7— 4 3
1.5
2 ” 2 ~~
O

7— 4
I
m

i-= 0 .2 7 3
I

Q i 4*Q|. 7+4 11
80

(C) Continuous Series :


Illustration 5 : Calculate the Quartile deviation and its
coefficient from the following data :

( loss Interval :
10-15 1 5 -2 0 2 0 -2 5 2 5 -3 0 3 0 -4 0 4 0 -5 0 5 0 -6 0 6 0 -7 0
I requency :
4 12 16 22 10 8 6 4
(C.A. Intermediate)

Solution i
Calculati on of Quartile Deviation and its coefficient.

Class Interval Frequency c.f.

1 0 -1 5 4 4
1 5 -2 0 12 16
2 0 -2 5 16 32
2 5 -3 0 22 54
3 0 -4 0 10 64
4 0 -5 0 8 72
5 0 -6 0 6 78
6 0 -7 0 4 82

82
=Size of i|L th item = ■= 20.5th item
x 4
lies in the class 2 0 -2 5
P ■

H*1

^ (q -O
II

'i
2^— 20
Qi = 20+ (2 0 .5 -1 6 )

20+ X 4.5

22.5
= 20 +
16
- 2 0 + 1 .4 1
= 21.41
81

3 N 3 X 82
<?3 =Size of —jj- th item = —- — = 6 1 .5th item

9 3 lies in the class 3 0 — 40

Q. = / 1 + M * (q?-c)
h ^
4 0 — 30
Qa = 30 + (6 1 .5 — 54)
10

= 30 + ~ X 7.5

= 30 + 7.5
= 37.5
Q3 — Q t 3 7 .5 — 21.41 _ 16.09
Q.O =8.045
2 ~ 2 2
Q ,— Q i 3 7 .5 — 21.41
Coefficient of Q.D. =
Q 3 + Q i ~ 3 7 .5 + 2 1 .4 1
16.09
= 0.27
58.91

3. The Main Deviation


The Mean Deviation is the arithmetic mean of the deviations
of the individual values from the average of the given data. The
average which is frequently used in computing the mean deviation or
average deviation is mean or median.

Computation of Mean Deviation : ^

(A ) Individual Series : The procedure of comeuting mean deviation


for individual series involves the following steps.

(i) Calculate the mean or median of the series.


(ii) Take the deviations of items from mean or median
ignoring±signs and denote these deviations by | D |
(iii) Obtain the total of these deviations, i.e., 2 | D |
(iv) Apply the following formula to arri/e at Mean Deviation.
82

2 ID I
Mean Deviation or M. D. = — -

Where | D | => Deviations from mean or median ignoring signs


N = Number of observations.

Coefficient of Mean Deviation :*


_ . _ M.D.
Coefficient of M.D. = n ---------- .
Mean or Median or Mode

Note : Only the measure of central value which has been used
for calculating deviations should be taken in the denominator.
Illustration 6 Calculate the Mean Deviation from (i) Mearv
(ii) Median and (iii) Mode in respect of the marks obtained by nine
students given below.
MarKs (out of 25) 7, 4, 10, 9, 15, 12, 7. 9, 7,
Solution t Calculation of mean deviation from mean, median and
mode.
Mean
„ 2X
* ” N
2 X = 7+4+10+9+15+12+7+9+7
= 80
v __ 80
A ~ g 8 .9

Median :

M = Size o f H + 1 th item

94-1
= size of — _ 5 th item

For calculating median the items are arranged in ascending


order :
4 7 7 7 9 9 10 12 15
Size of 5th item = 9
Hence median = 9
83

Mode :
M 0= 7 . (Since 7 is repeated the maximum number of
times i. e., 3)

Calculation of Mean Deviation

Deviations from Deviations from Deviations from


Marks Mean 8.9 Median 9 Mode 7
X I D | ID | I D |
7 1.9 2 0
4 4.9 5 3
10 1.1 1 3
9 0.1 0 2
15 6.1 6 8
12 3.1 3 5
7 1.9 2 0
9 0.1 0 2
7 1.9 2 0

2 I D | = 2 1 .1 2 D | =21 2 D | =23

3T | D I 21 1
Mean deviation (from Mean) = — —L = —g—= 2.34

Mean deviation (from M e d ia n ) = ^ - ^ J - = —^ — = 2.-33

Mean deviation (from Mo d e ) = — 2. 56


/V 9
Illustration 7. Calculate coefficient of Mean deviation *%•
using the data given in the illustration 6.

Solution:
Coefficient of M.D. (from M ean )= o ,26
' Mean 8.9
»* TOO
Coefficient of M.D (from Median) = - ■ — = — - — = 0 . 2 6
Median 9

Coefficient of M.D. (from Mode) = = 0.37


Mode 7
(B) Discrete Series The procedure of computing the maaff
deviation for Discrete Series involves the following steps :
84

(i) Calculate the mean or median of the series.


(ii) Take the deviations of the items from mean or median
ignorning signs and denote them by | D |

(iii) Multiply these deviations by the respective frequencies


and obtain the total "Zf \ D | .
(iv) Apply the following formula to arrive at mean deviation-

M .D ~ S M Q 1
N
Illustration 8. Calculate mean deviation (take deviations
from mean) from the following data. Also find out its coefficient.
X : 2 4 6 8 10
f : 1 4 6 4 1
(B.Com. Banarasl

Calculation of Mean Deviation

Deviations from Mean 6


X f fX \D \ f \ D\

2 1 2 4 4
4 4 16 2 8
6 6 36 0 0
8 4 32 2 8
10 1 10 4 4

A /= 1 6 £ /X := 96 2 f [ D | =24

-ZfX 96
= 6
N 16
D I 24
M.D - 2 / = 1 .5
N 16
Mean deviation
Coefficient of M D =
Mean

Illustration 9 . Calculate M.D. from Median for the following


series. Also find out its coefficient,
Marks : 10 15 20 30 40 50
Frequency : 8 12 15 10 3 2
85

Solution : Calculation of Mean Deviation

Marks f C. f Deviations from Median 20 f \ D 1


ID |
10 8 8 10 80
15 12 20 5 60
20 15 35 0 0
30 10 45 10 100
40 3 48 20 60
50 2 50 30 60

N = :50 y J \D \ =360

A/+1
Median = Size of — th item

50+1
:25.5th item

Size of 25.5th item is 20


Hence Median = 20

_ Zf\D I 360
M.D 7.2
” N ~ 50
MD. 7.2
Coefficient of M.D = 0.36
Median 20

( C) Continuous Series ; For calculating M.D. in continuous series


the procedure remains the same as discussed above in the discrete
series. The only difference is that here we have to obtain the mid­
points of the various classes and take deviations of these points
from Mean or Median. The formula is also the same, i.e.,

M.D - SM Q l
~ N

Illustration 10. Find out Mean deviation and its Coefficient


from the given data by using mean.
Class : 0 -3 3 -6 6 -9 9 -1 2 1 2 -1 5 1 5 -1 8 18-21
Frequency : 2 7 1 0 12 9 6 4
'iiiation : Calculation of Mean Deviation

(lass mp f (m— 1 0.5 )/3 fdx |D | f \ D |


m dx (m-X)

0— 3 1.5 2 —3 — 6 9.18 18.36


3— 6 4.5 7 —2 — 14 6.18 4 3.26
6r— 9 7.5 10 —1 — 10 3.18 3 1 .8 0
ft—12 10.5 12 0 0 0.18 2.16
12— 15 13.5 9 + 1 + 9 2.82 25.38
18— 18 16.5 6 + 2 + 12 5.82 3 4 .92
18—21 19.5 4 +3 + 12 8.82 3 5 .28

N: =50 -£ fd x = 3 2 / | D | = 191.16

X fd x
X=A+
N
4 = 1 0 .5
C= 3
■2fdx=3
A /=50

* = '° - 5+ 4 > x3

= 1 0 .5 + 0 .1 8 = 10.68
191.16
M .D = ^ = - -- 3.82
50

M .D _ 3.82
Coefficient of M .D = 0.36
X 10.66

Illustration 11. The following is the age distribution of 80


policy holders insured through an agent :

Age group : 1 6 -2 0 2 1 -2 5 2 6 -3 0 3 1 -3 5 3 6 -4 0
Mo. of Policy holders i 8 15 13 20 11
' r*■r'"'.; ,vv,: •• • v■L-
Age group : 4 1 -4 5 4 6 -5 0 5 1 -5 5 5 6 -6 0
.Mo. of Policy holders : 7 3 2 1
Calculate mean deviation from the median.
87

Solution i Calculation of Mean Deviation

Age group f c.f. m.p. (m— M ) f \D |


m m— 31.5
•"t r> » w . \D \

1 6 -2 0 8 8 18 13.5 108.0
2 1 -2 5 15 23 23 8.5 127.5
2 6 -3 0 13 36 28 3.5 45.5
3 1 -3 5 20 56 33 1.5 30.0
3 6 -4 0 11 67 38 6.5 71.5
4 1 -4 5 7 74 43 11.5 80.5
4 6 -5 0 3 77 48 16.5 49.5
5 1 -5 5 2 79 53 21.5 43.0
5 6 -6 0 1 80 58 26.5 26.5
oo
o

Zf\D =582
II

Median item = — = —-p = ' tem

Median lies in the class 3 1 -3 5 or 3 0 .5 -3 5 .5

M= L 1+ (m -c )
'i
3 5 .5 — 30.5
= 3 0 .5 + (4 0 —36)
20

= 3 0 .5 +

= 3 0 .5 + 1 = 31.5
^ f \ D \ _ 582
MD— 7.28
N ~ 80

4. The Standard Deviation


Standard Deviation may be defined as the square root of the
arithmetic mean of the squares of deviations from the arithmetic
mean. It is usually denoted by the Greek letter a (read as sigma).
It measures the absolute dispersion of a distribution.
88

Calculation of Standard Deviation :

(A) Individual Series : There are two methods of calculating


standard deviation :

(a) By taking deviation of the items from the actual mean


(b) By taking deviation of the items from an assumed mean.

(a) Deviations taken trom Actual Mean i The procedure of


computing standard deviation is given below.

(i) Calculate the mean of the series, i.e., X-

tii) Take the deviations of the items from the mean, i.e,, find
(X -X )- Denote these deviations by x.

(iii) Square these deviations and obtain the total 2x?.

(iv) Divide 2 x a by the total number of observations, i.e., N


and extract the square root. This gives us the value of
standard deviation.

Thus the formula :

(b) Deviations taken from Assumed Mean : When the actual


mean is in fractions, it would be difficult to take deviation from it.
In such a case the deviations should be taken from an assumed
mean.

The following steps must be followed :

(i) Take the deviations of the items from an assumed mean,


i,e., obtain (X—A ). Denote these deviations by d. Take
the total of these deviations, i,e., obtain ~S,d.

(ii) Square the deviations and obtain the total "Zdz.

(iii) Apply the following formula :


89

Illustration 12 : Compute standard deviation from tha


following data of Income of 10 employees of a firm.
Income (Rs.) 100, 125, 130, 140, 165, 170, 180, 160, 145, 185
Solution t_______Calculation of Standard Deviation________________
Income (Rs.) (X— X) X = 1 5 0 x2
X x
100 — 50 2500
125 — 25 625
130 — 20 400
140 — 10 100
165 + 15 225
170 + 20 400
180 + 30 900
160 + 10 100
185 + 35 1225
145 — 5 25
2X=1500 Zx=0 2 x 2= 6 5 0 0
Z X _ 1500
X 150
N 10
Sx2
<T
- V N
6500
10
= V 650
= 25.495
Illustration 13 ; Calculate the standard deviation of marks
obtained.
5, 10, 20, 25, 40, 42, 45, 48, 70, 8 0 .
Solution : Calculation of Standard Deviation
X (X— >4) >4=40 dz
d
5 — 35 1,225
10 — 30 900
20 — 20 400
25 — 15 225
40 0 0
42 + 2 4
45 + 5 25
48 + 8 64
70 + 30 900
80 + 40 1,600
'Zd = — 15 £ d z = 5,343
90

/ 5 -343 t -is y
V 10 v 10 /
/ 225
— / \ J 534.3 100

= \ / 534.3 — 2,25
= V 532.05
= 23.066

{ B) Discrete Series Any of the following methods may be


applied for computing standard deviation.
1. A ctual M ean M e th o d .
2. Assumed M ean M e th o d .
3. Step deviatio n M e th o d .
1. A ctu al M ean M e th o d : The following steps must be
followed for calculating standard Deviation.

(i) Calculate the mean of the series.

(ii) Find the deviation of the items from the mean i.e.,
(X— X ) and denote these deviations by x.
(iii) These deviations are then squared and multiplied by
the respective frequencies and then obtain the total
i.e., S f r 2
fti
(iv) Apply the following formula

(2 ) Assumed M ean M e th o d : The following steps must


be followed for calculating Standard Deviation :
(i) Take the deviation of the- items from an assumed mean
and denote these deviations by d.
(ii) Multiply these deviations by the respective frequencies
and obtain the total, ~2.fd.
91
(iii) Square the deviations, i.e., calculate d 2
<iv) Multiply the squared deviations by the respective frequencies I
and obtain the total, "2,fda.
<v) Apply the following'formula :

/2 W 2 f Zfd\'
V N V N )
Where d — ( X—A)
/4=Assumed mean

(3 ) Step D eviatio n M e th o d : This method is simila


the Assumed mean method and we also take a common factor from
the given data for computing standard deviation.

, S ftfa* / *Zfd,
^fdx\ ^ n
a ~ V N V N /
Where,
(X -A )
dx =
>1=Assumed mean
C=Common factor.
Note : The use of this formula simplifies calculations.

Illustration 14 : Calculate Standard Deviation with the help


of assumed mean :
Marks : 10 20 30 40 50 60
No. of students : 8 12 20 10 7 3
Solution : Calculation o* Standard Deviation

Marks
X (V ! „=39 fd fd 2
a
10 8 — 20 — 160 3 ,200
20 12 — 10 — 120 1,200
30 20 0 0 0
40 10 + 10 + 100 1,000
50 7 + 20 + 140 2,800
60 3 + 30 + 90 2,700

N = 60 % f d = + 50 = 10,900
92

/ 2W 2 ^' 's.fd y
r=V . N )

1 10,900 ( 50 Y
=v 60 V 60 /

/ 10,900 2500
=y 60 3600

V7 181.67 — 0.69
= \ / 180.98
= 13.45

iC) Continous Series ; In this series any of the methods discussed


lor discrete series can be used. However, in practice it is the step
deviation method that is mostly used.

The following steps must be followed for computing standard


deviation in this series.

(i) Find the Mid-points of various classes.


(ii) Take the deviations of these Mid-points from an assumed
mean and divide the same by a common factor.
(m—A )
ie.------------and denote this column by dx
c
(.iii) Multiply the frequencies of each class with these deviations
and obtain ~2,fdx

(iv) Square the deviations and multiply them with the respective
frequencies of each class and obtain S fr/x 2.

(v) Apply the formula :

C
T= v ^ _ f B V x c
V N N )
(m — A )
dx =

Where, A = Assumed mean


C = Common factor
m = Mid Point of each class.
Illu s tra tio n 'll'. Compute the standard deviation of tl|
following data :
Wages (Rs.) : 1— 3 3— 5 5— 7 7— 9 9— 11
No. of workmen: 15 18 27 10 6
(B.Com, Andhf(
Solution : Calculation of Standard Deviation

Wages m.p f (m— A )jc = d x fdx fdx•


m A ~ 6, C = 2
1 - -3 2 15 —2 — 30 60
3 - -5 4 18 —1 — 18 18
5 - -7 6 27 0 0 0
7 - ■9 8 10 + 1 f 10 10
9 - ■11 10 6 + 2 + 12 24
N = 76 :2 fd x = — 26 ■2fdx2 = 112

/2 W x a
g =
V N - { n r ) ' x c

-2 6 y v „
* / 112 (
V 76 V

— / 112
676 X 2
V 76 5776 X 2
= — 0 12X2
= V/1.35 X 2
= 1.16 X 2
f■l
2.32

Com bined S tandard D eviation : This is denoted by g ia and


is computed as follows :

_ //V 1g , a + N ag 22 + /V1rf13 + / \ W
13 ' A/ N\ + N 2
Where g ia = Combined standard deviation.
g( = Standard deviation of first group.
ga = Standard deviation of second group,
di = (^ i r/a= ( X * —
94

Note This formula can be extended to find out the standard


rfuvlation of three or more groups.

/ lustration 16. The number of members, mean and standard


tlfiviation of two distributions are :

Number of members 280 350


Mean 45 54
Standard deviation 6 4

Find the mean and standard deviation of the distribution


Firmed by the two distributions taken together. (B,A, Delhi)

Solution :
/v 1x , + n , x 2
* 1. N x+ N t
N y = 280, Wa = 3 5 0 , Xj - 45, X a = 54
(2 8 0 x 4 5 ) + (3 50X 54 ) 12600 + 18900
X ja " 280 + 350 ~ 630
31500
^ia 630 50

_ _ //Vi<Tia+A/a<T,*+/V1</Ja+/Varfaa
“ V 7^+N2
Nx = 280, Na = 350, <jx = 6, = 4
dx = ( X x - X .a ) = ( 4 5 - 5 0 ) = - 5
e/a =* (X a- X i a) - (5 4 -5 0 ) - 4
'280(6)2 + 3 5 0 (4 )a + 280 (—5)a + 350(4)*
>12
V 280 + 350

/2 8 0 (3 6 ) + 350(16) + 280(25) + 350 (16)


" V 630
080 + 5.600 + 7,000 + 5,600
V *
=v/2-S§°
<*ia = 6 .7
95

Coefficient of Variation: The s ta n d a r d


d e v i a t i o n d i s c u s s e d a b o v e i s an a b s o l u t e m e a su re
of d is p e r s io n . The c o r r e s p o n d in g r e la tiv e
m e a s u r e i s known a s t h e C o e f f i c i e n t o f s ta n d a r d
d e v ia tio n . The c o e f —f i c i e n t o f v a r i a t i o n is
d e n o te d b y C .V . and i s o b t a in e d a s f o l l o w s .

Coefficient of variation or C.V, = —^ — X 100

Coefficient of variation is used in such pioblems where we


want to compare the variability of two or more than two series. That
s e r ie s for which the coefficient of variation is greater is said to be
more variable.

fflustration 1 7 i A factory produces tw o types of electric


'amps A and B. In an experiment relating to their life, the fallowing
results were obtained.

Leght of life No. of Lamps No. of Lamps


(in hours) A B

5 0 0 — 700 5 A
7 0 0 — 900 11 30
9 0 0 — 1100 26 12

1100-1-1300 10 8
1300— 1500 8 6

60 60

Compare the variability of the life of the two varities using


coefficient of variation.

(C.A. Intermediate)
96

Calculation of Cofficient to Variation (Type A)

mp (m— 1000)
Length of Life f dx = fdx fdx*
3 m 200

5 0 0 -7 0 0 600 5 —2 — 10 20

7 0 0 -9 0 0 800 11 —1 — 11 11
9 0 0 -1 1 0 0 1000 26 0 0 0
1 100-13 00 1200 10 + 1 + 10 11
1300-1500 1400 8 +2 + 16 32

N = 60 2 f d x = 5 -2 fdx*
=73

2 fdx
Mean = A + ~~j\j X C

= 1 0 0 0 + — 60" x 2 00

= 10 0 0 + 16.67 = 1,016.6 7

J.fd x* /
<y= V / N V N )

_ / 73
( 5 ^ 200
60 V 60 )
73
25 X
- V - 60 :3600

= v/1 2 2 — 0.01 X 200

1.1 X 2 0 0 = 220

<T 220
C.V. = - j j - X 100 = ~1t0 16.67 X 100 = 21.64
(7 ) 97

Calculation of Coefficient of Variation (Type B)^

Length of Life m.p f (m— 1000) fdx fdx2


m. 200

5 0 0 - 700 600 4 —2 — 8 16
7 0 0 - 900 800 30 —1 — 30 30
9 0 0 -1 1 0 0 1000 12 0 0 0
1 1 0 0 -1 3 0 0 1200 8 + 1 -Jr 8 8
1 3 0 0 -1 5 0 0 1400 6 + 2 + 12 24

N--= 60 X f d x = — 18
-7 8

•2. fdx -1 8
CJ
X

:= a + — 1 0 0 0 + - 60 X 200 = 1000— 60 = 940


2:

fZ fd x 2 ( 2 fdx y
<j= x c
V N \ N

( - 18 y X 200
=
V/ 78
60 \ 60/

32 4 v 200
=
V/ 78
60 " 3600
= V 1.3 — 0.09 X 200

ssx v /1 .2 1 X 2 0 0

— 1.1 X 200 = 220

220
C .V . - -|x 100 X 1 00 = 23.40
9 40

Hence type B is more variable than type A,

V a ria n c e : According to R.A. Fisher, the term variance was


=used to describe the square of the standard deviation.

Variance «= a 2
or a - v/Variance
98

Important illustrations

Illustration 18. Calculate the quartile deviation and. its


Minfficient from the following data :

Months Monthly earnings Months Monthly earnings

1 39 7 42
2 40 8 43
3 40 9 43
4 41 10 44
5 41 11 44
6 42 12 45
(B.Com,, Meerut)

Solution i
Calculation of Quartile Deviation and its Coefficient.
The values are already arranged in ascending order.
/V4-1
Q, — Size of — j - t h item
4

= = 3 .2 5 item

Size of 3.25th item = 3rd ite m + 0 .2 5 (4th item— 3rd item)


= 4 0 + 0 .2 5 (4 1 — 40)
Q l = 40.25

Q8 = Size of 3 ^ th item

= = 9.75th item
4

Size of 9.75th item —9th ite m + 0 .7 5 (10th item— 9th item)


= 4 3 + 0 . 7 5 (4 4 — 43)
Qs= 4 3 .7 5

0 .0 = = ^ 7 5 -4 0 .2 5 = J O , 76
Am i. Z
99

Coefficient of Q. D. Q3— Qg_ 4 3 .7 5 — 40 25


Q3+ Q , 4 3 .7 5 + 4 0 .2 5
3.5
= 0.04.
84

Illustration 19. Calculate the appropriate measure of dis­


persion from the following data :
Class interval : Less than [35 '3 5 -3 7 3 8 -4 0 4 1 -4 3 o va 43
Frequency : 14 62 99 18 7

Solution i Due to open-end class intervals on the two extreme side*


of the distribution Q. D. would be the most appropriate one.

Calculation of Quartile Deviation

Class interval Frequency c.f.

Les6 than 35 14 14
3 5 — 37 62 76
38 — 40 99 175
41 — 43 18 193
over— 43 7 200

N 200
Q, = Size o f-j-th item = —j— = 5 0 th item
1 4 4

Q j lies in the class 3 5 — 37,

Qi = K +

= 34.5 + 3 - —g (50— 1 4)

= 34.5 + ^ X 36

= 34.5 + 1.74

= 3 6.24.
\

100

3 a/ 3 y 900
arg = Size of th *tem = — 4 — — 1 5 0 + item

@3 lies in the class 38 — 40

03 = /, + '^ P ^ ( m - c )
'i
4 0 .5 — 37.5
= 37.5 + (1 5 0 — 76)
99

= 37.5 X 74
99
= 37.5 + 2.24
~ 39.74
QD - Qa— Qx _ 3 9 - 7 4 - 3 6 .2 4 ^ 3.5
2 2 2

Illustration 20. The following are some of the particulars of


distribution of weights of boys and girls in a class.

Boys Girls
Number 100 50
Mean weight 60Kgs. 45Kgs.
Variance 9 4

(i) Find the standard deviation of the combined data

(ii) Which of the two distributions is more variable ?


(I.C .W .A .)

Solution : (i) Calculation of combined standard deviation :

_/A /1a 12+A /a( jJ2+ A/1d 12 + A/2d 22


V /V, + /V2
A/i = 100. N2 = 50,, X» = 60, X3 = 45

d i = (Xj—X 12 ). = ( X ; - X l 2)
N
Ni + Na ■

100X60 + 50X45
100 + 50
Coeffici'
55
#

mula

/)X 4 1 + (1 0 0 X 2 5 )+ (5 0 X 1 0 0 )
fia - * y —
100+50

, 900+200+2500+5000
” V ^50
8600
ygQ = v/5 7 .3 3 = 7.57

(ii) For finding out which of the two distribution is -a m


variable. We w ill have to compare the Coefficient of variatiataf
the two distributions.
, ()C» ;j ,}$ M
Boys :

C,V. X 100

3
= W x 100 = 5

Girls

C.V. = X 100

2
= “4 5 " X 1 0 0 = 4 .4 4

Since the coefficient of variation is greater for boys, h en o ift*


first distribution i, e,. that of boys is more variable.

Note : a - ^/variance
102

List of Im p o rta n t Fo rm ula e

Discrete and Continuous


individual Series Series

t , Range : Range = L—S


Range «■ L—S / _c
Coefficient of Range = — -
L—S
Coefficient of Range = j~^s

t. Quartile Deviation :
Q.D. = Q3“ Q l
Q 3- q i
2 Coefficient of Q.D.
Coefficient of Q.D. _ ^3—Q l
Q .-Q i Q s+Q x
Q »+Q i

1. Mean Deviation : M.D - ^ | D |


N
d _ ^ fD L
^ ' N Coefficient of M.D.
Coefficiant of M.D. M.D.
M D. ~ Mean or Median or Mode
" Mean or Medi an or Mode

♦ Standard Deviation: (a) Actual Mean Method


(a) Actual Mean Method
V N
V N
(b) Assumed Mean Method
(b) Assumed Mean Method

/2 d 8 ( Td \ a _ / S tt2 / V
'Zfd
c ” V N K N J ° V N \ N )
iiscrete and Continuous
CoefficJL
Series

Step Deviation Method

N V N )

/ tion or <jia

/ N ^ S + N o S + N ^ S + N td *
'u V Nt+ N t

d\ ™ X i Xia
d j — Xa X 12

6. Coefficient of variation or C.V.

a
c.v. -x 100

7. Variance :
Variance = <ja

or _________
<y *= y/Variance

8. Inter Quarti le Range = Q3 — Q1

y. Loe i f i c i e n t o f s ta n d a r d d e v i a t i o n =

it i
104

PROBLEMS

1. The following are the marks of 6 students in a class,


find the range of variation and also compute the coefficient of
■SMQ0.
20, 50, 90, 35, 80, 75
(Ans : (a) Range = 70
(b) Coefficient of Range = 0 6 3 6 )

2. The following are the marks of 80 students of a class,


t.nd the range of variation of marks and also compute coefficient
ol range.

Marks : 0 -1 0 1 0 -2 0 2 0 -3 0 3 a - 40 4 0 -5 0
No, of students : 4 12 20 18 15

Marks : 5 0 -6 0 6 0 -7 0 7 0 -8 0
No. of students : 8 §2 1

(B.Ctrm. Osmania)
(Ans : (a) Range = 80
(b) Coefficient of Range = 1)

3. Find out the value of quartile deviation and its coaff icient
from the following data :

of the week No. of units sold


1 24
2 90
3 40
4 56
5 80
6 30
7 60
(Ans : (a) Q.D. = 25
(b) Coefficient of Q.D. = 0.455.
105

4. From the following data, find quartile deviation and Its


coefficient.
Months : 1 2 3 4 5 6 7 8 9 10
Sales
( ’0 0 0 Rs.): 70 75 80 82 83 85 85 86 86 81
Months : 11 12
Sales
( ’000 Rs.): 89 92
(Ans : (a) Q.D. = 3.5
(b) Coefficient of Q.D.= 0 .0 4 )
5. Compute quartile deviation from the following data :
Height in inches 58 59 60 61 62 63 64 65 66
Not of Students : 15 20 32 35 33 22 20 10 8
(B.Com., Agra)
(Ans : Q.D. = 1.6)
6. Compute coefficient of quartile deviation from the follow­
ing data :
Marks: 15 20 35 40 55 70
No, of Students : 4 7 15 8 7 2
• . ITjXTzJ,pj mjm
(Ans : Coefficient of Q.D. = 0.33)*
7. Compute quartile deviation from the following data :
Size : 4 - 8 8 -1 2 1 2 -1 6 1 6 -2 0 2 0 -2 4 2 4 -2 8 2 8 -3 2
Frequency: 6 10 18 30 15 12 10

Size : 3 2 -3 6 3 6 -4 0
Frequency : 6 2
(C.A.)
(Ans / Q.D. = 5.21)
8. Calculate the quartile deviation and its coefficient fr
the given data :

W ag es: 2 0 -2 5 2 5 -3 0 3 0 -3 5 3 5 -4 0 4 0 -4 5
No. of workers: 2 10 25 16 7
.
(B.Com., Jodhpur)
(Ans i (a) Q.D. = 3.45
(b) Coefficient of Q.D. = 0 .10)
106
9. Compute the quartile deviation for the following distri­
bution :
Marks: 0 -9 1 6 -1 9 2 0 -2 9 3 0 -3 9 4 0 -4 9 5 0 -5 9
•'■quency : 3 9 15 30 18 5
(B,A., Madras)
(A n s: Q.D. = 8.17)

10. Calculate the appropriate measure of dispersion from


dm following data :
wages : Below 35 3 5 -3 7 3 8 -4 0 4 1 -4 3 over 43
No of wage
earners: 14 60 95 24 7
(B.Com., Bangalore)
( Ans i Q.D. = i.7
Hint : Due to open-end class intervals on the two extreme
iiilos of the distribution, quartile deviation would be the most app-
Miprlate measure of dispersion for the data.

11. From the following data compute quartile deviation and


h* coefficient.

Mid-point : 15 20 ?5 30 35 40 45
Frequency : 15 33 5fe 103 40 32 10
Ans : (a) G( D- = 4.56
(b) Coefficient of Q.D. = 0.156)

12. From the following distribution, calculate the value of


ilie median, mode and quartile deviation.

* f X f
300— 399 14 800— 899 62
400— 499 46 900— 999 48
5 00— 599 58 1 0 0 0 -1 0 9 9 22
600— 699 76 1 1 0 0 -1 1 9 9 6
700— 799 68
* c ;a „ Intermediate)
(Ans : M = 7 0 8 .3 2 ;: M 0= 6 6 5 .7 3 ; Q ,D .= 1 4 6 .1 6 )
13. Compute the mean deviation from (i) Mean (ii) Medltfi
and (iii) Mode in respect of the marks obtained by nine student
given below : ■
Marks (out of 50) : 14, 8, 20, 18, 30, 24, 14. 18, 14
(Ans t (i) M .D .= 4 .6 l
(ii) M .D .= 4 .6 |
(iii)M .D .«5.11)

14. Compute the coefficient of mean deviation from (I)


Mean (ii) Median and (iii) Mode for the above data. V
(Ans i (i) Coefficient of M .D = 0 .2 6
(ii) — 0.26
(iii) „ „ -0 .3 7 )

15. Calculate mean deviation and its coefficient from th*


following data (take deviations from Mean.)
Marks : 10 15 20 30 40 50
Frequency : 8 12 15 10 3 2
(Ans : (i) M .D .= 7 .8 4
(ii) Coefficient of M .D .= 0.36)

16. Find the mean deviation and the mean coefficient of


dispersion from median of the following series :
Size : 5 6 7 8 9 10
Frequency 8 12 18 8 3 1
(Osmania, B.Com )
(Ans : (i) M .D = 0 .9
(ii) Coefficient of M.D. = 0.13)

17. Calculate the mean deviation from mean for the follow­
ing series. Also find out its coefficient.
Marks : 0 -1 0 1 0 -2 0 2 0 -3 0 3 0 -4 0 4 0 -5 0
No. of students : 5 8 15 16 6
(B.Com.)
(Ans : (i) M.D = 9.44
(ii) Coefficient of M .D.= 0 .3 5 )
108

18. Calculate mean deviation from median from the follow­


ing data :
■>«!H-interval : 2 -4 4 -6 6 -8 8 -1 0
frequency : 3 4 2 1
(I.C.W .A., Intermediate)
(Ans: M.D. = 1 .4 )
19. Calculate mean deviation from median for the following
•fata :
Maiks less -than : 80 70 60 50 40 30 20 10
No. of students : 100 90 80 60 32 20 13 5
(B.Com. (Hons.) Delhi))
(Ans : M.D.= 1 4 .2 9 ).
20. deviation from median from the fo llo w -
Calculate mean
mg data :
Wages (in Rs.) No. of persons
Mid points
125 3
175 8
225 21
275 6
325 2
(Ans : M.D.= 3 1 .6 1 )
21. Compute standard deviation from the following data of
the income of 10 employees of a firm :
Income (Rs.) 1 5 0 ,1 2 5 , 2 0 0 , 3 5 0 , 2 5 0 , 2 2 5 , 3 0 0 , 3 2 5 ,4 0 0 ,4 5 0 .
(A ns: <j = 100.9)
22. Calculate standard deviation in respect of the marks
obtained by 10 students.

50, 55. 57, 49, 54, 61, 64, 59,, 58, 56.
(Ans : a = 4 . 3 8 )
23. Find the mean value and the standard deviation of the
daily out put of finished articles :
No. of articles : 18 19 20 21 22 23 24 25 26 27
No. of workers : 3 7 11 14 18 17 13 8 5 4
(B.Com. Madurai)
(Ans: X : 22.38
O : 2 .1 8 )
109

24. Compute the Standard Deviation of household size frfl


the following frequency distribution of 500 households covered lf|
sample survey :
House hold size : 1 2 3 4 5 6 7 8 f
No. of households : 92 49 52 82 102 60 35 24 f
(B.A. Calcutll
(A n s <y=2.10|

25. Calculate the Standard Deviation for the following data |


Marks obtained : 0 -1 0 1 0 -2 0 2 0 -3 0 3 0 -4 0 4 0 -$ |
No. of students : 5 8 15 16 6
(B.Com. Poon|)
(Ans : o = 1 1 .4 9 )

26. Calculate Standard Deviation from the following data :


Mid points : 1 2 3 4 5 6 7 8
Frequency 2 60 101 152 205 155 79 40 1
J.C .W .A )
(A ns: o = 1.57)

27. Calculate the mean and the standard deviation of th#


quantity of grain produced for the following data :
No. of kilograms No. of plots
of grain produced
2 0 0 — 299 5
3 0 0 — 399 9
4 0 0 — 499 15
5 0 0 — 599 17
6 0 0 — 699 22
7 0 0 — 799 17
8 0 0 — 899 7
9 0 0 — 999 6
1 ,0 0 0 — 1,099 2
(B.Com. Gauhati)
(Ans : X = 6 1 2.5
0 = 1 8 8 .5 )

28. From the following data calculate Mean and Standard


Deviation
/

110

*u't under : 10 20 30 40 50 60 70 no
hi of persons
dying : 15 30 53 75 100 110 115 125
(B.Com. Keral.i)
(Ans : X ■ 35.16
a : 19.76)

29. From the following data, calculate Mean, Median, Mode


•••nl Standard Deviation.

M a ry : 100-110 110-120 120-130 130-140 140-150


No. employed : 12 34 72 120 156
•olary : 150-160 160-170 170-180 180-190
No. employed : 112 82 36 16

(•4/w: X = 1 45.53; M = 145.26; M a=144.50; <r = 17.34>

29 (a) From a central frequency distribution consisting of 18


observations the X and the <j were found to be 7 and 4 respectively,
»nt on comparing with the original data it was found that a figure
l * was miscopied as 21 in calculations. Calculate the correct X
*nd <y.
Ill ;•\ ' :.. • 1; 'i "■ \
(Ans : Correct X -6 .5 ; Correct cr —2.5)

30. From the data given below state which series is more
variable :

Variable Series A. Series B.


1 0 -2 0 10 18
2 0 -3 0 18 22
3 0 -4 0 32 40
4 0 -5 0 40 32
5 0 -6 0 22 18
6 0 -7 0 18 10
(C.A.)
(Ans : C.V. (A) - 3 3 .34
K it C.V. (B) - 37.2
Series B is more variable compared to A)

109

24. Compute the Standard Deviation of household size from


the following frequency distribution of 500 households covered in a
sample survey :
House hold size : 1 2 3 4 5 6 7 8 9
No. of households : 92 49 52 82 102 60 35 24 4
(B.A. Calcutta)
(A ns.: < j= 2 .1 0 5 )

25. Calculate the Standard Deviation for the following data :


Marks obtained : 0 -1 0 1 0 -2 0 2 0 -3 0 3 0 -4 0 4 0 -5 0
1 No. of students : 5 8 15 16 6

> (B.Com. Poona)


If ' ’ 11 ’* f ,f 1 *' ' (A ns: o = 1 1 .4 9 )

f 26. Calculate Standard Deviation from the following data :


Mid points 1 2 3 4 5 6 7 8 9
Frequency : 2 60 101 152 205 155 79 40 1

1 vl.C.W.A)

If (A ns: <j = 1.57)

27. Calculate the mean and the standard deviation of the


quantity of grain produced for the following data :
■ ,n
l®1 No. of kilograms No. of plots
of grain produced
2 0 0 — 299 5
3 0 0 — 399 9
4 0 0 — 499 15
5 0 0 — 599 17
6 0 0 — 699 22
7 0 0 - 799 17
8 0 0 — 899 7
9 0 0 — 999 6
1 ,0 0 0 — 1,099 2
(B.Com. Gauhati)
v'v;.',' ■ ■ c ,y" (Ans : X = 6 1 2.5
i f . ■>■■ ■■•'JJ $ 'fif:. : t4'|‘! 1 t *'■. •
0 = 188.5)
i:; 'll. .... .*
28. From the following data calculate Mean and Standard
Deviation :
110

Age under 10 20 30 40 50 60 70 80
No. of perrons
dying 15 30 53 75 1 00 1 10 115 125
(B.Com. Kerala)
01, ( Ans : X : 35.16
1j : 19.76)

calculate Mean, Median. Mode


and Standard Deviation.

Salary ' 100-1 1C 120-1 130 130-140 140-150


No. employed : 12 34 72 120 156
Salary : 150-16C 170-1180 180-190
No. employed : 112 82 36 16

.26; M„ = 144.50; <y = 17.34>

29 (a) From a central frequency distribution consisting of 18


observations the X and the a were found to be 7 and 4 respectively.
Bui bn Comparing with the original data it was found that a figure
12 was miscopied as 21 in calculations. Calculate the correct X
and <r.
(Ans : Correct X “ 6.5; Correct or —2.5)

30. From the data given below state which series is more
variable :
Variable Series A. Series B.

1 0 -2 0 10 18
2 0 -3 0 18 22
3 0 -4 0 32 40
4 0 -5 0 40 32
5 0 -6 0 22 18
6 0 -7 0 18 10
. 4fy ;'V': (C.A.)
(Ans : C.V. (A) - 3 3 .3 4
W p;| X C.V. (B) = 37.2
Series B is more variable compared to A)
111
31. Calculate the Standard Deviation from the following
data :
Temperature 'C' No. of Days
_ 4 0 to — 30 10
— 30 to — 20 28
— 20 to — 10 30
— 10 to 0 42
0 to 10 65
10 to 20 180
20 to 30 10
(B. Com. Nagpur)
{Ans: 0 = 14.73)

3 2 . An analysis of the monthly wages paid to workers in


the firms A and B belonging to the same industry gives the follow-
ing results.
Firm A Firm B
No. of workers 500 600
Average monthly wage (Rs) 180 175
Variance of distribution of
wages (Rs.) 81 100

(i) Which firm pays a larger wage Bill ?

(ii) In which firm is there greater variability in individual


wages.
(iii) Find the combined mean wage of the two firms taken
together.

(Arts : (i) Firm B pays larger wage bill.


(ii) There is greater variability in case of firm B in individual
wages.
(iii) X l2 - 177.27 )

33. Mean and standard deviations of two distributions o


100 and 150 items are 50, 5 and 40, 6 respectively. Find the mean
and standard deviations of all the 250 items taken together.
(I.C .W .A .)
(Ans Xia •= 44
<Jia = 7.46)
f
112

34. A test to compare the life span of two makes of bulbs


yinlds the following :
Make I Make II
Average -610 hrs 640 hrs
Standard deviation 40 48
N 36 36
Compare variability and find which make is better.
(B.Com. Osmania, May 1983)

(Ans : C V. of Make 1 - 6.56; C.V. of Make II — 7.5.


Make is better because it is more consistent than Make II)
35. The average height of a group of 100 selected students
r- 168.6 cm. with the coefficient of variation of 3.2. What is the
sinndard deviation ?
(B.Com. Osmania May 1983)
(Ans o ■« 5 4)

36. You are supplied the following data about heights of


boys and girls in a college ;
Boys Girls
Number 3 ,3 7 2 4,538
Average height 68" 61"
Variance 296 456
You are requested to find ;
(a) Combined average of heights of boys and girls.
(b) Coefficient of variation for each group.
(B.Com. Osmanra Man 1983)
(Ans : (a) X u ■= 63.98
(b) C.V. (Boys) = 25.29
C.V. (Girls) = 35)

37. Calculate the standard deviation from the following data:

Marks in Cost Accounting : 0 -1 0 1 0 -2 0 2 0 -3 0 3 0 -4 0


No. of students : 5 7 14 12

Marks in Cost Accounting : 4 0 -5 0 5 0 -6 0 6 0 -7 0


No. of students : 9 6 2
(B.Com., Osmania May 1984)
(Ans : a = 15.57)
jr

( 8) 113

3 8 . Coefficient of variation of two series are 90% and 7 6 |


and their standard deviations 18 and 15 respectively. Find theif
means.
(Ans : X (Series I) = 20; X (Series II) —20)

39. The following is the record of number of bricks laid each


day for 10 days by two brick-layers A and B.
Calculate the coefficient of variation, in each case.
A : 700 675 725 625 650 7C0 650 700 600
B : 550 600 575 550 650 600 550 525 625
A : 650
B : 600
(Ans : C .V .(A )= 5 .5 7 ; C.V. (B )= 6 .3 8 )

40. Weights of a group of individuals are given below In


kilogram, Find out the standard deviation and its coefficient :
Weights (in kg) : Above 0 10 20 30 40
No. of persons : 100 88 55 25 10
(Ans : o = 1 1 4 5 ; Coefficient of S.D. = 0.50)
‘h ' • -:»• «; -*..i;iavA W ^
4 t . A sample of 50 Cars each of two makes X and Y is taken
and average running life in years is recorded :
. ' O il# - ? ' O jf ' I 't i * ' > f';-
Life Number of Cars
(No. of years) Make X Make Y
0- 5 8 6
5 -1 0 12 10
1 0 -1 5 17 20
1 5 -2 0 10 12
2 0 -2 5 3 2

(a) Which of the two makes gives higher average life ?

(b) Which of the tw o makes shows more consistency in


performance ?
(A ns: ( a ) M a k e -Y (B. Com., Osmania, Nov. 1985)
(b ) M ak e -Y )
I 'V f ’ i t V - -f i f ft- I J .3 . iv J -rB

|
i
f
t
/
CHAPTER IV

Skewness

The term skewness refers to lack of symmetry, i.e., when a


luiribution is not symmetrical (or is asymmetrical) it is known as
. ni owed distribution. In other words skewness describes the shape
-i it distribution.
. ' 1;'i i
Measures o f Skewness : Measures of skewness informs as
ilia direction and extent of asymmetry in a series, and allowS us to
. urftpare two or more distributions, th ey may either be absolute or
mliiti ve.

There are two important measures of skewness, namely.


I, The First Measure of skewness.
II, The Second Measure of skewness.

I. The First Measure of skewness : Several formulas are ir»


«ommori use. The important one is given by karl pearson

(a) Absolute S K =M ean — Mode


Where SK=Skewness.

When mode is ill-defined. The following formula w ill be used.

Absolute S K = 3 (Mean— Median)

(b) Relative Measure of skewness or


„ ,, , . . Mean— Mode
Coefficient of skewness (j) = ------p-— -j—-r-K— r-r.-----------
Standard Deviation

or Karl Pearson's coefficient of

a —z
skewness (or SKP )
&
115
Where a=M ean. z=M o d e. <T=Standarcl deviation

When Mode is ill-defined

Coefficient of skewness - 3 —Median)


Standard deviation
II. The Second Measure of skewness :

The one formula in use is given by Bowley,


(a) Absolute SK = Qg-J-Qi— 2 Median

(b) Coefficient of skewness (or SKB)


_ Qg + Q ,— 2 Median
Qg---Q x

SKb = Bowley's coefficient of skewness

Bowley's measure of skewness is based on quartiles.

Illustration 1. From the following date find out the la


Pearson's Coefficient of skewness.

Measurement, 10 11 12 13 14 15
Frequency : 2 4 10 8 5 1

Solution ! Calculation of Coefficient of Skewness

Measurement (X -1 2 )
X f d fd fd*

10 2 —2 —4 8
11 4 —1 —4 4
12 10 0 0 0
13 8 + 1 +8 8
14 5 + 2 + 10 20
15 1 + 3 +3 9

N = 30 ^.fd = 13 == 49
116

Mean = A + ~ - i >1 = 12, £ /c /= 13, N = 30

= 12 + = 1 2 + 0 .4 3 = 12.43

Standard deviation :

/lE fd *
° - V N V N )

- . / — ------ f - 4 5 - V = V 1 .6 3 — 0.19 = y /l .4 4 = 1.2


V 30 V 30 y v
Mode : Since the maximum frequency is 10, the mode cories-
ponds to this value, i.e., 12,
Mean— Mode 12.43— 12
Coefficient of skewness— ---------- —------- = -------- ~--------

0.43
0.36
1.2
Illustration 2 . Calculate the coefficient of skewness for the
following distribution.
Class-interval 0 - '10 1 0 -2 0 2 0 -3 0 3 0 -4 0 4 0 -5 0 | 5 0 -6 0
f 3 5 7 10 12 15
Class-interval : 6 0 -•70 7 0 -8 0 8 0 -9 0 9 0 -1 0 0
1 : 12 6 2 8

Solution : Calculation of Coefficient of Skewness

m.p. (m -5 5 )/ 10
Class-interval f m dx fdx fdx2

0 — 10 3 5 —5 — 15 75
10— 20 5 15 —4 — 20 SO
20 — 30 7 25 —3 — 21 63
3 0 — 40 10 35 —2 — 20 40
4 0 — 50 12 45 —1 — 12 12
50— 60 15 55 0 0 0
6 0 — 70 12 65 + 1 + 12 12
70— 80 6 75 + 2 + 12 24
80 — 90 2 85 + 3 + 6 18
90 — 100 8 -05 + 4 + 32 128

N = 80 = = — 26 S fd x * = 452
tr—
o

>4 = 55i, C = = 80.


X = A + — c == 03
x fjdgx- KX ^ 5 5 + ^ X 1 0 = 5 5 - 3 . 2 5 =51.781
-r "ao
Mode: By inspection Mode lies in the class 5 0 -6 0

Mod, = L + x,0=50+ - f x . O =55

Standard Deviation:
V 5.J65 0.11 X 10
O' - 4 fdx2 - f *-fd x \
ff . = v 5.54 X TO

• = 2.35 X 10 = 23.5
0- . /~ Z z M*\ x C o -e ffic ie n t of Skewness = Mean - Mode
JO/
51.75 - 55 -3.25
V

Illustration 3. Find Bowley's coefficient of skewness fortha


following data :
Wages (in Rs.) : 20 21 22
25 26 27 28 23 24
No. of workmen : 8 10
20 25 15 11 9 5 16
Solution : Calculation of Bowley's Coefficient of Skewness

Wages No. of workmen


X f c f.

20 8 8
21 10 18
22 11 ■ :! 29
23 16 '■rjy “'■!■,1 45
24 20 65
25 25 8v . 90
26 15 105
27 9 114
28 5 119

Q 3 4 -Q 1 — 2 Median

= Size of ^ t-^ th item = —- 9. - - - = 3 0 th item


4 4

Hence Q x = 23 Q s = Size of — th itenr


3X120
“ — 4 — 7 ~ 90th ,t0* Hence Q3 = 25
118

M&dian = Size of ite m = - 1 = 60th item

Hence Median = 24
2 5 + 2 3 -2 (2 4 ) _ 4 8 -4 8 _
- 25 — 23 2

Illustration 4. Calculate the coefficient of skewness based


xA the quart! les :
Variable : 1 0 -2 0 2 0 -3 0 3 0 -4 0 4 0 -5 0 5 0 -6 0 6 0 -7 0
frequency : 5 9 14 20 25 15

Vbriable : 7 0 -8 0 8 0 -9 0
frequency : 8 4

Solution
Calculation of Bowley's Coefficient of Skewness

Variable f c.f.

1 0 -2 0 5 5
2 0 -3 0 9 14
3 0 -4 0 14 28
4 0 -5 0 20 48
5 0 -6 0 25 73
6 0 -7 0 15 88
7 0 -8 0 8 96
8 0 -9 0 4 100

qx=;Sizeof -^ th item = = 25th item ((+ lies in 30— 40)

3 N 3X100
3s=S ize of th item = ----- ^-----= 7 5 th item (Q3 lies in 6 0 — 70)

N 100
Median=Si7e of -^-th item = - y = 50th item (M lies in 5 0 — 60)

Q i= ^ + qQ' c)
'l

= 30 + ^ ° I= ? -0 ( 2 5 - 1 4 ) = 3 0 + X 11 = 3 0 + 7 .8 6 = 3 7 .8 6
14
119

/ _l —__—1(q ,
tl+ h
■JA CA 4A
= 60 + n r — ( 7 5 - 7 3 ) = 6 0 + 4 ^ - X 2 = 6 0 + 1 .3 3 = 6 1 .3 3
10 10

M = Z.1+ ^ ~( Ll (m—c)

= 50 + 6° ~ ! ° (50— 4 8 ) = 5 0 + 4 j - X 2 = 5 0 + 0 .8 - 50 8
zo zb
Q 3+ Q x— 2 Median
SK
Q3— Q j
6 1 .3 3 + 3 7 .8 6 — 2 (50.8)
6 1 .3 3 — 37.86

9 9.19— 101.6 — 2.41


-0.103
23.47 23.47

Im p o rtan t Illustrations
lilustation 5. For a moderately skewed distribution Mean =
160, Mode = 157 and standard deviation = 40. Find (i) Coeffi­
cient of variation (ii) Pearson's coefficient of skewness and (iii)
Median.
Solution :
40
(i) C.V. = X 100 = X 1 0 0 = 25.
160
Mean—Mode 160— 157
(ii) Pearson's coefficient of S K = = 0.08
O 40
(ifi) Median : With the help of the following formula

we can find out the value of median.


Mode = 3 Median — 2 Mean

157 = 3 Median — 2 (160)

1 5 7 - 3 Median — 320

— 3 Median - — 3 2 0 — 157
477
Median = 159
120

Illustration 6. Tha Karl paarsbn's coafficiant of s’o w ia s s of


a distributioms 0.32. Its a = 6.5 and the X = 2 9 .6 . Find tha value
of Mode.
i •••- • ' ■ '• ' ■ !

Solution :
_ tc . , i Mean—Mode
Coefficient of skewness = --------- ---------
a
0 32 _ 2 9 .6 —Mode
6.5
Mode - 29.6— 0 .3 2 x 6 .5
— 29 6 —2.08
= 27.52

List of■ Formula


1. Absolute Skewness : '
(a) Absolute Skewness = Mean — Mode (Karl pearson's)
When mode Is ill-defined
Absolute skewness = 3 (Mean— Median)

(b) Absolute Skewnass=Q3-J-Q1— 2Median (Bowley's)


2. Relative Skewness :
„ , , . . . Mean— Mode
(a) Karl pearson s coefficient of skewness — --------- —--------

When mode is ill-defind


3 (Mean—Median)
Coefficient of skewness = * -----5---------------------- —
a

(b) Bowley'S coefficient of skewness-* Q * — 2 Median


U 3— U i
121

PROBLEMS

1. Calculate the pearson's coefficient of skewness from the


given data :

x : 14.5 15.5 16.5 17.5 18.5 19.5 20.5 21.5


f : 35 40 48 100 125 87 43 22

(Ans : Coefficient of SK ■
------- 0 .2 4 ;

2. Calculate any measure of skewness from the foil awing


data :

x : 0 1 2 3 4 5 6 7
f : 12 27 29 19 8 4 1 0

(Ans : Coefficient of SK = 0)

Hint : We should prefer karl pearson's coefficient of ske­


wness because it is considered to be the best measure for calcu lating
skewness

3. From the data given below, calculate karl pearso


coefficient of skewness.

W eekl/w ag es No. of Weekly wages No. of


(in Rs.) persons (in Rs.) persons

70 — 80 12 110— 120 50
8 0 — 90 18 120— 130 45
9 0 — 100 35 1 3 0 — 140 20
100— 110 42 140— 150 8

(Ans : Coefficient of SK = -- 0.33) (B.Com. Osmania)

4. Calculate the coefficient of skewness based on mean


mode and standard deviation.

x : 0-5 5-10 10-15 15-20 20-25 2 5 -3 0


f : 3 7 9 15 10 6

(Ans : Coefficient of SK = -- 0 .18) (B.Com. Bombay)


122

5. Calculate Bowley's coefficient of skewness from the


following data :

Mid-value Frequency

75 35
100 40
125 48
150 100
175 125
200 80
225 50
250 22

(Ans - 0 .0 3 ) (B. Com., Osmania Nov. 1985)

6. Calculate Karl Pearson's coefficient of skewness from


the following data :

Marks No. of students Marks No. of students

Above 0 150 Above 50 70


„ 10 1 40 60 30
20 100 70 14
„ 30 8C 80 0
40 80
(B.Com., Punjab)

(Ans : Coefficient of S K = — 0.75)

H in t: It is a bi-modal series.

7, Calculate Karl Pe,arson's coefficient of skewness from


the data given below :

Mid-value : 10 20 30 40 50 60
frequency : 8 12 20 10 7 3

(Ans : Coefficient of SK = 0 .1 0 ) (B.Com., Bangalore)


123

8 . Calculate the coefficient of skewness based on M


and Mode from the following distribution :

x: 0 -1 0 1 0 -2 0 2 0 -3 0 3 0 -4 0 4 0 -5 0 5 0 -6 0 6 0 -7 0
f: 6 12 22 48 56 32 18
x ■ 7 0 -8 0
f: 6
yh 4';

(Ans : Coefficient of SK = — 0 .0 5 2 )

9. Calculate coefficient of skewness from the following


A
G *

Weight No. of persons Weight No. of persons


7 0 — 80 8 110— 120 30
8 0 — 90 to 120— 130 40
9 0 — 100 15 130— 140 5
1 6 0 —110 25 1 4 0 — 150 3
(Ans : Coefficient of SK = — 0 .1 6 )
Hint : Prepare a grouping and analysis Table tor ascertain­
ing mode.

10. Calculate the coefficent of skewness with the help of


the following data :
Age Below : 10 20 30 40 50 60
No. of persons : 15 25 40 50 55 60
(B.Com., Osmania, May 1983)
(A n s: C o e ffic ie n t o f SK = - 0 .0 5 )

11. Calculate the Pearson's coefficient of skewness from


the following -
Wages (Rs ; : 0 -1 0 1 0 -2 0 2 0 -3 0 3 0 -4 0 4 0 -5 0
No. of workers : 15 20 30 25 10
(B.Com., Osmania.Nov., 1983)
(A n s: C o ffic ie n t o f SK “ - 0 .1 9 )

12. Mean = 120, M o d e = 1 2 3 and Karl Pearson's coeim,iom


of skewness = — 0 .3 . find the coefficient of variation.
C - 0 I* ,v .(I'fv’ : ' ,lkV:j •• !v |
(B.Com , Osmania, May 1984)
(Ans : C.V. = 8 .3 3 )
124

13. In a certain distribution, the following results warn


obtained :

Mean = 4 5 , Median = 4 8 , e f f i c i e n t of SK= — 0.4


Calculate the value of standard deviation.

(Ans ■ a - 22.5).

14. The Karl Pearson's coefficient of skewness of a distribu­


tion is 0 .3 2 . Its standard deviation is 6.5 and the mean is 29 .6.
Find the mode.
(Ans : Mode =■ 2 7 .5 2 )

15. In a distribution M ean- 6 5 , Median = 70 and coefficient


of skewness is — 0.6. Find (i) Mode (ii) Coefficient of variation.

(A ns: (i) Mode = 80; (ii) C.V. = 38.46>

16. From a moderately skewed distribution of retail prices for


men's shoes, it is found that the mean price is Rs. 20 and the
median price is Rs. 1 9 . If the coefficient of variation is 20%; find
the Pearsonian coefficient of skewness of the distribution.

(Ans : Coefficient of skewness — o. 75)

Hint : Calculate coefficient of variation, to find out


standard deviation.
=" j!« 4 0 4; ;« fl . (Jf p | in' ■ i I r i; | 1 <}
17. Karl Pearson's coefficient of skewness of a distribution
is 0.4. Its standrad cleviation is 8 and mean 3 0 . Find the Made and
Median of the distribution.
(Ans : Mode *» 26.8)
(Median = 28.9)

18. For a moderately skew date, the arithmetic mean is 200,


the coefficient of variation is 8 and Karl Pearson's coefficient of
skewness is 0.3. Find the mode and median,
■./*. "*i ’u c i 1 V*■ h.-:. ' A
(A ns. M ode«-195.2
M edian=19 8.4)
125

H in t : To determine the standard deviation, we have to apply


the following formula :

C.V. = f-X 100

(ii) The value of median can be determind from the follow­


ing formula :
M ode=3 Median— 2 Mean.
19. Pearson's coefficient of skewness of a distribution is
0 .6 4 . Its mean is 82 and mode 50. Find the standard deviation
of the distribution.
(Ans t a = 50)
20. Compute pearson's coefficient of skewnes :

Variable : 2 1 -2 3 2 4 -2 6 2 7 -2 9 3 0 -3 2 3 3 -3 5 3 6 -3 8
Frequency : 17 193 399 194 27 10
(Ans: Coefficient of SK = 0.07)
21. Find Bowley's coefficient of skewness for the following
frequency distribution.

Income (Rs) : 100 150 80 200 250 180


No, of persons : 24 26 16 20 6 30
(Ans : Coefficient of SK = — 0.25)
22. Calculate coefficient of skewness based on quarti les from
the following data :

Size : 1 0 -2 0 2 0 -3 0 3 0 -4 0 4 0 -5 0 5 0 -6 0 6 0 -7 0
Frequency 6 10 18 30 12 10
Size : 7 0 -8 0 8 0 -9 0
Frequency : 6 2
(B.Com. Osmania)
(Ans : Coefficient of Skewness=:0.04)
23. Find a suitable measure of skewness from the following
distribution :

Annual Sales (Rs. '000) 0-20 20-50 50-100 100-250


No. of firms 20 50 69 30
Annual Sales (Ps. '000) 250-500 500-1000
No. of firms 25 19
(Ans : Bowley’s coefficient of S K = 0.55)
126

Hint : Since class intervals are of unequal width, Bowley's


measure of skewness w ill be more appropriate.
24. Find the appropriate measure of skewness from the
following data :
Age (Years) No. of employees Age (Years) No. of employees
Below— 20 13 35-40 112
2 0 — 25 29 40-45 94
2 5 — 30 46 45-50 45
3 0 —35 60 50 and above 21
(Ans : Bowley's coefficient of S K = — 0.104)
H in t : Since the distribution has open - end classes, Bowley's
coefficient of skewness w ill be more appropriate meassure to
compute.
25. From the information given below, calculate Karl pear-
son's coefficient of skewness and also quartile coefficient of
skewness :
Measure Place A Place B
Mean 150 140
Median 142 155
Standard Deviation 30 55
Third Quartile 195 260
First Quartile 62 80
(B.Com. Osmania)
A B
(A n s: (i) Karl pearson's coefficient of SK : 0.8 — 0.82
(ii) Quartile coefficient of SK : — 0.20 + 0 .1 7
26. Wages earnad by tw o groups of workers during a week
are as follows :
Weekly wages No. of workers
group - A group -
10— 20 16 14
2 0 —30 18 20
30— 40 25 24
40— 50 30 28
50— 60 20 20
6 0 — 70 14 16
70— 80 6 4
More than— 80 2 1
C a l c u l a t e t h e Q u a r t i l e D e v i a t i o n and the
C o e f f i c i e n t o f s ke wn es s (B.Com.Osmania)
(Ans : Group-A Q.D. = 12.65; Skewness = — 0.0 2 .
Group-B Q.D. = 12.85; Skewness = — 0.02.)
127

///(I/ tribution the inter-quartile range >s


03 efficient of skewness calculated by
O
te two quartiles.
»
•< (B.Com., Osrri3nia)
range—Q3—Q,.

28 In a frequency distribution, the coefficient of skewness


I immhI upon quartiles is 0.6. If the sum of the upper and the lower
quartiles is 100 and the median is 38, find the value of the upper
quartile.
(Ans : Q3= 7 0 )

2g, Find the coefficient of variation of a frequency distribu­


tion, given that its mean is 100. mode is 102 and Karl pearson's
coefficient of skewness is — 0.4.
(B. Com., Osmania)
(Ans ; C .V .= 5 )
■. | i,f , > . *■'* 1' , . 3||
30. The mean, median and the coefficient of variation cf
the weekly wages of a group of workers are Rs. 45, Rs 43 and
Rs. 40. Find the (i) Mode, (ii) Variance, and (iii) Coefficient of
skewness, for the distribution of wages.
(Ans ; (i) 39 (ii) 3 2 4 (iii) 0.33>

31. A frequency distribution gives the following results.


(i) Karl Pearson's coefficient of skewness=0.5.
(ii) Coefficient of variation = 0 .5 .
(iii) Standard Deviation=2.
Find the (i) Mean and (ii) Mode of the distribution.
(Ans : (i) 400 (ii) 399)
y
32. Given Q ,= 1 8 . Q3= 2 5 . M ode=21, Mean: : 1 8 ,
find the coefficient of skewness.
(Ans: 0.71)
Hint : To calculate median, we have to apply the following
formula.
Mode= 3 Median— 2Mean.
CHAPTER V

Correlation

Correlation is a statistical technique used for analysing the


behaviour of two or more variables, According to Ya Lun Chou
'Correlation analysis attempts to determine the degree of relation­
ship between variables"

The following are the various methods Of ascertaining whether


two variables are correlated or not

1. Karl pearson's coefficient of correlation


2. Rank Method
3. Concurrent Deviation Method
4. Scatter Diagram Method
5. Graphic Method

Of these, the first three are the mathematical methods where


as the last two are based on the knowledge of digrams and graphs.

1. Karl pearson's C o e fficie nt of C o rre la tio n This method is


most widely used in practice. It is denoted by the symbol r.

There are two ways of calculating pearson's coefficient of correlation

(i) When deviations are taken from actual mean :


Txy
r ~ N o * <Ty

Where r = The correlation coefficient

x - (X -X ); Y - (Y— Y)
2 x y =• Sum of the product of deviations in

X and Y Series calculated with reference to their arithmetic


means.
<9 ) 129

<jx = Standard deviation or series X

<Ty = Standard deviation of series Y

J\l = Number of pairs of observations.

An alternate form of the above formula is :

“ \ / r x 9X 2 y *

Where x = (X—X)

Y = (Y— Y )

The following steps are involved, when this formula is applied

(a) Calculate means of the tw o series, X end Y

<b) Take deviations of the tw o series from their respective


means and denote these deviations by x and y

(c) Square the deviations and obtain the sum of the res­
pective squares of deviations, i. e., 2 x * and 2 y 9

(d) M ultiply the deviations of X and / series and obtain the


total, i. e., 2 x y

(e) Substitute the values of 2"xy, ^ x * and 2 y 9 in the


formula.

The value of the coefficient of correlation always lie between


± 1. When r = -J- 1. it means there is perfect positive correlation
between the variables. When r = — 1, it means there is perfect
negative correlation betweeh the variables. When r*= 0, it means,
there isnorelationship between the tw o variables.
130

PROBABLE €RROR

The probable error of the coefficient of correlation helps in


interpreting its value. The probable error is obtained by the follow ­
ing formula :
1 — r2
P.E. r = 0.6 7 4 5 X ■ _ -■
\/ N

Where r = the coefficient of correlation.


N — the number of pairs of items.

In regard to the probable error, the interpretation is done as


follows :

(a) If the value of r is less than the probable error there is


no evidence of correlation, r. e , the value of r is not at
all significant.

(b) If the value of r is more than six times the probable error
the coefficient of correlation is practically certain, i.e.,
the value of r is significant.

Illustration 1 ■ Calculate the coefficient of correlation from,


the following data :
X : 3 4 6 7 10
Y : 9 11 14 15 16

Solution:
Calculation of coefficient of correlation

X (X -X ) X = 6 Y (Y -Y ) 'Y = 13
X X2 y ya xy

3 —3 9 9 —4 16 + 12
4 —2 4 11 -2 4 + 4
6 0 0 14 + 1 6
7 + 1 2 15 + 2 4 + 2
10 +4 16 16 +3 9 + 12

2X=30 2 x = 0 2 x a= 3 0 2 V = 65 2 y = 0 2 y ?= 3 4 S * y = 3 0
131

2 xy

x = (X— X) y = (Y — Y)
2 X 2 /
1
'X N Y~ n r

30 65
= 6 13

2 x y - 30, S xa 31, 2 y 2 = 34

_ 30 30 30
+ 0 .9 4
r " v/3 0 x 3 4 “ V 1020 3 1 .9 4

(ii) When Deviations are taken from an Asssumed Mean :


\ i- f-f/& .-ii.. fijj
When actual means are in fraction the use of the above formate
becomes time consuming. In that case the assumed mean
should be used. Under this method, the following steps must be
followed :

(a) Some convenient values (usually values near the ac*E»J


means) are taken as assumed means in series X and Y.

(b) Take the deviations of X series from an assumed rasan


and denote these deviations by and obtain the total,
i.e..

(c) Take the deviations of / series from an assumed maara


and denote these deviations by r/y and obtain the fatal
i.e., 2 cfy

(d) The deviations dy and dy are squared up and their totals


2 </x 2 and 2 cfya are obtained.
f : '•* . . . t, .... . 1 ... f'; . ‘ i,: .... , , . , | .

(e) Multiply dy with dy and obtain the total ’Z d y dy

(f) Substitute the value of ~2.dy dy , 2cfx. 2c/y , Z d y * am#


"Zdy 2 in the formula given below to obtain the coefficient
of correlation.
132

2 d xX 2</y
2 d xd ,
N
P-—
/ ( 2 d x)* (-2d,)*
V 3 /,* — N V xdy N
Illustration 2. Find the coefficient of correlation from the
following data :
X: 46 54 56 56 58 60 62
Y: 36 40 44 54 42 58 54
Solution:
Calculation of Coefficient of Correlation

X (X-56) Y (y -4 5 )
dx dx* dy d,* dxdy

46 — 10 100 36 — 9 81 +90
54 — 2 4 40 — 5 25 + 10
56 0 0 44 — 1 1 0
56 0 0 54 + 9 81 0
5.8 + 2 4 42 — 3 9 — 6
60 + 4 16 58 + 13 169 + 52
62 + 6 36 54 + 9 81 + 54

447 + 200
*

0 160 + 13
II

2 d xdy-
N

(.2 d x)*
2d* -
' N
0X13
200 —
7

74 4 7 - (1 3 )a
V 7
_________ 200_______
= V/ T60" V 422.86
200 _ 200
~ 1 2 .6 5 x 2 0 .5 6 ~ 2 6 0 . 13 “ + ° - 769
133

Illustration 3. Calculate the probable error. If r = 0.8 and


N - 7.

Solution:
1— ra
P.E. r = 0 .6 7 4 5 X
v/7v-

1— (0 .8 )2
— 0 ,6 7 4 5 X

1— 0.64
= 0.6 7 4 5 X
2,65

= 0.6 7 4 5 X - 2 ' ^ - = 0-6745 X 0.14


2 65
= 0.0 9 . ♦

2. Rank Method : The method of ascertaining the coeffic


of correlation by ranks was devised by Charles Spearman. The
formula for coefficient of rank correlation is.

6SD2
A/3—

re = Spearman's rank correlation.


D = Difference of ranks.
N = Number of pairs of observations

Generally we have two types of problems in this method :

(i) Where ranks are given

(li) Where ranks are not given


Ah
(i) W h ere ranks are given : The following procedure must
be adopted for computing rank correlation :

(a) Take the differences of the tw o ranks, i. e., (/?x—--^a) and


denote these differences by D.
(b) Square these differences and obtain the total S D *

(c) Apply the formula given above.


134

frustration 4. Two judges X and Y assign the following ranks


to five entries in a beauty contest. Calculate rank correlation
coefficient.

A B O D E
X 1 2 3 5 4
V 2 1 4 3 5

Solution i5
*
Calculation of Rank Correlation Coefficient

X y
*2 D

1 2 —1 1
2 1 + 1 1
3 4 —1 1
5 3 + 2 4
4 5 —1 1
2 D a= 8

= W = N ? D * = 8 ,N = 5

6X8 48 - 1 48 0 .4 = 0 .6
5 3— 5 “ 1 125— 5 ~ 1 120

(ir) W h e re ranks are not given : When we are given the


actual data and not the ranks, it will be necessary to assign the
ranks. Ranks can be assigned by taking either the lowest value as 1
or the highest value as 1. But whether we start with the lowest
value or the highest value we must follow the same method in case
of both the variables.

Illustration 5. Calculate rank correlation coefficient from the


following data :

X 99 47 68 57 76 61 30 44 82 58
y 75 63 72 42 74 41 20 64 68 67
Illustration .7
he given data :
N - 7. ■3
CD
elation Coefficient
Solution “ O
* ? -

I1 I i D D2
CD (O
0 0
o /r 1 1
a u> 1 1
-1 1
/4 2 1 1
5 41 9 -4 16
30 10 20 10 0 0
44 9 64 6 3 9
82 2 68 4 -2 4
58 6 67 5 1 1
2 D *= 3 4

6 20*
tm — 1-
A/3—/V
6X34 204 204
1 = 1— 1— 0 .2 1 = 0 .7 9
io»—io: -
1 0 0 0 — 10 990

Equal Ranks Where equal ranks are assigned to some


entries and adjustment in the above formula is made.
1
The adjustment consists of adding (/n*— m) to the value

of 2 D * , where rn stands for the number of items whose ranks are


common. If there are more than one such group of items with
common rank, this value is added as many times the number of such
groups The formula can thus be written.

, _ , 6[ 2D*+ TT </"5- ' n)+ fT (n,s- /n>+ •••}


_ _ _

Illustration 6. From the following data calculate the coeffi*


cient of rank correlation between X and Y :
X 20 25 60 45 80 25 55 65 25 75
Y 45 50 55 50 60 70 72 78 80 63
Solution :
Calculation of C oefficient of Rank Correlation

X fli Y (/?}
D D1
2 0 1 45 1 0 0 . 0 0

25 3 50 2.5 0.5 0.25


60 7 55 4 3 9.00
45 5 50 2.5 2.5 6.25
80 1 0 60 5 5 2 5 .0 0
25 3 70 7 —4 16 0 0
55 6 72 8 — 2 4 00
65 8 78 9 —1 1 . 0 0

25 3 80 1 0 —7 49.00
75 9 63 6 + 3 9.00

2 D 2 = 1 19.5

In X serie s three are ranked equal at

2 nd place, so they are given the ta v e rage rank 3

In Y series tw o are ranked equal at 2 nd place, so they are


24-3
given the rank —— = 2*5
JL

6 / S D 24- - A - ( m s— m ) + (m 3— m) \
re = 1 — 1 12 12 J
N s—N
The item 25 is repeated 3 tim es in series X, So m = 3 ir>
series / th e item 5 0 repeated 2 tim es so m = 2.

:; < 1 1 9 .5 + + (3 » -3 ) + + ( 2 .- 2 )
_____

6 [ 1 1 9 .5 + 2 + 0 .5]
= 1
990
6X122 732
1 — 0 .7 4 = 0 .2 6
990 990
137

3, Concurrent Deviation Method : Under this method we havn


to find out the direction of change of X variable and Y variable. To
compute correlation, the following steps are required :

(a) First, find out the direction of change of X variable, i e.


as compard with the first value, whether ihe second value is increas­
ing or decreasing or is constant. If it is mcreasing put a - f sign, if
it is decreasing put a — sign and if it is constant put zero. In the
same manner, as compared to second value find out whether the third
value is increasing, decreasing or constant. Repeat the same process
for other values. Denote this column as Dx,

(b) Similary, find out the direction of change of V' variable


and denote this column as Dy

(c) Multiply Dx w ith Dy and determine the value of C, i. e.,


the number of positive signs

(d) Apply the following formula.

/ 72 c—n \
rc - ± a/ ± ( — 77— j

Where rc = Coefficient of correlation bv the concurrent


deviation method.

c — The number of positive signs.

n = Number of pairs of observations which is one


less than the actual number \ . e . , n = (N— 1 )

Illustration 7. Calculate coefficient of correlation by con­


current Deviation Method from the following data :

X 145 150 165 140 175

Y 170 155 180 210 220


138

Solution :
Calculation of correlation by concurrent Deviation Method.

X Direction of change Y Direction of change D XD ,


Dx Dy

145 170
150 + 155 — —

165 + 180 + +
140 — 210 + —
175 + 220 + +
C-= 2

rc - ± V / ± ( M r ?)

c = 2. n <= 5 — 1 = 4

/ / 2 X 2— 4 ) - \ / o
fc - ± V 4 V "4~= ° '

Important illustrations
Illu s t r a t io n 8 . Ten competitors in a beauty contest are
tanked by three judges in the following order :

1st Judge 1 5 4 8 9 6 10 7 3 2
2nd Judge 4 8 7 6 5 9 10 3 2 1
3rd Judge 6 7 8 1 5 10 9 2 3 4

Use the Rank correlation coefficient to discuss which pair


of judges have the nearest approach to common tastes in beauty.
Calculation of Rank correlation coefficient

Rank by Rank by Rank by


1st Judge 2nd Judge 3rd Judge ( f V R a)a (R ,-R a)* (R,-R,)»
Ri Ra Rj 0* 0» 0a

1 4 6 9 25 4
5 8 7 9 4 1
4 7 8 9 16 1
8 6 1 4 49 25
9 5 5 16 16 0
6 9 10 9 16 1
10 10 9 0 1 1
7 3 2 16 25 1
3 2 3 1 0 1
2 1 4 1 4 9

74 156 44

<a) Rank correlation between the Judgement of 1st and


2nd Judges :
6 FOa
r .= 1 -
N 3—N
6 X 74 444
1— 1—0.45 «= 0.55
103— 10 = 990

<b) Rank correlation between the Judgement of 1st and


3rd Judges :
6£D a
N 3— N
6x156 , 936_ ,
= 1 0.95 = 0.05
10 s— 10 31 990“ 1
'ft i »«;
(c) Rank correlation between the Judgement of 2nd and
3rd Judges :
62 0 *
= 1 N 3—N
6X44 264
1 — 0.27 = 0.73
103— 10 990
140

Since coefficient of correlation is maximum in the judgement


m Hu) 2nd and 3rd judges, we conclude that they have the nearest
tttyttiKich to common tastes in beauty.
' ‘h. 'fy- f "fri !' ’I H ' X x ■ 'i)> ii:

List of Formulae
;V. ■ ■ i; " '% $j , f ;fi, k t :i! 0 'ii'/ '& ? > M ' ■1

1. Karl Pearson's coefficient of correlation.


-

(i) When deviations are tcken from actual mean :

X *y 2xy Sx2
or ; (Tx =
/Vcrx<Ty s/^^'ZY N
/ — *2
x = (X— X ) ; y — (y—-y ) ; (T,
Vor
(ii) When deviations are taken from assumed mean :
S cfx x ~Zdy
S cfx dy —
/V
r — (2»/x)a ('ZdyV
Scfx "2,dy
V N N

2. Rank correlation coefficient by Spearman :


6£D a
rs = 1 (A/3— N)
In case of equal ranks :

2(m*— m) + yL + ... )
's = 1 ( * ° 2+V.
N s—N
'
3. Concurrent Deviation Method :

r c= ±
4. Probable error :

P.E. r = 0.6745
VN
5. covariance _
= Ix_y
N
141

PROBLEMS
1. Find out the coefficient of Correlation between the height
of father and son from the following data :
Heights of father (in inches) : 65, 66, 67, 67, 68, 69. 70, 72
Heights of son (in inches) 67, 68, 65, 68, 72, 72, 69, 71.
(Ans • r — + 0.60) (B.Com., Osmama)

2. Calculate coefficient of correlation between the following


two variables :
x : 45 24 19 34 33 16 18 40 36 53.
y '■ 28 10 1 1 25 20 18 1 1 21 30 18.
(Ans : r = + 0 . 6 0 ) (B,Com.,)

3. Find the coefficient of correlation and probable error from


the following :
X : 10 28 45 60 70 50
y 100 80 50 40 33 45
(Ans : r = — 0.98) (B Com., Osmania, May 1983)

4 Calculate the Karl Pearson's coefficient of correlati


between ages of husbands and wives and comment on the results.
Age of Husband : 20 30 40 50 60 70 80
Age of wife . 14 25 30 32 40 45 65
(Ans . r = + 0 .9 6 ) (B.Com. Osmania, Nov'83)
5. Ten students of B.A. obtained the following percenta
of marks in English in the Internal Assessment Test (x) and Univer­
sity examination (y).
Calculate Karl Pearson's coefficient of correlation from actual Mean
and its probable error.
x : 50 60 75 84 47 52 59 44 33 46
y : 45 52 50 65 40 65 50 60 32 51
(A n s : r = 0 .5 5 ) (B .Com. Osmania, May 1984)
6. Find V 1for the following values :
x : 36 80 50 58 72 60 56 68
V : 35 65 60 39 48 44 48 61
(Ans: +0.63) (M.Com. Osmania)
142

7. Calculate 'r' for the following data :


x: 4 22 8 14 10 12 12 18 10 14
y : 8 38 12 36 16 32 18 28 20 24
(A ns: + 0 .8 5 ) (M.Com. Osmania)

8. Find the coefficient of correlation between the income (x)


and expenditure (y). (Fig in 000's)
X : 20 30 40 60 90 50 70 65
y : 18 27 32 48 70 37 58 53
(Ans: + 0 .9 9 ) (M.Com. Osmania)

9. Calculate the correlation coefficient for the following


data :
x; 12 9 8 10 11 13 7
y : 14 8 6 9 1 1 1 2 3
(Ans: + 0 . 9 4 ) (M.Com. Osmania)

10. Calculate’coefficient of correlation from the following


data :
x: 30 50 60 80 100 110 130
y: 100 200 300 400 500 600 700
Hint : Coefficient of correlation is independent of change
of scale and origin. Hence in order to simplify
calculations let every value of x be divided by 10
and every value of y by 100 and denote these
series by x' and y'. The formula for calculating
correlation coefficient remains the same.
(Ans: + 0 . 9 9 )
11. Given the following values of ten Pairs in the two
series x and y, calculate r.
(1) Sum of the squares of the Series Series
deviations measured from x y
their respective means... 28 96
(2) Sum of the products of the
deviations of the two series
measured from their respective means. 42
Ans : r = + 0.8) (B.Com. Osmania)
143
12. Two judges in a beauty competition rank the 12 ent­
ries as follows :
x: 1 2 3 4 5 6 7 8 9 10 11 12
y : 1 2 9 6 1 0 3 5 4 7 8 2 11 1

What degree of agreement is there between the judges 7


(Ans. rs = -0 .4 5 }

13. Ten competitors in a beauty contest are ranked by


three judges in the following order.
1 St Judge 1 6 5 10 3 2 4 9 7 8
2nd Judge 3 5 8 4 7 10 2 1 6 9
3rd Judge 6 4 9 8 1 2 3 10 5 7
Use the Rank correlation coefficient to determine which
pair of judges has the nearest approach to common tastes
in beauty.
(Ans : correlation coefficients in the judgement of
(i) 1st and 2nd Judges = — 0.21
(li) 2nd and 3rd Judges = — 0.30
(iii) 1st and 3rd Judges = -)- 0.64)
14. Find the rank correlation coefficient for the following
data :
iff
X 48 58 72 62 56 45 39 52 25
Y 63 78 65 70 38 54 60 32 28

(Ans : /■«, = 0.67)

1!j^ Apply rank method to find out correlation between


X and Y from the following data :
X 15 -20 20 20 50
Y 10 12 60 60 IQ
(Ans : rs = 0.8)

16. Calculate rank correlation coefficient from the followina


data :
X 12 18 25 25 50 25
Y 10 12 18 18 15 40
(Ans : rs = 0.54)
144

17. Calculate coefficient of correlation by concurrent Devia­


tion method.
Price (X) 370 384 386 360 345 380 395 404 400
(Imports) Y 23 21 25 20 23 26 24 29 28
Price (X) 380
(Imports) Y 27
(Ans : rc = 0 58)
18. Find out correlation by the concurrent deviation method
Irom the following data :
Supply (Kg) 200 240 300 200 320 340
Price (Rs ) 160 180 180 200 240 364 (Ans : rc =0.45)
19 Coetficient of correlation between two variates X and
Y is 0.4. Their covariance is 9. The variance of X is 16. Find the
standard deviation of Y series.
(Ans : t f y - 5 . 6 3 )
20. If r = 0.6. and /V=64, find out the probable error of
the coefficient of correlation.
(Ans . P.E.r = 0.05)
21. Calculate Pearson's co-efficient of correlation for the
following data :
X: 105 11 1 104 1 12 1 18 98 Vi 6 103 116 112
Y: 62 64 53 60 72 56 68 60 69 65
(Ans: + 0 .8 7 ) (B.Com, Osmania May 1935)
22. A computer while calculating correlation co efficient
between two variables x and y from 25 pairs of observations
obtained the following resu’ts :
N=25; 1 X = 1 2 5 ; £ x 2 =6 5 0 ; % Y = 1 0 0 ; J y2 =460;
X X Y =508.
It was however discovered at the time of checking that two
pahs of observations ( 8 , 1 2 ) and ( 6 , 8 ) were wrongly taken as ( 8 , 6 )
and ( 6 . 14). Calculate the correct correlation coefficient.
(Ans: + 0 .6 6 ) (B.Com., Osmania, Nov. 1 985)
C H A P T E R VI
Regression Analysis
The literal meaning of regression is "moving backward" 1
or Mthe returning to the average value". The term was
first used by Sir Francis Gallon in the later-paft of the
19th century in connection with some studies he made on
estimating the extent to which the stature of the.sons of
tall parents reverts or regresses back to the mean stature
of the population. But today the word regression as used
in statistics has a much wider perspective. Regression
Analysis, in the general sense, means the estimation or
prediction of the unknown value of one variable from the
known value of the other variable. This is a very useful
statistical tool which is used both in natural and social
sciences. In the field of business this statistical tool
is very widely used. Businessmen are interested in predic­
ting future production, consumption, sales, profits etc.,
In fact the success of a businessmen depends on the corre­
ctness of the various estimates that he is required to
make. The regression study which confines itself to a
'study'.of only two variables is known as Simple Regression.
The regression study which studies more than two variables
at a time is called Multiple Regression.
In the simple regression analysis there are two
variables - The variable which is used to predict the
variable of interest is called the independent varibale or
regresser or predictor and the variable we are trying to
predict is called the dependent variable or explained
variable. The independent variable is denoted by X and the
dependent variable by Y. Thus Regression Analysis provides
estimates of values of' the dependent variable from values
of the independent variable. The device used to accomplish
this estimation procedure is the regression line.
Lines of Regression:- Line of regression is the line
which gives the best estimate of one variable for any
given value of the other variable. In case of two
variables X and Y, we shall have two lines of regression,
(1) The regression line of X on Y; and (2) the regression
line of Y on X. The regression line of X on Y gives the
most probable values of X for giveii values of Y. Simi­
larly, the regression line.of Y on X gives us the most
probable values of Y for given values of X.

Regression Equations:- Regression equations are alge­


braic expressions of the regression lines. There are two
regression equations as there are two regression lines.
The regression equation of X on Y is used to describe the
variations in the values of X for given changes- in the
values of Y and similarly, the regression equation of Y on
146
X is used to describe the variation in the values of Y for
given changes in the values of X.
The two regression equations are given b e l o w :-
(i) The regression equation of X on Y is:
X = a + bY
(ii) The regression equation of Y qn X is:
Y = a + bX
In these equations the values of a and b are constant
which determine the positions of the lines of regression.
The parameter 'a' indicates the level of the line of
regression. •
The parameter 'b' determines the slope of the line
i.e., the corresponding change in X in relation to per
unit change in Y or vice versa. The values of a and b are
found by the method of Least Sqaures with the help of
normal equations given below:
In case of Regression equation of X on Y:
X a h bY
£x Na + b £ y

£.XY = atY + b £ Y
In case of Regression equation of Y on X:
Y a - bX
e* Na + b£X

£XY a?LX + b£x


Deviations taken from Means of X and Y: - The above method
of finding out regression equations is very tedious. The
calculations can very much be simplified if instead of
dealing with the actual values of X and Y we take the
deviations of X and Y series from their respective mean
values.
The formula for the calculation of Regression Equa­
tions by this method is:
(i) Regression Equation of X on Y
_ tr x _
X - X = r ----- (Y - Y)
tr y
Where X denotes the actual mean of the X series and
Y the actual mean of Y series, r denotes coefficient of
correlation between X and Y series and trx and try denotes
the standard deviations of X and Y series.
TT*X
r ----- is known as the regression coefficient of
try
of X on Y and is denoted by the symbol bxy. It measures
the change in X corresponding to a unit change in Y. Thus
T

147

c r*
bxy or r ----- value is Ce l
< ry 2
Cy
Instead of finding out the value of r,8"x a n d c y we can
directly find out the value of regression coefficient by
.h
calculating £_xy and £_y and dividing the former by the
latter.

Note: In case when we deal with actual values of X *


and Y variables and not the deviations, bxy is *
obtained as follows: *
*
n £x y - (£X) (£Y) *
bxy = *
2 2 *
n £Y - (LY) *

{i i ) Regression Equation of Y on X
rr y _
Y - Y = r ---- - (X - X)
s- x
<r x
r ----- is the regression coefficient of Y on X
y
It is denoted by byx. It measures the change in Y
corresponding to a unit change in X. Thus byx or
cry
(
£ x

* Note: In case when we deal with actual values of *


X X and Y variables and not the deviations, byx is *
X obtained as follows: . Z ,-V*% X

X X
^0 |i’ll
X N£_XY - (£ x ) (C O *
X byx = --------------- *
vt 2 *
X N£x (£ x ) *

Thus from the above discussions it is very clear that


the following steps must be taken to calculate regression
equations if the means of two series (X and Y) are whole
nu mbers:
148

1) Find out the mean of X series. The formula is


_ £ *
x = ------
N
2) Take the deviations of X series from the mean_ of
X and denote these deviations by x. (x = X - X)
3) Square these deviations and obtain the total,
a
i.e., £ x
4) Find out the mean of Y series. The formula is

N
5) Take the deviations - of Y series from tha_
mean of Y and denote these deviations by y(y= Y-Y)
6) Square these deviations and obtain the total
r
i-e.,£.y
7) Multiply the deviations of x and y series and
obtain the total, i . e . , £ x y .
(Note: The above steps are same as in the case of
correlation)
_ __ ?. %
8) Substitute the values of X, Y, £_xy, £ x and£y
in the above equations.
Now the two regression equations can be written as
follows:
(i) Regression Equation of X on Y

X - X = ----- (Y - Y)
a
L y
(ii) Regression Equation of Y on X
C xy _
Y - Y = — (X - X)
a

Deviations taken from Assumed Means (Short-cut Me t h o d ):-


When the actual means of the two series X and Y are in
fractions, the calculations can he simplified by taking
the deviations from the assumed means. However, the entire
procedure of finding regression equations remains the
same. The two regression equations are:
(i) Regression Equation of X on Y:
cr x _
X - X = r ---- (Y - Y)
^ y
149

<£_dxdy - £ d x X £.dy
CT X N
The value of r ----- = --------------------
tr y 2 a
£dy
(£dy) -
N
Where dx = (X - A ) ; dy = (Y - A)
A = Assumed Mean.
(il) Regression Equation of Y on X:
_ c ry _
Y - Y = r — - (X - X)
5" x

p dxdy - £ dx X £ d y
try N
The values of r ---
S' X i u
£_dx - (£.dx)
N

N o t e :- The above computations are same as in the case of


correlation (Refer to Page 131)

It should be noted that the under root of the product


of two regression coefficients gives us the value of
correlation coefficient symbolically:

r = J bxy X byx

The following are the points worth noting about the


regression coefficients:
(a) Both the regression coefficients cannot exceed one.
(b) Both the regression coefficients will have the same
algebraic signs.
(c) r will have the same sign as that of regression
coefficients.

Illustration.!: From the following data obtain the two


regression equations and calculate the correlation
coefficient:
X : 2 4 6 8- 10 12 14 16 18
Y: 18 16 20 24 22 26 28 32 30

Estimate the value of Y which should correspond on an


average to X = 6.2
S o lu tio n :
CALCULATION OF REGRESSION EQUATIONS AND
CORRELATION COEFFICIENT

X (X - X) X Y (Y - Y)
X X y y xy

2 8 64 18 _ 6 36 + 48
4 - 6 36 16 - 8 64 + 48
6 - 4 16 20 - 4 16 + 16
8 - 2 4 24 0 0 0
10 0 0 22 - 2 4 0
12 + 2 4 26 + 2 4 + 8
14 + 4 16 28 + 4 16 + 16
16 f6 36 32 + 8 64 + 48
18 + 8 64 30 + 6 36 + 48

£_x=90 £ x = Q £x=240 £Y=216 £y= o £y=240 6 xy = 232

(i) Regression Equation of X on Y:


_ c rx _
X - X = r ---- (Y - Y)
Cy

£_X 90 _ £Y 216
x = ---= ---- = 10, Y ----- = ---= 24
N 9 N 9
crx g_xy 232
r ---= ----- = ----- = 0.97
fy % 240
fe y
Substituting the values:
X - 10 = 0.97 (Y - 24)
X - 10 = 0.97Y - 23.28
X = 0.97Y - 23.28 + 10
X = 0.97Y - 13.28
(ii) Regression Equation of Y on X:
rry
151

Y - 24 = 0.97 (X - 10)
Y - 24 = 0.97 X - 9.7
Y = 0.97 X - 9.7 + 24
Y = 0.97 X + 14-3
Y = 0.97 (6.2) + 14.3
Y = 6.01 + 14-3
Y = 20.31
Cc/rrelation Coefficient i.e. ,

r = J bxy x byx

bxy = 0.97; byx = 0.97

Hence 'r = 7 1 ) 7 9 7 x 0.97 = 0.97

Illustration 2. From the following data obtain the


two regression equations.
X: 2 4 6 8
Y: 10 20 25 30
Solution:
Computation of the Regression Equations

Y (X - A) Y (Y - A)
A = 5 a A = 20 r
dx dx dy dy dx dy

o - 3 9 10 - 10 100 30
*+ - 1 1 20 0 0 0
6 + 1 1 25 + 5 25 5
8 + 3 9 30 + 10 100 30

a a
: X=20 £dx=0 £dx=20 0>85 £ d y = 5 £dy =225 £dxdy=65

_ 5 -x _
(i) Regression Equation of X on Y: X - X = r ---- (Y - Y)

£. X 20

N 4

g_Y 85
= ----- = ----- = 21.25
N 4

Assumed Mean 5; Assumed Mean = 20


152

£_ dxdy - £dx X£dy 65 - 0 X 5


ir * 4

try t. /i
£_dy - .( £ d y ) 225

N
65 65 65
0.30
225 -25 225 - 6.25 218..75
4
X - 5 = 0 .3 0 (Y - 2 1.25)
X - 5 = 0.30Y - 6 • 375
X- = 0.30Y - 6.375 + 5
X = 0.30Y - 1.375
( i i ) R e g re ssio n E q u atio n o f Y on X:
try
Y - Y = r ------ (X - X)
tT x
£_dxdy -tdx_X tdy 65 - 0 X 5
<r y N 4 65
= 3-25
<r x 'i '1 - 2 0 - 0 20
£dx - (£dx) 4
N
Y - 2 1 .2 5 = 3-25 (X - 5)
Y = 3.2 5 X - 16.25 + 21.25
Y = 3.2 5 X + 5

*****

PROBLEMS

1. From th e fo llo w in g d a ta o b ta in th e two re g re s s io n


e q u a tio n s :
X: 91 97 121 108 124 67 51 73 H i 57
Y: 71 75 97 69 91 70 39 61 80 47
(Ans: X = 1.36Y - 5 .2 7 ; Y = 0.61 X + 15.1)

2. From th e fo llo w in g d a ta o b ta in th e two r e g r e s s io n


e q u a tio n s and c a l c u l a t e th e c o r r e l a t i o n c o e f f i c i e n t :
X: 1 2 3 4 5 6 7 3 9
Y: 9 8 10 1 2 1 1 13 14 1-6 15

E s tim a te th e v a lu e o f Y which s h o u ld c o rr e s p o n d on an
a v e rag e to X = 6 .2 (B.Cora, B a n g a lo re , 1981)
(Ans:X=0.95 Y -6 .4 ;Y = 0 .9 5 X + 7 .2 5 ;r= 0 .9 5 ;V alu eo f Y=13.14)
3

Y - on equations which may be


Y values and calculate karl
correlation
55 89 98 66
58 64 76 58
(B.Com, Delhi)
'.37 X + 35.27; r = 0.90)
f ind out :
dilations.
.^.-xoient of correlation between the ages
of husbands and ages of wives.
(c) Most likely age of husband when wife's age iq 25
years.
(d) Most likely age of wife when husband's age is 30
years.
Age of Husband
(Years) : 22 23 23 24 26 27 27 28 30 30
Age of wife
(Years) : 18 20 21 20 21 22 23 24 25 26
(Ans: (a) X = 1.1Y + 1.8; Y = 0.8 X + 1.2; (b) r = 0.94
(c) 29.3 (d) 25.2.
5. Calculate the coefficient of regression for the
distribution given below:
X: 8 6 4 7 5
Y: 9 8 5 6 2 (Ans: bxy = 0.4; byx = 1.2)
6. The following are the marks in statistics (X) and
Mathematics (Y) of the students:
X: 56 55 58 58 57 56 60 64 69 57
Y: 68 67 67 70 65 68 70 66 68 66
Calculate the coefficient of ■
correlation and estimate
the marks in Mathematics of a student who secures 62
marks in statistics. (B.Com, Bombay)
(Ans: r = + 0.78; Y = 0.029 X + 65.77; Y = 67.57)
7. Calculate coefficient of correlation and regresion
line of Y on X for the data below:
X: 1 2 3 4 5 6
Y: 9 8 10 12 11 *y 13 (M. Com. Andhra)
(Ans: r = 0.893; Y= 45 X + 9.93)
8. In a correlation study the following values are
obtained:
X Y
Mean 65 67
Standard Deviation 2.5 3-5
Coefficient Correlation 0.8
Find the two regression equations that are associated
with the above values (M.Com, Kashmir)
(Ans: X = 0.57 Y + 26.72; Y = 1.12 X - 5.8)
154

9. Given the following results for the height (X) and


weight (Y) in appropriate units of 1000 students.
X Y
Mean 68 150
Standard Deviation 2.5 20
Coefficient Correlation 0.60
Obtain the equation of the two lines of regression.
Estimate the height of a student A who weighs 200
units and also estimate the weight of the student B
whose height is 60 units.
(Ans: X = 0.075 Y + 56.75; Y = 4-8 X - 176.4;) When
Y = 200, X = 71.75; 'When k = 60, Y = 111.6)
10. You are given:
Variance of X = 9; Regression equations are
8 X - 10Y + 66 = 0
40 X - 18Y = 214
Find (a) The Mean values of X and Y
(b) Coefficient of Correlation between X and Y,and
(cl Standard Deviation of Y. (C.A.)

(Ans: (a) X = 13, Y = 17 (b) r = 0.6 (c) Standard


Deviation of Y=4)
11. Find the mean values of the variables X and Y and
correlation coefficinet from the' following regression
equations: (M.Com, H.P. University)
2Y - X - 50 = 0
3Y - 2X - 10 = 0 (Ans: X = 130; Y = 90; r = 0.87)
12. The correlation coefficient betwen two dependent
variables X and Y is 0.8 and their means are
respectively 25 and 22 and standard deviations are
respectively 4 and 5.
(i) Find the two lines of regression
(ii) Find the expected value of X when the value of Y
is 18 and
(iii) Find the expected value of Y when the value of X
is 25
(Ans: (i) X = 0.64Y + 10.92; Y = X -3
. (ii) X = 22.44 (iii) Y = 22)
18 25
13• Given:
x x
£.X = 56, £ Y = 40, £ X = 524, £ Y = 256,
<£.XY = 364, N = 8
(i) Find the regression equation of X on Y and
(ii) The Correlation coefficient (I.C.W.A)
Hint: Apply Actual Values formulae for calculations of bxy
and byx (Ans: (i) X = 1.5Y - 0.5; (ii) r = 0.98)
155
14. From the following data calculate
(i) Correlation Coefficient
(ii) Standard Deviation of Y
X = 0.85Y Y = 0.89X
Standard Deviation of X = 3 (M.Com, Rajasthan)
(Ans: (i) r = 0.869; (i i ) Standard Deviation of Y = 3-07)
15. The Correlation coefficient between two _variables
_ X and Y is r = 0.6. If Sx = 1 . 5 , % = 2.00 X = 10 and
Y = 20, find the regression lines of
(i) X on Y and (ii) Y on X
(Ans: (i) X = 0.45Y + 1 and (ii) Y= 0.8 X + 12)
II : -
‘ V// ADDITIONAL PROBLEMS

CHAPTER - I I : MEASURES OF CENTRAL TENDENCY

1. The marks secured by a student in an examination are as


follows:
Subjects Marks Subjects Marks
Economic Development 60 S tati sties 90
Advanced Accounts 80 Cost Accounts 85
Mercantile Law 45 Income Tax 75

I he Subjects Advanced Accounts and Cost Accounts are twice


important than others. Calculate Mean Marks. (Ans. Kw = 75)

2. The following figures denote the monthly salaries (in Rs)


paid to sixty employees of an organisation.4

970 1340 1060 860 875 1400 1280 1020 600


1155 1240 660 780 1100 980 960 920 800
680 1275 840 560 1140 600 650 1500 560
1225 985 880 840 1200 1300 970 1085 760
580 1040 540 860 1120 680 775 830 775
1130 1080 820 580 645 620 800 900 990
1000 1480 760 780 640 1000

The Organisation has announced bonus a t the following rates;

Monthly Income Bonus


(in Rs) (in Rs)
500-600 50
600-700 75
700-800 100
800-1000 125
1000-1200 150
1200 and more 175

Calculate the average Bonus per Employee payable by the


organi sation. (B.Com, Osm) (Ans.X= Rs.122.08)

3. The Mean age of 250 employees is 25.8 years. The mean age
of 150 of them is 24. Find out the mean age of remaining
employees. (Ans. Mean Age = 28.5 years)

4. The sum of two quartiles is 40 and th e ir difference is 12.


Find out the two Q uartiles. (Ans. = 14, Qg = 26)

1
157

Draw a histogram from the following data and measure tne


Modal Value:

Size Frequency
0-5 15
5-10 ;;r:§§ - 32
10-15 49
15-20 56
20-25 51
25-30 23
30-35 .i i . ) | ; 17
35-40 7
( B.Com. Osm) (Ans. = 17.92)

Calculate the average daily wage for the workers of two


facto ries:
Factory A Factory B
No. of wage earners 350 300
Average daily wage Rs. 2.00 Rs.2.50
( B.Com, Osm.) (Ans. Rs. 2.23)

7. An investor purchased securities of a company investing a


sum of Rs.2,400 every month. I f he bought these securities
at Rs.10, Rs.12 and Rs.15 during the three months
respectively. What is the average price paid by him.
Hint: Use H.M. (Ans. Rs.12) (B.Com, Osm, A p ril, 1987)

8. An investor buys Rs.120 worth of shares in a company each


month. During the f i r s t 5 months he bought the shares at a
price of Rs.10, Rs.12, Rs.15, Rs.20 and Rs.24 per share.
After the 5th month what is the average price paid for the
shares in his possession? (B.Com, Osm, Oct. 1987)
Hint: Use H.M. (Ans. Rs.14.63)

9. Determine the median wage graphically from the following


data:

Wages No .of Workers Wages No.c f Workers


in Rs) (in Rs) j.f f ' j ■i' f ■
20-40 4 100-120 12
40-60 6 120-140 7
60-80 10 140-160 3
80-100 16 (B.Com, Osm) (Ans. M = 91.25)
Hint: Draw a cummulative frequency curve by the Less than
method and determine Median.

10. The number of divorces per 1,000 marriages in a big town


in some foreign country increased from 96 in 1965 to 120
158
in 1985. Find the annual rate of increase of the divorce
rate for the period 1965 to 1985. (Ans. 1.1%)

11. A factory pays workers on piece rate basis and also a


bonus to each worker on the basis o f individual output in
each quarter. The rate of bonus payable is as follows:

Output in units Bonus of Rupees


70-74 40 -
75-79 45
80-84 50
85-89 60
90-94 70
95-99 80
100-104 100
The individual output of a batch of 50 workers is given
below:-
89 94 87 84 85 83 97 88 97 83 78 92 88 80 75 76 84 81 75 72 88
82 95 93 83 86 80 86 101 98 93 85 99 82 77 80 83 81 87 91 98 87
39 71 82 103 90 72 80 82

By suitable c lassific atio n you are required to find:

( i ) Average bonus per worker for the quarter.


( i i ) Total quarterly bonus paid to the whole batch
( i i i ) Average output per worker (C.A.)

(Ans. f i ) Rs.59.3 ( i i ) Rs.2965 ( i i i ) 85.9 un its.

12 Calculate the most suitable average giving reasons for


your choice.
Value Frequency
Less than 100 40
100-200 89
200-300 148
300-400 64
400 and above 39
(C .A .) (Ans. M = 241.2)

13. A Vehicle, when climbing up a gradient, consumes petrol


at the rate of one l i t r e per 8 kms., while climbing down,
i t gives 12 kms. per l i t r e . Find its average rate of
consumption for a to and fro travel between two places
situated at the two ends of a 25 kilometres long gradient.
(Ans. H.M. = 9.6 kmsl
'' Cv \■ ''■ V djjrV; * I*; 11 v < i' .f </vV/Svlt1

14. A c y c lis t covers his f i r s t three miles at an average speed


of 8 m.p.h. Another two miles at 3 m.p.h and the la s t two
159
miles at 2 m.p.h. Find the average speed for the entire
journey. (Ans. H.Mw.= 3.4 m .p.h.)

15. Calculate Mean, Median and Mode from the following data:-

Class Interval f Class Interval f


10-20 4 10-60 124
10-30 16 10-70 137
10-40 56 10-80 146
10-50 97 10-90 150
(Ans.T = 46.33; M = 44.63; M0 = 4 0.67).

Find the class interval s i f the arithmetic mean of the


following d istrib u tio n is 486 and assumed mean 450.
Step Deviation : -3 -2 -1 0 1 2 3
Frequency : 10 18 20 26 30 28 18

17. In a certain examination, the average grade of a ll


students in class A is 68.4 and students in class B is
71.2. I f the average of both classes combined is 70, Find
the ra tio of the number of students in class A to the
number in class B. (Ans. 30:40)

18. Claculate arithm etic mean from the following data.


Temperature No .o f days
-40 to -30 10
-30 to -20 28
-20 to -10 30
-10 to 0 42
0 to 10 65
10 to 20 180
20 to 30 10 (Ans. T = 4.29°C)

19. A Person travels the f i r s t km at 10 Km per hour, the


second km at 3 km per hour and the th ird km at 6 km per
hour. What is his average speed- (Ans. 5 Km p.h)

20. An aeroplane travels distances of 2,500, 1200 and 500 kms


at speeds 500, 400 and 250 kms per hour respectively. Find
the average speed for the en tire t r ip , Commenting an the
choice of your average. (Ans.H.Nto = 420 Knv.p.h)

21. The average monthly sales for the f i r s t eleven months of


the year in respect o f a certain salesmen were Rs.12,000,
but Hue to his illn e s s during the la s t month, the average
monthly sales for the whole year came down to Rs.11,375.
What\Was the value of his sales during the la s t month?
(Ans. Rs.4,500)
160
22. Calculate the median and the Quartiles for the following
data:
Amount of wages per day No.of workers receiving
such rate of wages
Not exceeding Rs. 8 85
Over Rs.8 belt not exceeding Rs.10 65
Over Rs.10 but not exceeding Rs.12 59
Over Rs.12 but not exceeding Rs.14 50
(Ans. M = 9.37; Q,= 7.523; Q3 = 11.5)

23. The following table gives you the d istribution of marks


secured by some students in an examination:
Marks No .of Students
0-20 42
21-30 38
31-40 120
41-50 84
51-60 48
61-70 36
71-80 31
(1) Median Marks
( i i ) The percentage of fa ilu re s i f the mil
pass is 35 marks. (Ans. ( i ) = -40.'

24. The following table gives the d istrib u tion of males in an


Indian town. Find the median age.
Age Groups Males Age Groups Males
0-9 2756 50-59 610
10-19 2124 60-69 245
20-29 1677 70-79 67
30-39 1481 80-89 6
40-49 1021 90-99 3
(Ans. M = 20.2 years)

25. The following table gives the weekly wages in rupees in a


certain commercial organisation:
Weekly Wages (Rs) f Weekly Wages (Rs) f
30-32 2 40-42 62
32-34 9 42-44 39
34-36 25 44-46 20
36-38 30 46-48 11
38-40 49 48-50 3
Calculate:- ( i ) The median and the th ird q u a rtile wages
( i i ) The number of wage earners receiving between
Rs.37 and Rs.45 per week.
(Ans. ( i ) M = Rs.40.3; Q3 = 42.5; ( i i ) 175)

5
161

26. Twenty percent of the workers in a firm employing a total


of 2,000 earn less than Rs.4 per hour, 440 earn from Rs.4
to Rs.4.49 per hour, 24% earn from Rs. 4.50 to 4.99 per
hour, 370 earn from Rs.5.00 to 5.49 per hour, 12% earn
from Rs.5.50 to 5.99 per hour and the rest earn Rs.6.00 or
more per hour. Set up a frequency table and calculate the
model value. (Ans. MO = Rs.4.625)

27. A train goes at a speed of 20 kms.per hour for the f ir s t


16 kms, at a speed of 40 kms per hour for 20 kms. I t
covers the la s t 10 kms at a speed of 15 kms per hour. Find
out its average speed. (Ans. H.Mw. = 23.4)

28. You take a tr ip which en tails tra v e llin g 900 miles by


train at an average speed of 60 m .p.h., 3000 miles by boat
at an average of 25 m .p.h.; 400 miles by plane at 350
m.p.h. and fin a lly 15 miles by taxi at 25 m.p.h. What is
your average speed for the entire distance.
(Ans. H.Mw = 31.5 m .p.h.)

29. Following is the distribution of marks obtained by 50


students in Accounts:
Marks No .of Students
More than)
0 50
10 46
20 40
30 20
40 10
50 3
Calculate the median marks. I f 60 percent of the students pass
this te s t, find the minimum marks obtained by a pass candidate.
(Ans.M= 27.5; P4Q = 25)

30. The average marks in S ta tis tic s in a class of 30 were 52.


The top six students had an average of 31. What were the
average marks of the other students ? (Ans.= 57.25)

31. F ifty students appeared in an entrance examination. The


re s u lt of those who secured less than f i r s t class marks
(60%) is given below:

Marks No .of students


1-10 8
10-20 12
20-30 20
162

I f the average for a ll the students was 25 marks, find out


the average marks for those who secured 60% or more.
(Ans. 53 Marks)

32. A candidate was subjected to oral test o f 25 marks each


and w ritten tests of 75 marks each in three subjects A,B,
and C. He secured 15, 11 and 9 marks in oral tests and 55,
32 and 28 marks in w ritten tests in subjects A, B and C
respectively. Calculate weighted average of marks obtained
in w ritten tests taking as weights % of marks obtained in
corresponding oral te sts. (Ans. Xw = 40.8)

CHAPTER - I I I : MEASURES OF DISPERSION:

1. Find middle 50%, middle 80% and C o -efficien t of Quartile


deviation and percentile deviation from the following data.

Variable : 20 40 60 80 100 120


f : 6 3 12 10 4 5

(Ans. ( i ) C o -e ffic ie n t of Q.D = 0.14


( i i ) C o -e ffic ie n t of
percentile Deviation: = 0.71)
Hint: ( i ) Middle 50% means Q^and Q3

(ii) Middle 80% means P1Q and PgQ

2. Calculate the standard Deviation, given the squares of


deviations for two series calculated from th eir Actual
Means.
Series N Sum of the squares of
Deviations from the
Mean
A 150 2,40,000
B 20 5,00,000
(Ans. (TA = 40 ;CT B = 50)

Calculate (T of f i r s t eight natural numbers.


(Ans. <T= 2.29)

Hint: (T = \ 1/12 (n2- 1) n = No .o f items.


V rj: if j)' I'::, • ,/ ' .; j ' \' " :( ''!V i I!
4. The following data pertains to two workers doing the same
job in a factory.
Worker “A" Worker *,'B"
Mean time of completing
job (minutes) 40 42

7
163
Standard Deviation (minutes) 8 6
Who is more consistent worker ? ( B.Com, Osm, A p ril, 1986)
(Ans. Worker B is more consistent)
5. Two Typists were employed on identical job work. Show the
following results.
Typist "X" Typist "Y"
Mean time of completing
job (minutes) 40 30
Standard Deviation (minutes) 5 6

(i) Which worker appear to be faster in completing the


job?
(ii) Which worker appear to be more consistent ?
(Ans. ( i ) As the time taken by
"Y" is less than "X"
Y is faster.
( i i ) Worker "X" i s more
consi s te n t ) .

6. The mean runs scored by three batsmen X,Y,and Z in a test


series are 10, 45 and 75. The Standard Deviation of their
runs is 4, 10.6 and 12.8 respectively. Who is the most
consistent of the three. (Ans. Z is more consistent of
the three).

7. The following are the particulars submitted to you.


Brand "A" Brand "B"
T 50 40
Cr 3.5 2.8

Which Brand is to be preferred and why ?


(Ans. Arithmetic Mean for Brand "A" is higher therefore,
Brand "A" is preferred)

8. For a moderately skewed data the Arithmetic Mean is 90.


The Co-efficient of variation is 40 and Karl Pearson's co­
e f f ic ie n t of skewness is 0.35. Find out the Mode and
Median. (Ans. Mode=77.4; Median = 85.80) 0
1
9

9. For a frequency distribution the Semi-inter Quartile range


is 18, i f upper Quartile is 60 find out the lower
Quartile. (Ans. Q = 24)
1
10. The sum of two Quartiles is 30 and their difference is 10.
Find the Quartile Deviation and it s Co-efficient.
(Ans. Q.D. = 5
Co-efficient of Q.D. 0.33)
VIII Typical Problem s

I. M easures of Central Ten d en cy


1 From a batch of 13 students who had appeared for an
examination 4 students had failed. The marks of the successful
candidates were 4 1 , 5 7 , 3 8 , 6 1 , 3 6 , 5 5 , 7 1 , 5 0 . 4 4 . Calculate
the Median mark.
(Ans \ Median mark= 41)
(B.Com., Osmania 1986)

2. The annual rates of growth of out put of a factory in 5


years are 5.0, 7.5, 2.5, 5.0 and 10 respectively. What is the com­
pound rate of growth of out put per annum for the period ?
(Ar.s : 5.9% per annum)
Hint : Use G.M.
3. The Population of a country rose from 84 million in 1961
to 108 million in 1971. Find the annual rate of growth of population.
Solution : Let the rate of growth be denoted by r. Applying the
compound interest formula :
Pr?=Po (1 + r ) n; Where Pn—The value at the end of the
period; Po — The value at the beginning of the period; r=R ate of
change ahd n=Length of time oeriod.
1 0 8 = 8 4 ( 1 + r ) 10
Log 108 — Log 84
Taking Logarithms, Log ( 1 + r ) =
10
2.0334 — 1.9243
Log ( 1 + r ) : = 0.0109
10
1 + r = A.L. [0.01 09] = 1.025
= 1.025 — 1 = 0 .0 2 5 = 2.5%*4

4. The population of a State was 300 million in 1951. It


became 640 million in 1971. Calculate the percentage compound
rate of growth per annum.
(Ans : 3.9%;
(B.Com., Osmania 1986)
5. The mean weight of a member in a family of 6 members »■ Msasuroe o f Dispersion
is 120 lb. The individual weight of 5 of them is 118; 110; 130;
H. Given the following information, find the standard devi-
118 and 118 (in lb) respectively. What is the weight of the sixth
member in the family ? *= 1 0 . SX=80, 2 X a= 1 , 0 0 0 .
(Arts : 130 lb.) ( A m t <r = 8)
(B.Com., 0sm ani8, 1979)
- ..... -V >. M "' . : [H in t: o = ------ this formula is the
6. Find the average rate of increase in population which In
the first decade has Increased by 20%, in the second decade by 30% second direct method of computing the <y. where the values of the
and in the third decade by 40%. variables are used directly]
(Ans : 29.7%)
( H in t: Use G M.)
12. Find the missing information from the following ;
v?fh Group I Group II
7. An aeroplane flies round a square the sides of which* Group III Combined
measure 100 miles each. The aeroplane covers at a speed of 100- Number 50 ? 90 200
miles per hour the first side, at 200 m.p.h. the second side, at 300 <J 6 7 ? 7.746
m.p.h. the third side and at 400 m.p.h. the fourth side. What is •
X 113 ?
the average speed of the aeroplane around the square ? 115 116

(Ans: 192 m.p.h.) (Ans: N , = 60; X » = 1 2 0 ; ^ = 8)


(H in t: Use H.M) (B.Com., Delhi)

13. The first of two sub groups has 100 items with mean 15
8. A person purchases one Kilogram of Cabbages from each
of the four places at the rate of 20 kg; 1 6kg; 12 kg and 10 kg per- and standard deviation 3. If the whole group has 250 items with
rupee respectively. On the average how many kilograms of mean 15.6 and standard deviation \ / 1 3.44, find the standard devia­
cabbages has he purchased per rupee. tion of the second sub-group.
(Ans : 13.5 Kgs. per rupee)
(Ansi <ra= 4 )
(Hint : Use H. M .) 9
0
1

14. The mean and standard deviation of a series of 20 items


9. If the industrial production increased 3% in First year,
4% in Second year and 5% in Third year, what is the average annual were calculated by a student as 20 cm. and 5 cm. respectively. But
increase for three years. (Ans : G. M. = 4%) while calculating them an item 13 was misread as 30. Find the
>i v m .■...•• !'M,f fi' • I » C ! f ’i s K i-a h * correct mean and standard deviation.
10. Average rainfall of a city from Monday to Saturday is
0.3 inch. Due to heavy rainfall on Sunday, the average rainfall for {Ans : Correct m e a n = 1 9 . l 5 and correct or= 4 .6 6 )
the week increases to 0.5 inch. What was the rainfall on Sunday ?
H in t: O =
(Ans : 1.7 inch)
67

6 laid deviation of a set of 100 obser-


I ' 120 * 10 and 5 respectively by a computer
ne 5 0 in place o f 40 for one observation,
and Variance.
/ct Mean = 39.9; correct variance=23.99.)
TV*
~ (X)a

16. Mean of 200 items is 8 0 and their standard deviation is


10. Find the sum of the items and elso the sum of squares of all
the items.
(Ans : 2 x = 1 6 ,0 0 0 ; 2 x a=: 13,00,000)

17. Given the following results relating to two groups


containing 20 and 30 observations, calculate the coefficient of
variation of ail the 5U observations by combining both the groups.

Groups
I
. 2X 45 55
2Xa 118 132

(Ans : C.V. of both groups combined ^ i.e - ^ 1" • X 1 00 =50.2% )

18. Find the actual class groups from the data given below:
d1 -3 -2 -1 0 1 2 3
f 10 15 25 25 10 10 5
You are given that the mean of the distribution is 31 and
standard deviation 15.9.
(Ans : C = 1 0 ; Assumed mean = 35 and class groups
are : 0-10, 10-20, 20-30, 30-40, 40 50, 50-60 and
60-70)

19. The mean and standard deviation calculated from 2


observations are 15 and 10. If an additional observation 36, left
out through oversight, be included in the calculations, find the
corrected mean and standard deviation.
(Ans : Correct X = 16 and correct cr = 10.73)
168

20. The mean and standard deviation of 15 items were found


to be 8 and 2 respectively. On checking, it was discovered that
one item il had been misread as 5. Calculate the correct mean and
standard deviation.
(Ans : Correct m ea n = 8 .4 ; Correct cr = 1.96)
(B.Com., Osmania May 1986)

III. Skewness
21. In a certain d istrib u tio n the fo llo w in g results were
obtained :
Mean =» 45; Median = 48; C oefficient of skewness = — 0.4.
Estimate the value of cx. (Ans : <j,= 2 2 .5 )
22 For a distribution Bow ley's coefficient of skewness is
— 0.3 6, Q j = 8.6 and Median = 1 2.3. What is its Q uartile C oeffi­
cient of dispersion.
(Ans : 0.24)
23. Karl Pearson's co e fficie n t of skewness of a distribution
is —
j—0.40. Its or is 8 and X is 30. Find Mode and Median of the
distribution. ( Ans : Mode = 26.8
Median =■ 28.9)
24. The sum of a set of 100 numbers is 4000 and the sum
of its square is 162500 and Median is 4 1 . Find the coefficient of
skewness. (Ans : — 0.6)
25. In a frequency d istrib u tio n the co e fficie n t of skewness
based on quartiles is 0.6. If the sum of the upper and the lower
quartiles is 100 and the median is 38 find the value of the Upper
Quartiie. (Ans : G 8 = 70)
26. In a distribution, the difference of tw o quartiles is 15,
their sum is 35 and median 20, find the coefficient of skewness.
(Ans : — 0 .3 3 )
IV. CORRELATION
27. Calculate Correlation C oefficient fio m the fo llo w in g
results :
N = 10; 2 x = 1 4 0 ; 2 y = 1 5 0 .
2 ( x — 10)2 = 180; £ ( y — 1 5 )a~ 2 1 5 .
S (x — 10) (y— 1 5) = 60.
(Ans ! r—4-0.818)
169

28. The coefficient of rank correlation of the marks obtained


by 10 students in statistics and accountancy was found to be 0.8
It was later discovered that the difference in ranks in the two
subjects obtained by one of the students was wrongly taken as 7
instead of 9. Find the correct coefficient of rank correlation.

(Ans : 0.606)

29. If covariance between x and y variables is 10 and the


variance of x is 16 and standard deviation of y is 3, find the coeffi­
cient of correlation. (Ans : r = + 0 .8 3 )

30. The rank correlation coefficient between marks in


statistics and Economics is obtained as 0.8. If the sum of the
squares of the differences in ranks is given as 33, find the number
of students in the group. (Ans : 10)
(ES.Com., Osmania, May'83)

31. From the data given below find the number of items 'n'
r = 0 .5 , £ x y = 120, S.D. of y = 8, 2 x a= 9 0
(Ans : 10)
(B.Com., Osmania, May’85)

32. Calculate coefficient of correlation from the following


data :
x: 2.5 1.7 3.1 6.5 4.2
y: -5 .2 -3 .5 + 4 .1 -6 .2 + 2 .8
(Ans : r = - 0 .1 6 9 )
■ifl
YjiH
33. In a question on correlation the value of r is 0.64 and
its P.E. = 0 .1312 . What was the value of N ? (Ans ; 9)

34. Calculate the coefficient of concurrent deviations from


the following data :

No. of pairs of observations = 96


igo. of pairs of concurrent deviations = 36.

(Ans : — 0.577)
(B.Com., Osmania, May 1984)

1
L o g a rith m s 170

Mean Differences
0 i 2 3 4 S 6 7 8 9 1 2 3 4 5 6 7 8 9

10 0000 0043 0086 0128 0170 0212 0253 0294 0334 0374 4 8 12 17 21 25 29 33 37

11 0414 0453 0492 0531 0569 0607 0645 0682 0719 0755 4 8 11 15 19 23 26 30 34
12 0792 0828 0864 0899 0934 0969 1004 1038 1072 1106 3 7 10 14 17 21 24 28 31
13 1139 1173 1206 1239 1271 1303 1335 1367 1399 1430 3 6 10 13 16 19 23 26 29
14 1461 1492 1523 1553 1584 1614 1644 1673 1703 1732 3 Q 9 12 15 18 21 24 27
15 1761 1790 1818 1847 1875 1903 1931 1959 1987 2014 3 6 8 11 14 17 20 22 25
16 2041 2068 2095 2122 2148 2175 2201 2227 2253 2279 3 5 8 11 13 16 18 21 24
17 2304 2330 2355 2380 2405 2430 2455 2480 2504 2529 2 5 7 10 12 15 17 20 22
18 2553 2577 2601 2625 2648 2672 2695 2718 2742 2765 2 5 7 9 12 14 16 19 21
19 2788 2810 2833 2856 2878 2900 2923 2945 2967 2989 2 4 7 9 11 13 16 18 20
20 3010 3032 3054 3075 3096 3118 3139 3160 3181 3201 2 4 6 8 11 13 15 17 19

21 3222 3243 3263 3284 3304 3324 3345 3365 3385 3404 2 4 6 8 10 12 14 18 18
22 3424 3444 3464 3483 3502 3522 3541 3560 3579 3598 2 4 6 8 10 12 14 15 17
23 3617 3636 3655 3674 3692 3711 3729 3747 3766 3784 2 4 6 7 9 11 13 15 17
24 3802 3820 3838 3856 3874 3892 3909 3927 3945 3962 2 4 5 7 9 11 12 14 16
25 3979 3997 4014 4031 4048 4065 4082 4099 4116 4133 2 3 5 7 9 10 12 14 13

26 4150 4168 4183 4200 4210 4232 4249 4265 4281 4298 2 3 5 7 8 10 11 13 15
27 4314 4330 4346 4362 4378 4393 4409 4425 4440 4456 2 3 5 6 8 9 11 13 14
28 4472 4487 4502 4518 4533 4548 4564 4579 4594 4609 2 3 5 0 8 9 11 12 14
29 4624 4639 4654 4669 4683 4698 4713 4728 4742 4757 1 3 4 6 7 9 10 12 13
00 4771 4786 4800 4814 4829 4843 4857 4871 4886 4900 1 3 4 6 7 9 10 11 l3

31 4914 4928 4942 4955 4969 4983 4997 5011 5024 5038 1 3 4 6 7 8 10 11 12
32 5051 5065 5079 5092 5105 5119 5132 5145 5159 5172 1 3 4 5 7 8 9 11 12
33 5185 5198 5211 5224 5237 5250 5263 5276 5289 5302 1 3 4 5 6 8 9 10 12
34 5315 5328 5340 5353 5366 5378 5391 5403 5416 5428 1 3 4 5 6 8 9 10 11
35 5441 5453 5465 5478 5490 5502 5514 5527 5539 5551 1 2 4 5 6 7 9 10 11

36 5563 5575 5587 5599 5611 5623 5635 5647 5658 5670 1 2 4 5 6 7 8 10 11
37 5682 5694 5705 5717 5729 5740 5752 5763 5775 5786 1 2 3 5 6 7 8 9 10
38 5798 5809 5821 5832 5843 5855 5860 5877 5888 5899 1 2 3 5 6 7 8 9 10
39 5911 5922 5933 5944 5955 5966 5977 5988 5999 6010 1 2 3 4 5 7 8 9 10
40 6021 6031 6042 6053 6064 6075 6085 8096 6107 6117 1 2 3 4 5 6 8 9 10

41 6128 6138 6149 6180 6170 6180 6191 6201 6212 6222 1 2 3 4 5 6 7 8 9
42 6232 6243 6253 6263 6274 6284 6294 6304 6314 6325 i 2 3 4 5 6 7 8 9
43 6335 6345 6355 6365 6375 6385 6395 6405 6415 6425 1 2 3 4 5 6 7 8 9
44 6435 6444 6454 6464 6474 6484 6493 6503 6513 6522 1 2 3 4 5 6 7 8 9
45 6532 6542 6551 6561 6571 6580 6590 6599 6809 6618 1 2 3 4 5 6 7 8 9

46 6628 6637 6646 6656 6665 6675 6684 6693 6702 6712 1 2 3 4 5 6 7 7 8
47 6721 6730 6739 6749 6758 6767 6776 6785 6/94 6803 1 2 3 4 5 5 6 7 8
48 6812 6821 6830 -6839 6848 6857 6866 6875 6884 6893 1 2 3 4 4 5 6 7 8
49 8902 6911 6920 6928 6937 6946 6955 6964 6972 6981 i 2 3 4 4 5 6 7 8
50 6990 6998 7007 7016 7024 7033 7042 7050 7059 7067 i 2 3 3 4 5 6 7 8

5f 7076 7084 7093 7101 7110 7118 7126 7135 7143 7152 1 2 3 3 4 5 6 7 8
52 7160 7168 7177 7185 7193 7202 7210 7218 7226 7235 1 2 2 3 4 5 6 7 7
53 7243 7251 7259 7267 7275 7284 7292 7300 7308 7316 1 2 2 3 4 5 6 6 7
54 7324 7332 7340 7348 7356 7364 7372 7380 7388 7396 1 2 2 3 4 5 6 6 7
Logarithms 171

Mean Differences

0 i 2 3 4 5 < 7 8 9 i 2 3 4 5 6 7 8 t

55 7404 7412 7419 7427 7435 7443 7451 7459 7466 7474 1 2 2 3 4 5 5 8 7

56 7482 7490 7497 7505 7513 7520 7528 7536 7543 7551 1 2 2 3 4 5 5 6 7
57 7559 7566 7574 7582 7589 7597 7604 7612 7619 7627 1 2 2 3 4 5 5 6 7
58 7634 7642 7649 7657 7664 7672 7679 7686 7694 7701 1 1 2 3 4 4 5 6 7
59 7709 7716 7723 7731 7738 7745 7752 7760 7767 7774 1 1 2 3 4 4 5 6 7
60 7782 7789 7796 7803 7810 7818 7825 7832 7839 7846 1 1 2 3 4 4 5 6 6

61 7853 7860 7868 7875 7882 7889 7896 7903 7910 7917 1 1 .2 3 4 4 5 6 6
7924 7931 7938 7945 7952 7959 7966 7973 7980 7987 1 1 2 3 3 4 5 6 6
s'ssa 8 S 8 3 8 S2SS2 83333! 3 2 3 3 3 o 8 2 3 £ 8 2 8 8

7993 8000 8007 8014 8021 8028 8035 8041 8048 8055 1 1 2 3 3 4 5 5 6
8062 8069 8075 8082 8089 8096 8102 8109 8116 8122 1 1 2 3 3 4 5 5 6
8129 8136 8142 8149 8156 8162 8169 8176 8182 8189 1 1 2 3 3 4 5 5 6

8195 8202 8209 8215 8222 8228 8235 8241 8248 8254 1 1 2 3 3 4 5 5 6
8261 8267 8274 8280 8287 8293 8299 8306 8312 8319 1 1 2 3 3 4 5 5 6
8325 8331 8338 8344 8351 8357 8363 8370 8376 8382 1 1 2 3 3 4 4 5 6
8388 8395 8401 8407 8414 8420 8426 8432 8439 8445 1 1 2 2 3 4 4 5 8
8451 8457 8463 8470 8476 8482 8488 8494 8500 8506 1 1 2 2 3 4 4 5 8

8513 8519 8525 8531 8537 8543 8549 8555 8561 8567 1 1 2 2 3 4 4 5 5
8573 8579 8585 8591 8597 8603 8609 8615 8621 8627 1 1 2 2 3 4 4 5 5
8633 8639 8645 8651 8C57 8663 8669 8675 8681 8686 1 1 2 2 3 4 4 5 5
8692 8698 8704 8710 8716 8722 8727 8733 8739 8745 1 1 2 2 3 4 4 5 5
8751 8756 8762 8768 8774 8779 8785 8791 8797 8802 1 1 2 2 3 3 4 5

8808 8814 8820 8825 8831 8837 8842 8848 8854 8859 1 1 2 2 3 3 4 5 5
8865 8871 8876 8882 8887 8893 8899 8904 8910 8915 1 1 2 2 3 3 4 4 5
8921 8927 8932 8938 8943 8949 8954 8960 8965 8971 1 1 2 2 3 3 4 4 5
8976 8982 8987 8993 8998 9004 9009 9015 9020 9025 1 1 2 2 3 3 4 4 5
9031 9036 9042 9047 9053 9058 9063 9069 9074 9079 1 1 2 2 3 3 4 4 5

9085 9090 9096 9101 9106 9112 9117 9122 9128 9133 1 1 2 2 3 3 4 4 5
9138 9143 9149 9154 9159 9165 9170 9175 9180 9186 1 1 2 2 3 3 4 4 5
9191 9196 9201 9206 9212 9217 9222 9227 9232 9238 1 1 2 2 3 3 4 4 5
9243 9248 9253 9258 9263 9269 9274 9279 9284 9289 1 1 2 2 3 3 4 4 5
9294 9299 9304 9309 9315 9320 9325 9330 9335 9340 1 1 2 2 3 3 4 4 5

9345 9350 9355 9360 9365 9370 9375 9380 9385 9390 1 1 2 2 3 3 4 4 5
9395 9400 9405 9410 9415 9420 9425 9430 9435 9440 0 i 1 2 2 3 3 4 4
9445 9450 9455 9460 9465 946* 9474 9479 9484 9489 0 1 1 2 2 3 3 4 4
9494 9499 9504 9509 9513 9513 9523 9528 9533 9538 0 1 1 2 2 3 3 4 4
9542 9547 9552 9557 9562 9566 0571 9576 9581 9586 0 1 1 2 2 3 3 4 4

9590 9595 9600 9605 9609 9614 9619 9624 9628 9633 0 1 1 2 2 3 3 4 4
9638 9643 9647 9653 9657 9661 9666 9671 9675 9680 0 1 1 2 2 3 3 4 4
9685 9689 9694 9699 9703 9708 9713 9717 9722 9727 0 1 1 2 2 3 3 4 4
9731 9736 9741 9745 9750 9754 9759 9763 9768 9773 0 1 1 2 2 3 3 4 4
95 9777 9782 9786 9791 9795 9800 9805 9809 9814 9818 0 1 1 2 2 3 3 4 4

9823 9827 9832 9836 9841 9845 9850 9854 9859 9863 0 1 1 2 2 3 3 4 4
96
97 9868 9872 9877 9881 9886 9890 9894 9899 9903 9908 0 1 1 2 2 3 3 4 4
9912 9917 9921 9926 9930 9934 9939 9943 9948 9952 0 1 1 2 2 3 3 4 4
98
9956 9961 9965 9969 9974 9978 9983 9987 9991 9996 0 1 1 2 2 3 3 3 4
99
A n tilo g a rith m s 172

Mean Differences
0 1 2 3 4 5 6 7 8 9 1 2 3 4 5 6 7 8 9

00 1000 1002 1005 1007 1009 1012 1014 1016 1019 1021 0 0 1 1 1 1 2 2 2

.01 1023 1026 1028 1030 1033 1035 1038 1040 1042 1045 0 0 1 1 1 1 2 2 2
02 1047 1050 1052 1054 1057 1059 1062 1064 1067 1069 0 0 1 1 1 1 2 2 2
■03 1072 1074 1076 1079 1081 1084 1086 1089 1091 1094 0 0 1 1 1 1 2 2 2
•04 1096 1099 1102 1104 1107 1109 1112 1114 1117 1119 0 1 1 1 1 2 2 2 2
05 1122 1125 1127 1130 1132 1135 1138 1140 1143 1146 0 1 1 1 1 2 2 2 2

-06 1148 1151 1153 1156 1159 1161 1164 1167 1169 1172 0 1 1 1 1 2 2 2 2
•07 1175 1l76 1180 1183 1186 1189 1191 1194 1197 1199 0 1 1 1 1 2 2 2 2
-08 1202 1205 1208 1211 1213 1216 1219 1222 1225 1227 0 1 1 1 1 2 2 2 3
•09 1230 1233 1236 1239 1242 1245 1247 1250 1253 1256 0 1 1 1 1 2 2 2 3
•10 1259 1262 1265 1268 1271 1274 1276 1279 1282 1285 0 1 1 1 1 2 2 2 3

•11 1288 1291 1294 1297 1300 1303 1306 1309 1312 1315 0 1 1 1 2 2 2 2 3
•12 1318 1321 1324 1327 1330 1334 1337 1340 1343 1346 0 1 1 1 2 2 2 2 3
•13 1349 1352 1355 1358 1361 1365 1368 1371 1374 1377 0 1 1 1 2 2 2 3 3
•14 life) 1384 1387 1390 1393 1396 1400 1403 1406 1409 0 1 1 1 2 2 2 3 3
•15 1413 1416 1419 1422 1426 1429 1432 1435 1439 1442 0 1 1 1 2 2 2 3 3

•16 1445 1449 1452 1455 1459 1462 1466 1469 1472 1476 0 1 1 1 2 2 2 3 3
•17 1479 1483 1486 1489 1493 1496 1500 1503 1507 1510 0 1 1 1 2 2 2 3 3
•18 1514 1517 1521 1524 1528 1531 1535 1538 1542 1545 0 1 1 1 2 2 2 3 3
•19 1549 1552 1556 1560 1563 1567 1570 1574 1578 1581 0 1 1 1 2 2 3 3 3
•20 1585 1589 1592 1596 1600 1603 1607 1611 1614 1618 0 1 1 1 2 2 3 3 3

•21 1622 1626 1629 1633 1637 1641 1644 1648 1652 1656 0 1 1 2 2 2 3 3 3
•22 1660 1663 1667 1671 1675 1679 1683 1687 1690 1694 0 1 1 2 2 2 3 3 3
•23 1698 1702 1706 1710 1714 1718 1722 1726 1730 1734 0 1 1 2 2 2 3 3 4
•24 1738 1742 1746 1750 1754 1758 1762 1766 1770 1774 0 1 1 2 2 2 3 3 4
•25 1778 1782 1786 1791 1795 1799 1803 1807 1811 1816 0 1 1 2 2 2 3 3 4

•26 1820 1824 1828 1832 1837 1841 1845 1849 1854 1858 0 1 1 2 2 3 3 3 4
•27 1862 1866 1871 1875 1879 1884 1888 1892 1897 1901 0 1 1 2 2 3 3 3 4
•28 1905 1910 1914 1919 1923 1928 1932 1936 1941 1945 0 1 1 2 2 3 3 4 4
•29 1950 1954 1959 1963 1968 1972 1977 1982 1986 1991 0 1 1 2 2 3 3 4 4
30 1995 2000 2004 2009 2014 2018 2023 2028 2032 2037 0 1 1 2 2 3 3 4 4

•31 2042 2046 2051 2056 2061 2065 2070 2075 2080 2084 0 1 1 2 2 3 3 4 4
•32 2089 2094 2099 2104 2109 2113 2118 2123 2128 2133 0 1 1 2 2 3 3 4 4
•33 2138 2143 2148 2153 2158 2163 2168 2173 2178 2183 0 1 1 2 2 3 3 4 4
•34 2188 2193 2198 2203 2208 2213 2218 2223 2228 2234 1 1 2 2 3 3 4 4 5
•35 2239 2244 2249 2254 2259 2265 2270 2275 2280 2286 1 1 2 2 3 3 4 4 5

•36 2291 2296 2301 2307 2312 2317 2323 2328 2333 2339 1 1 2 2 3 3 4 4 5
•37 2344 2350 2355 2360 2366 2371 2377 2382 2388 2393 1 1 2 2 3 3 4 4 5
•38 2399 2404 2410 2415 2421 2427 2432 2438 2443 2449 1 1 2 2 3 3 4 4 5
•39 2455 2460 2466 2472 2477 2483 2489 2495 2500 2506 1 1 2 2 3 3 4 5 5
-40 2512 2518 2523 2529 2535 2541 2547 2553 2559 2564 1 1 2 2 3 4 4 5 5

•41 2570 2576 2582 2588 2594 2600 2606 2612 2618 2624 1 1 2 2 3 4 4 5 5
•42 2630 2636 2642 2649 2655 2661 2667 2673 2679 2685 1 1 2 2 3 4 4 5 6
•43 2692 2698 2704 2710 2716 2723 2729 2735 2742 2748 1 1 2 3 3 4 4 5 6
•44 2754 2761 2767 2773 2780 2786 2793 2799 2805 2812 1 1 2 3 3 4 4 5 6
•45 2818 2825 2831 2838 2844 2851 2858 2864 2871 2877 1 1 2 3 3 4 5 5 6

•46 2884 2891 2897 2904 2911 2917 2924 2931 2938 2944 1 1 2 3 3 4 5 5 6
•47 2951 2958 2965 2972 2979 2985 2992 2999 3006 3013 1 1 2 3 3 4 5 5 6
'48 3020 3027 3034 3041 3048 3055 3052 3069 3076 3083 1 1 2 3 4 4 5 6 6
49 3090 3097 3105 3112 3119 3126 3133 3141 3148 3155 1 1 2 3 4 4 5 6 6
I

173
Loflnrlt' Mean Differences
/7 i 9 1 2 3 4 5 6 7 8 9
1 « / / S
/ 3214 3221 3228 1 1 2 3 4 4 5 6 7
,i 3289 3296 3304 1 2 2 3 4 5 5 6 7
<7 3365 3373 3381 1 2 2 3 4 5 5 6 7
1 36 3443 3451 3459 1 2 2 3 4 5 6 6 7
r\ >16 3524 3532 3540 1 2 2 3 4 5 6 6 7
-55 Ox. . 597 3606 3614 3622 1 2 2 3 4 5 6 7 7
•M 3631 3639 3648 3681 3690 3698 3707 1 2 3 3 4 5 6 7 8
■57 3715 3724 3733 3741 3 ,^ 3767 3776 3784 3793 1 2 3 3 4 5 6 7 8
■58 3802 3811 3819 3828 3837 3846 3855 3864 3873 3882 1 2 3 4 4 5 6 7 8
■59 3890 3899 3908 3917 3926 3936 3945 3954 3963 3972 1 2 3 4 5 5 6 7 8
SO 3981 3990 3999 4009 4013 4027 4036 4046 4055 4064 1 2 3 4 5 6 8 7 a
•St 4074 4083 4093 4102 4111 4121 4130 4140 4150 4159 1 2 3 4 5 6 7 3 9
. 62 4169 4178 4138 4198 4207 4217 4227 4236 4246 4256 1 2 3 4 5 6 7 8 9
■63 4266 4276 4285 4295 4305 4315 4325 4335 4345 4355 1 2 3 4 5 6 7 8 9
•64 4365 4375 4385 4395 4406 4416 4426 4436 4446 4457 1 2 3 4 5 6 7 8 9
•85 4467 4477 4487 4498 4503 4519 4529 4539 4550 4560 1 2 3 4 5 6 7 8 9
•68 4571 4581 4592 4603 4613 4624 4634 4645 4856 4667 1 2 3 4 5 6 7 9 10
•67 4677 4688 4699 4710 4721 4732 4742 4753 4764 4775 1 2 3 4 5 7 8 9 10
•68 4786 4797 4808 4819 4831 4842 4853 4884 4875 4887 1 2 3 4 6 7 8 9 10
■69 4898 4909 4920 4932 4943 4955 4968 4977 4989 5000 1 2 3 5 6 7 8 9 10
•70 5012 5023 5035 5047 5058 5070 5082 5093 5105 5117 1 2 4 5 6 7 8 9 11
71 5129 5140 5152 5164 5176 5188 5200 5212 5224 5236 1 2 4 5 6 7 8 10 11
72 5248 5260 5272 5284 5297 5309 5321 5333 5346 5358 1 2 4 5 6 7 9 10 11
•73 5370 5383 5395 5408 5420 5433 5445 5458 5470 5483 1 3 4 5 6 8 9 10 11
•74 5495 5508 5521 5534 5546 5559 5572 5585 5598 5610 1 3 4 5 6 8 9 10 12
•75 5623 5636 5649 5662 5675 5689 5702 5715 5728 5741 1 3 4 5 7 8 9 10 12
76 5754 5768 5781 5794 5803 5821 5834 5848 5861 5875 1 3 4 5 7 8 9 11 12
■n 5888 5902 5918 5929 5943 5957 5970 5984 5998 6012 1 3 4 5 7 8 10 11 12
•78 6026 6039 6053 6067 6031 6095 6109 6124 6138 6152 1 3 4 6 7 8 10 11 13
•79 6166 6180 6194 6209 6223 6237 6252 6266 6281 6295 1 3 4 8 7 9 10 11 13
80 6310 6324 6339 6353 6368 6383 8397 6412 6427 6442 1 3 4 6 7 9 10 12 13
81 6457 6471 6486 6501 6516 6531 6546 6561 6577 8592 2 3 5 6 8 9 11 12 14
■82 6607 6622 6637 6653 6668 6683 6699 6714 6730 6745 2 3 5 8 8 9 11 9 14
•83 6761 6776 8792 6808 6823 6839 6855 6871 6887 6902 2 3 5 6 8 9 11 13 14
84 6918 6934 6950 6966 6982 6998 7015 7031 7047 7063 2 3 5 8 8 10 11 13 15
85 7079 7096 7112 7129 7145 7161 7178 7194 7211 7228 2 3 5 7 8 10 12 13 15
86 7244 7261 7278 7295 73'1 7328 7345 7362 7379 7396 2 3 5 7 8 10 12 13 15
87 7413 7430 7447 7464 7*82 7499 7516 7534 7551 7568 2 3 5 7 9 10 12 14 16
88 7586 7603 7621 7638 7656 7674 7691 7709 7727 7745 2 4 5 7 9 11 12 14 16
89 7762 7780 7798 7816 7834 7852 7870 7889 7907 7925 2 4 5 7 9 11 13 14 16
•90 7943 7962 7980 7998 8017 8035 8054 8072 8091 8110 2 4 6 7 9 n 13 15 17
91 8128 8147 8166 8185 8204 8222 8241 8260 8279 8299 2 4 6 8 9 11 13 15 17
92 8318 8337 8356 8375 8395 8414 8433 8453 8472 8492 2 4 6 8 10 12 14 15 17
93 8511 8531 8551 8570 8590 8610 8630 8650 8670 8690 2 4 3 8 10 12 14 16 18
•94 8710 8730 8750 8770 8790 8810 8831 8851 8872 8892 2 4 6 8 10 12 14 16 18
95 8913 8933 8954 8974 8995 9016 9036 9057 9078 9099 2 4 6 8 10 12 15 17 19

96 9120 9141 9162 9183 9204 9226 9247 9268 9290 9311 2 4 6 8 11 13 15 17 19
•87 9333 9354 9376 9397 9419 9441 9462 9484 0506 9528 2 4 7 9 11 13 15 17 20
98 9550 9572 9594 SGI 8 S638 9661 9683 9706 0727 9750 2 4 7 9 11 13 16 18 20
•99 9772 9795 9817 9840 9863 9886 9908 9931 6954 9977 2 5 7 9 11 14 16 18 20
CHAPTER-IX
ANALYSIS OF TIME SERIES

Introduction :
One of the most important tasks before economists and businessmen
these days is to make estimates for the future. For example, an economist
is interested in knowing the likely population in the coming year so that
there can be proper planning to meet situations like food supply, job
opportunities, etc. Similarly, a businessman is interested in knowing what
would be his sales for the coming year so that he could adjust his
production accordingly. In this connection, we collect figures at specified
intervals of times usually at equal intervals and the same is known as time
series. For example, if we observe production, population, sales, imports,
exports, etc., at different points of time say over the last 10 or 15 years, the
set of observations formed shall constitute time series. The analysis of
such figures is called the Analysis of Time Series. Time series analysis is
done primarily for the purpose of making forecasts for future and also for
the purpose of evaluating past performances.

Definition
According to Ya-Lun-Chou “A time series may be defined as a
collection of magnitudes belonging to different time periods, of some
variable or composite of variables, such as production of steel, per capita
income, gross national product, price of tobacco or index of industrial
production”.

According to Croxton and Cowden “A time series consists of data


arranged chronologically”.

Utility of Time Series Analysis


The Time series analysis is useful in every sphere. It is very
important in economics, business, state administration, planning, science,
sociology, biology, research work, etc., because of the following reasons :
(i) It helps in understanding past behaviour and it will help in
predicting the future behaviour.
(ii) It helps in planning and forecasting. Future cannot be planned
without past. The time series analysis helps in planning future
operations.
(iii) It helps in evaluating current accomplishments. The actual
performance can be compared with the expected performance
and the causes of deviation analysed.
A-2

(iv) It helps in making comparative studies. Once the data are


arranged chronologically between one time period and another is
facilitated and important conclusions drawn therefrom.

The above points indicate the utility of time series analysis. However
like all other statistical tools, the time series analysis also gives us only
approximate indicators.

Components of Time Series


There are a large number of forces affecting time series. As a result,
there are fluctuations of time series. There are four basic types of
variations and these are called the components or elements of time series.
They a r e :

1. Secular Trend or long term movements (T)


2. Seasonal variations (S).
3. Cyclical variations (C).
4. Irregular variations (I).

Symbolically Y = T + S + C + I. It is called the additive model.


Where Y = result of four elements. Another approach is to treat each
observation of a series as the multiplication of these components i.e., Y =
T x S x C x I.

1. Secular T rend : The word secular is derived from the Latin word
‘Saeculum’ which means a generation or age. Secular trend, is also known
as a long term trend which requires 15 or 20 years. If we study various
types of economic and business data, we can observe that there is an
upward or downward trend in time series. For e.g., population, price,
income, etc., are rising, whereas death rate, purchasing power of money
are falling. The former is an upward trend and the latter is a downward
trend.

When we plot the time series values on a graph and if it forms a


straight line, then it is called the Straight Line Trend or Linear Trend. On
the other hand, when we plot the time series values on a graph and if it
forms a curve or a non-linear one, then it is called Non-linear or
Curvilinear Trend.

2. Seasonal variation : A variation which occurs with some degree of


regularity within a specific period of one year or shorter is called seasonal
A-3

variation. There are weekly, monthly or quarterly variations also. Business


;md economic activities vary during some seasons in an year. The seasonal
variation may occur due to climate and weather conditions, customs,
traditions and habits.

Seasonal variation is useful to businessmen, sales managers,


agriculturists, producers etc.

3. Cyclical variation : The term ‘cycle’ refers to the recurrent variations


in time series that usually last longer than a year and are regular.

Most of the time series relating to economics and business show


some kind of cyclical variation. Cyclical fluctuations are long-term
movements that represent consistently recurring rises and declines in
activity. There are four phases in a business cycle. They are prosperity
(boom), recession, depression and recovery.

The study of cyclical variation is very useful to businessmen and


economists for framing suitable policies and stabilising the level of
business activities.

4. Irreg u lar variation : Irregular fluctuations are caused by factors like


wars, earth quakes, lockouts, floods, famines, etc., which are unpredictable
in nature. By their very nature these movements are very irregular and
unpredictable.

MEASUREMENT O F TREND
The various methods by which trend values can be determined are :
(A) Freehand or Graphic method
(B) Semi-average method
(C) Moving average method
(D) Method of Least squares

(A) Freehand o r G raphic M ethod : Firstly all the values of a time series
are plotted on a graph paper, certain up and down movements are noted.
After this a freehand smoothed curve is drawn through these points in such
a way that the curve represents the general tendency of the data. In the
process of drawing a smooth curve the seasonal, cyclical and irregular
variations are eliminated.
A-4

Merits :
(i) This is the simplest method of measuring trend.
(ii) This is adaptable and flexible, and it can be used to describe all
types of trend i.e., linear and non-linear.

D em erits :
(i) The main demerit of this method is that it is highly subjective
Different persons will get different lines or curves.
(ii) Because of the subjective nature of the free hand trend curve, it
will be dangerous to use it for forecasting or making predictions
(iii) Dependable freehand curves can be drawn only by experienced
persons not by novices (new persons).

(B) Semi-average method : In this method, the whole time series data is
divided into two equal parts. However, in case of odd number of years, the
two equal parts are obtained on omitting the values for the middle period
Having divided the given series into two equal parts, an arithmetic mean of
each part is obtained. These means are called Semi-averages. Then these
semi-averages are plotted as points against the middle point of the
respective time periods covered by each part. The line joining these points
gives the straight line trend fitting the given data The line can be extended
downward or upward to get intermediate values or to predict future values
I '- t.'- h f •> !,»■,, ». i t A

Merits :
(i) This method is simple and easier to understand than the moving
average and the method of least squares.
(ii) This method is an objective method of measuring trend as
everyone who applies the method is bound to get the same result.
(iii) As the line can be extended both ways, we can get the
intermediate values and predict the future values.

D em erits :
(i) This method assumes the presence of linear trend which may not
exist.
(ii) It is affected by the limitations of arithmetic mean.

(C ) Moving Average M ethod : According to this method, first the


period of moving average is selected which may be either 3 yearly moving
average, 5-yearly moving average, 8 -yearly moving average etc., The
period of moving average is to be decided in the light of the length of the
cycle. Moving totals of the given data are then obtained. Each of these
A-5

iinures is divided by the period of moving average w h ic h gives the trend


values.

Merits:
(i) This method is simple and easy to understand as compared to the
method of least squares.
(ii) This method is more flexible than other m ethods.
(iii) This method is not only used for the m easurem ent of trend, but
also for the measurement of seasonal, cyclical and irregular
fluctuations.

Demerits:
(i) In this method we cannot get the trend v alu es for all the given
observations. We have to forego the tre n d values for some
observations at both the extremes depending on the period of the
moving average.
(ii) Since the moving average is not represented by a mathematical
function, this method cannot be used in forecasting or predicting
which is one of the main objectives of trend analysis.
(iii) Utmost care lias to be exercised in selecting the period of
moving average. No hard and fast rules are available for the
choice of the period and one has to use his ow n judgement.

(D) Method of Least Square : By the help of L east Square, a straight


line trend can be fitted to the given time series of data, it is a mathematical,
ns well as, an analytical method. With its help, economic and business time
series data can be fitted and this helps in forecasting and predicting The
trend line is called the line of best fit.

The equation of the straight line or linear trend is ;


Yc = a + bx
Where ‘Yc’ represents computed trend value
‘a’ is the value of Y variable when X = 0
‘b ' represents the slope of the straight line
‘X ’ is the independent variable represents time.

The value< of ‘a’ and ‘b ’ is obtained by solving the following two


normal equations.

LY = Na + bXX
EXY = aXX + bSX 2
A -6

If deviations are taken from the middle period so that EX = 0 then the
values of a and b can be obtained from the following formula :

EY EXY
a = ----- and b = --------
N EX 2

It should be noted that in case of odd number of years, when the


deviations are taken from the middle year. EX would always be zero.
However, in case of even years also EX will be zero if the X origin is
placed mid way between the two middle years. If EX is not zero the
values of ‘a ’ and ‘b ’ will be obtained by directly solving the two
equations.

The following steps are worthnoting in this context:


(i) X represents time. Find out the time deviation from the middle
year and add up the values. The total time deviations will be zero
(EX=0).
(ii) Multiply the time deviation by the given value (Y). Add up the
products (EXY).
(iii) Square up the time deviations and add them up (EX2).
EY EXY
iv) Since EX = 0, a = -------, b = --------
N EX 2
v) Find out the equation of the straight line trend Yc = a + bX.

M erits :
(i) This is the most objective method, as trend values are calculated
with the help of a mathematical formula.
(ii) The line obtained by this method is called the line of best fit,
because sum of the deviations of the actual values and trend
values is zero and the sum of the squares of these deviations is
minimum.
(iii) By this method we can estimate or predict the values of the
variable for the future or intermediate periods of the given series.
(iv) Trend values for all the years ar e obtained.

Demerits :
(i) It is difficult for a non-mathematical person to calculate and it
will take more time to calculate than the other methods.
A-7

(ii) Future forecasts made by this method are based only on trend
values. The seasonal, cyclical or irregular variation are
completely ignored.
(iii) If we add a new observatibn, then the calculations must be done
once again.

The following illustrations will clearly explain the various methods by


which trend values can be determined.

Illustration 1 : Fit a trend line to the following data by the free-hand


method.
Year 1972 1973 1974 1975 1976 1977 1978 1979 1980 1981 1982
V a l­ 64 82 97 71 78 112 115 131 88 100 146
ues

Solution
Drawing of a free hand curve

values

l<Iii
From the above graph the trend values for any year can be read on the
trend line.

Illustration 2 : Draw a trend line by the method of semi averages from


the following data.

Year 1992 1993 1994 1995 1996 1997 1998 1999


Sales
( ’0 0 0 10 0 108 1 1 0 90 106 108 1 1 0 1 1 2

units)

Solution : Computation of Trend values by the method of Semi-Averages

Year Sales Semi-Totals Semi-average


1992 lOO-i
1993 108 408
1994 1 1 0 408 -------= 10 2
4
1995 90 ^
1996 106
1997 108 436
1998 1 1 0 436 -------= 109
4
1999 1 1 2 —

These two figures, namely 102 and 109, shall be plotted at the middle
of their respective period, i.e,. at the middle of the years 1993 and 1994
and 1997 and 1998. By joining these two points we get a trend line which
describes the given data.

Illustration 3 : Fit a trend line for the following data by the method of
semi-averages.

Year 1990 1991 1992 1993 1994 1995 1996


Production ■
(in thousands of 10 0 107 108 104 1 1 0 114 1 1 2

tons)
f

A-9

Solution : Computation of T rend values by the method of semi-averages.


Year_________ Production Semi-Totals Semi-average
1990 lO o S
1991 107 315
= 105
1992 __________ 108 —
1993 _____ 104
1994 1 1 0 -"
336
1995 114 336 =112
1996 3
1 1 2 - 1

Since seven years are given the middle year shall be left out and an
average of first three years is 105 and the last three years is 112. These two
ixiints 105 and 112 which shall be plotted corresponding to their respective
middle years i.e., 1991 and 1995. By joining these two points we shall
obtain the required trend line.

Illustration 4 :

The following figures relate to the profits of a commercial concern


for 8 years.
Year 1976 1977 1978 1979 1980 1981 1982 1983
(I’rofil)
15,420 14,470 15,520 21,020 34,670
Rs.) 26,120 31,950 35,370

Calculate three yearly moving average.

Solution : Calculation of three yearly moving aver.w


Year Profits 3 yearly 3 yearly moving averages
moving total (Trend of profits) Rs.
1976 15,420
1977 14,470 45,410 15,137
1978 15,520 51,010 17,003
1979 2 1 , 0 2 0 62,660 20,887
1980 26,120 79,090 26,363
1981 31,950 93,440 31,147
1982 35,370 1,01,990 33,997
1983 34,670
0

From the above is loving average of the data given


trend line. iE>
J

B
Illustrate 2 L . 1995 1996 1997 1998
1 <T>
the fo ilii


o
in 1I 5. 58 62 61 65

m w jm h o h . v y o iv u ic r u u n u i i u u i J C O U J itlO V lI lg B V 6 IB § 6

(1 ) (2 ) (3) (4) (5) = (4) 8

Year Production Totals of Totals of 4 yearly


of four from two from centred
wheat Col. (2) Co. (3) moving
averages
1991 51
1992 53
215
1993 56 437 54.625
2 2 2

1994 55 453 56.625


231
1995 58 467 58.375
236
1996 62 482 60.250
246
1997 61
1998 65

In the above illustration we have first taken four yearly totals and of
these total we have again taken two-yearly totals and divided these totals
by 8 to get the trend values.

Illustration 6 :
The Production of Iron during 1977-1983 is given below :
Year 1977 1978 1979 1980 1981 1982 1983
Production
48 50 58 52 45 41 49
(in lakhs of tons)

Fit a straight line trend to these figures.


A-l 1

Solution :
Fitting the Straight Line Trend
Production Trend
Year X XY X2
Y Values (Yc)
1977 48 -3 -144 9 52
1978, 50 -2 -100 4 51
1979, 58 -1 -58 1 50
1980. 52 0 0 0 49
1981 45 +1 +45 1 48
1982 41 +2 +82 4 47
1983 49 +3 +147 9 46
><

N=7 £ Y = 343 EXY = -2 8 EX2 = 28 Yc = 343


II
o

The Equation of straight line trend is Yc = a + bx

£Y 343
Since XX = 0; a = ---- = ------- = 49,
N 7

IX Y -28
b= ------- = ------- = - l
IX 2 28

Hence the equation of Straight Line Trend


Yc = 49 - IX

Trend values are computed as follows :


When X is -3 , Yc = 49 + (-1 x -3 ) = 52
When X is -2 , Yc = 49 + (-1 x -2 ) = 51
When X i s -1 , Yc = 49 + ( - l x -1 ) = 5 0
When X is 0, Yc = 49 + (-1 x 0) = 4 9
When X i s +1, Yc = 49 + ( - l x 1) = 4 8
When X is +2, Yc = 49 + (-1 x 2) = 47
When X i s +3, Yc = 49 + ( - l x 3 ) = 4 6

Illustration 7 : Fit a straight line trend by the method of least squares to


the following data. Assuming that same rate of change continues. What
would be the predicted earnings for the year 1987 ? * ______ ______
Year 1978 1979 1980 1981 1982 1983 1984 1985
Earnings
(in lakhs 38 40 65 72 79 60 87 95
Rs.)
A-12

Solution :
Fitting Straight line Trend by the Method of Least Square
Year Earning Deviation Deviation XY X2
0 0 from x Multiplied
1981.5 by 2 (X)
1978 38 -3.5 -7 -266 49
1979 40 -2.5 -5 - 2 0 0 25
1980 65 -1.5 -3 -195 9
1981 72 -0.5 - 1 -72 1

1982 79 +0.5 + 1 +79 1

1983 60 + 1.5 +3 +180 9


1984 87 +2.5 +5 +435 25
1985 95 +3.5 +7 +665 49
N= 8

w
2XY= 626 EX2 = 168

X
o
EY = 536

11
Yc = a + bX

2Y 536
N 8

IX Y 626
il
%<
w

168

Y = 67 + 3.72X
For 1987 X will be +11
X = 67+ 3.72(11)
= 67 + 40.92= 107.92
Estimated earnings for 1987
= 107.92

Illustration 8 : Below are given figures of production (in thousand


quintals) of a sugar factory______ ______ ______ ______
Year 1975 1977 1978 1979 1980 1981 1984
(Production qtls.) 77 88 94 85 91 98 90

(i) Fit a straight line by the ‘least squares’ method, and tabulate the
trend values.
A-13

(ii) Eliminate the trend. What components of the time series are thus
left over ?
(iii) What is the monthly increase in the production of sugar ?

Solution : Fitting of Straight Line by Method of Least Squares


Year Production Deviation XY Trend
(‘000 Qtls) from 1979 Values Yc
Y X
1975 77 -4 -308 16 83
1977 88 - 2 -176 4 86

1978 94 - 1 -94 1 87
1979 85 0 0 0 89
1980 91 +1 +91 1 90
1981 98 + 2 + 196 4 92
1984 90 +5 +450 25 96
N=7 EY = 623 EX = +1 ZXY= 159 ' EX2 = 5i EYC= 623
The equation of the straight line trend is
Yc = a + bX
Since EX is not zero we have to solve the following two normal
equations
EY = Na + b E X
EXY = a2X + bZX 2
623 = 7a + b ... (i)
159 = a + 5 1 b ...(ii)
Multiplying eq. (ii), by 7
623 = 7a + b
1113 = 7a + 357 b

-490 = - 356b
356b = 490
490
b = -------- = 1.376 or 1.38
356

Substituting the value of (b) in eq. (i)


7a + 1.38 = 623
7a = 6 2 3 - 1.38 = 621.62
621.62
a = = 88.803
7
A-12

Solution :
_______ Fitting Straight line Trend by the Method of Least Square
Year Earning Deviation Deviation XY X2
0 0 from x Multiplied
1981.5 by 2(X)
1978 38 -3.5 -7 -266 49
1979 40 -2.5 -5 - 2 0 0 25
1980 65 -1.5 -3 -195 9
1981 72 -0.5 -1 -72 1

1982 79 +0.5 + 1 +79 1


1983 60 + 1.5 +3 +180 9
1984 87 +2.5 +5 +435 25
1985 95 +3.5 +7 +665 49

W
N= 8 EXY= 626 EX2 = 168

X
>*

o
O

II
II

Yc = a + bX

EY 536
a = ------- ------ =67
N 8

exy 626
b = ------- ------ =3.72
EX2 168

Y = 67 + 3.72X
For 1987 X will b e +11
X = 67+ 3.72(11)
= 67 + 40.92= 107.92
Estimated earnings for 1987
= 107.92

Illustration 8 : Below are given figures of production (in thousand


quintals) of a sugar factory.______ ______ ______ ______ _____________
Year 1975 1977 1978 1979 1980 1981 1984
(Production qtls.) 77 88 94 85 91 98 90

(i) Fit a straight line by the ‘least squares’ method, and tabulate the
trend values.
A-13

(ii) Eliminate the trend What components of the time series are thus
left over ?
(iii) What is the monthly increase in the production of sugar ?
^ . . 't : (. /
Solution : Fitting of Straight Line by Method of Least Squares
Year Production Deviation XY X2 Trend
(‘000 Qtls) from 1979 Values Yc
Y X
1975 77 -4 -308 16 83
1977 88 - 2 -176 4 86

1978 94 - 1 -94 1 87
1979 85 0 0 0 89
1980 91 + 1 +91 1 90
1981 98 + 2 +196 4 92
1984 90 +5 +450 25 96
M

N=7 XX = +1
Lfi

XY = 623
II

XXY= 159 EYC= 623

The equation of the straight line trend is


Yc = a + bX
Since XX is not zero we have to solve the following two normal
equations
XY - N a + b XX
XXY = aXX + bXX2
623 = 7a + b ...(i)
159 = a + 5 1 b ...(ii)
Multiplying eq. (ii), by 7
623 = 7a + b
1113 = 7a + 357 b

-490 = - 356b
356b = 490
490
b = —------ = 1.376 or 1.38
356

Substituting the value of (b) in eq. (i)


7a + 1.38 = 623
7a = 6 2 3 - 1.38 = 621.62
621.62
a = ----------- = 88.803
7
A -14

Hence Y = 88.803 + 1.38X


When X = -4 , Y shall be 88.803 + 1.38(—4) = 88.803-5.52 = 83.283
When X = -2 , Y shall be 88.803 + 1.38 (-2) = 88.803-2.76 = 86.043
When X = -1 , Y shall be 88.803 + 1.38 (-1) = 88.803-1.38 = 87.423
When X = 0, Y Shall be 88.803
When X = +1, Y shall be 88.803 + 1.38 = 90.183
When X = +2, Y shall be 88.803 + 1.38(2) = 88.803 + 2.76 = 91.563
When X = +5, Y shall be 88.803 + 1.38(5) = 88.803 + 6.9 = 95.703

(ii) After eliminating the trend we are left with cyclical and irregular
variations.
(iii) The monthly increase in the production of sugar is
b/12 = 1.38/12 = 0.115 thousands qtl.

Illustration 9 : The following table gives the sterling assets of the R.B.I
in crores of rupees :
(a) Fit a straight line trend
(b) Show the trend on the graph
Y ear 1976-77 1977-78 1978-79 1979-80 1980-81 1981-82
Assets 83 92 71 90 169 191
Also estimate the figures for 1986-87.

Solution :______ (a) Fitting of Straight line trend


T im e
deviation
Trend
Y ear Y from the X2 XY
values Yc
m id y e a r
1979.5 (X)
1976-77 83 -2 .5 6.25 -2 0 7 .5 60
1977-78 92 -1 .5 2.25 -1 3 8 .0 82
1978-79 71 -0 .5 0.25 -3 5 .5 105
1979-80 90 +0.5 0.25 +45.0 127
1980-81 169 +1.5 2.25 +253.5 150
1981-82 191 +2.5 6.25 +477.5 172

XX2 = ZX Y =
N= 6 Z Y = 696 zx=o 17.50 395
ZY C = 696
A-15

Yc = a + bX

IY 696
a = = 116
N 6

IX Y 395
22.6
EX 2 17.50

Yc = 116+ 22.6X

.-. Trend value for 1976-77 = 1 1 6 + (22.6 x - 2 .5 ) = 116-56 = 60

The other values can be obtained by adding the value of ‘b ’ to the


preceding value.
Estimation of the Sterling assets for 1986-87
Time deviation from the middle year = +7.5
Trend value for 1986-87 = 116+ (22.6 X 7.5)
= 116+ 169.50
= Rs. 285.50 crores.

Shifting of Origin :
When we compute the trend value, the origin is generally-taken in the
middle year of the series. But some times to compare the trend values or to
convert a trend equation from an annual to half-yearly or monthly basis, it
is necessary to change the origin of the trend equation to some other point
in the equation. We shall explain the technique by an example.
Yc = a + bX
For example if we shift the origin from 1985 to 1990 then the new
trend equation on changing the value of X to X+5 would be
Yc = a + b (X + 5),

Similarly, if we want to shift the origin to 1982, i.e., 3 years back, the
new trend equation becomes :
Yc = a + b ( X - 3 ) ,
Thus shifting of origin only affects the value of constant ‘a ’ in the
equation, while the slop ‘b’ of the equation remains the same.
T

A-16

The following illustrations would further clarify these points.

Conversion of Annual Trend equation to Monthly Trend Equation :


We can calculate monthly trend equation from annual trend equation
by dividing ‘a ’ by 12 and ‘b ’ by 144. Y = a/ n + b/ i 4 4 X If the time unit of
X in the trend equation represents only 6 months, then ‘b’ is divided by 72,

Illustration 10 : Given the following equation Yc = 210 + 1.5X


The origin is 1995. Time unit is one year shift the origin to 2000.

Solution : The origin has to be shifted from 1995 to 2000 i.e., 5 years
hence. The required equation would be
Yc = 210+ 1.5 (X+5) or
Yc = 210 + 1.5X + 7.5 or
Yc = 217.5 +T.5X

Illustration 11 : The trend equation in a problem is Yc = 150 + 2X :


origin is 1990 ; Time unit one year. Shift the origin to 1987.

Solution : The origin has to be shifted from 1990 to 1987 i.e., 3 years
backward. The required equation would be
Yc = 1 50 + 2 (X-3) or
Yc = 150 + 2 X - 6 or
Yc = 144 + 2X

Illustration 12 : The trend of annual sales of a company is described by


the following equation.
Yc = 50 + 0.8X
Origin 1995; X - unit = 1 year; Y - unit = annual production.
Convert the equation to a monthly trend equation.

Solution : Monthly Trend Equation would be

50 0.8
Yc = ------- + -------X
12 144

Yc = 4.167 + 0.0056X
A-17

MEASUREMENT OF SEASONAL VARIATIONS

Seasonal variations are periodic movements that recur after a period.


It may be weekly, monthly, quarterly, or half yearly. Seasonal variations
occur due to (a) climate and weather conditions and (b) habits, traditions
and customs.

It is very useful to businessmen, producers and customers. Basically


there are two objectives in studying seasonal variations.

(i) to study seasonal variation, and


(ii) to eliminate them.

The following are some of the methods more popularly used for
measuring seasonal variations:

(A ) Method of Simple Averages.


(B) Ratio - to - Trend Method.
(C) Ratio - to - Moving Average Method.
(D) Link relative Method.

(A) Method of simple averages : This is the simplest and easiest


method of calculating a seasonal index. The steps are :
(i) Arrange the data for each month or quarter for all the years.
(ii) Find the totals of each month or quarter.
(iii) Divide each total by the number of years for each data are given.
(iv) tain an average of monthly averages by dividing the total of
>nthly averages by 1 2 .
(v) Taking the average of monthly average as 100, we can get the
seasonal index as follows :

Monthly average for January


Seasonal Index for January = x 1 0 0

Average of monthly averages

M erits and Demerits : This is the simplest and the easiest of all the
methods of measuring seasonality. However, it is not a very good method.
It assumes that there is no trend component in the series. But this is not a
justified assumption.

(B) Ratio to T rend Method : This method is based on the assumption


that the seasonal variation for various seasons are a constant factors
A-18

of the trend value. It is based on the multiplicative model. The steps


are :

(i) Obtain the trend values by the method of least squares.


(ii) Eliminate the trend values. In a multiplicative model the trend is
eliminated by dividing the time series value by the
corresponding trend value and multiplying by 1 0 0 .
This percentage will include seasonal, cyclical and irregular
fluctuations.
(iii) To eliminate the cyclical and irregular movements, the seasonal
figures are averaged with any one of the measures of central
tendency, mean or median. Thus we obtain the indices for
seasonal variation for different seasons.
(iv) These indices are adjusted to a total of 1200 for monthly data or
400 for quarterly data by multiplying each of them by a suitable
factor.

1200 400
= ---------------------------------- or = --------------------------------------
Sum of monthly indices Sum of quarterly indices

Thus we get seasonal Index.

M erits and Demerits : This method gives a more representative value of


seasonal variation than the simple average method. It is simple to calculate
and easy to understand.

The main defect of this method is that it does not follow the actual
data as a 12 month moving average method. So the seasonal index
calculated by this method is more biased.

(C) Ratio to Moving Average M ethod : This is the mostly widely used
method of measuring seasonal variations. The steps are :
(i) Calculate 12 month moving average, (in case of quarterly data,
calculate 4 quarter moving average).
(ii) Express the original values as a percentage of centres moving
average values for all months, i.e.,

Original Value
-------------------------------- x 1 0 0

Moving Average Value


A -19

Thus the ratio to moving average represents the seasonal and


irregular components.

(iii) Arrange these percentages season wise for all the years. By
averaging these percentages for each month, we can eliminate
irregular variations. Mean or Median can be used for averaging

(iv) The sum of these indices should be 1,200 (400) for monthly or
quarterly data. If it is not so, an adjustment is made, i.e.,

1200 400
= — t----------------------------- or = --------------------------------------
Sum of monthly indices Sum of quarterly indices

This expresses the preliminary index as a percentage of their


arithmetic mean. These are the ratio to moving average seasonal
indices.

Merits and limitations : This method is widely used. It is the most


satisfactory method. It is flexible. If the cyclical fluctuations are regular in
periodicity, this will give true seasonal indices. One obvious limitation of
this method is that there are no seasonal indices at the beginning and at the
end of the series.

(D) Link Relative Method : This method is also known as the Pearson’s
Method. The steps are :
( i f Calculate the seasonal link relatives for each seasonal figures by
V the following method :

Current Season’s Figure


Link Relative = ------------------------------------- x 100
Previous Season’s Figure

(ii) Calculate the average of link relative for each season. Arithmetic
Mean or Median can be used for this purpose.
(iii) Convert the link relative (L.R.) into chain relatives (C.R.) on the
basis of the first season. The formula is

L.R of the season x C.R of previous season


Chain Relative (C.R) =
100
A-20

(iv) Calculate the chain relative of the first season on the basis of the
last season.
(v) For correction, the chain relative of the first season calculated by
the first method is deducted from the chain relative of the firiit
season calculated by the second method. The difference i«
divided by the number of seasons. The resulting figure
multiplied by 1, 2, 3 are deducted respectively from the chain
relatives of the 2nd, 3rd, 4th seasons. There are corrected chain
relatives.
(vi) The seasonal indices are available when we express the corrected
chain relatives as percentages of their respective averages.

M erits and Limitations : Though this method, is not so easy to


understand as the moving average method, the actual computations
involved in this method are much less extensive than the Ratio to moving
Average or Ratio to Trend method.

In case of Ratio to moving Average method there are no seasonal


indices in the beginning and at the end of the series, while is case of link
relative method only the first link is not there. Thus, link relative method
uses the data more completely.

W hich M ethod to Use : In general it may be said that because of


theoretical and practical advantages, Ratio to Moving Average method
should be preferred to other methods.

Deseasonalisation of Data : Elimination of the seasonal effects from the


given values is known as deseasonalisation of the data. It helps us to adjust
the given time series for seasonal variations, thus leaving us with trend,
cyclical variations and irregular variations.

Deseasonalisation is very helpful to business executives for planning


future production programmes, for forecasting and for managerial control.
It is also helpful in the proper interpretation of data

In a multiplication model where Y = T x S x C x I , the deseasonalised


values are obtained by dividing the given values by corresponding indices
of seasonal variations. Thus in this model

Y TSCI
Deseasonalised values = ------- or --------- = TCI
S S
A-21

In a n a d d itiv e m o d e l w h e r e Y = T + S + C + I, d e s e a so n a lisa tio n c a n


1x5 d o n e b y s u b s tra c tin g th e se a s o n a l v a ria tio n s fro m th e v a lu e s o f Y . T h u s
a d e se a o n a lise d se rie s w o u ld b e Y - S = T + C + I.

W e n o w g iv e b e lo w so m e e x a m p le s to illu stra te th e c a lc u la tio n o f


se aso n al indices.

I llu s tr a tio n 13 : C a lc u la te th e se aso n a l in d e x fo r th e fo llo w in g d a ta b y


usin g th e a v e ra g e m e t h o d . ______________ _______________ ____________
Y ear 1st Q u a rte r 2nd Q u a rte r 3rd Q u a rte r 4 th Q u a rte r
1974 72 68 80 70
1975 76 70 82 74-
1976 74 66 84 80
1977 76 74 84 78
1978 78 74 86 82

S o lu tio n :
C o m p u tatio n , o f S e a so n a l In d ice s
Y ear l “ Q u a rte r 2 Kd Q u a rte r 3rd Q u a rte r 4 th Q u a rte r
1974 72 68 80 70
1975 76 70 82 74
1976 74 66 84 80
1977 76 74 84 78
1978 78 74 86 82
T o ta l 376 352 416 384
A v erag e 7 5 .2 7 0 .4 83.2 7 6 .8
S easo n al 9 8 .4 3 92 .1 5 108.90 10 0 .5 2
In d ex

S e aso n al In d ex lias b e e n c a lc u la te d a s fo llo w s :

7 5 .2 + 7 0 .4 + 83.2 + 76.8
Average of quarterly average = ----------------------------------
4

3 0 5 .6
= 76.4
4
A-22

7 5 .2
S e a so n a l In d ex fo r 1st Q u a rte r = x 100 l= 98 .4 3
7 6 .4

7 0 .4
S e aso n al Index fo r 2nd Q u a rte r = x 100 = 92 .1 5
7 6 .4

8 3 .2
S e aso n al In d ex fo r 31,1 Q u a rte r x 1 0 0 = 108.90
7 6 .4
7 6 .8 . V,ifl
S easo n al In d ex fo r 4 th Q u a rte r = x 1 0 0 = 100.52 m
7 6 .4

I llu s tr a tio n 14 : F in d se a s o n a l v a r ia tio n b y th e ra tio -to -tre n d m e th o d fro m


th e d a ta g iv e n b e l o w :_______
Y ear 1st Q u a rte r 2 nd Q u a rte r 3rd Q u a rte r 4 * Q u a rte r
1990 ->r
30 40 35 35
1991 34 52 50 44
1992 40 58 54 48
1993 54 76 68 62
1994 80 92 86 82

S o lu tio n : F o r d e te rm in in g se a so n a l v a ria tio n b y ra tio -to -tre n d m e th o d ,


firs t w e w ill d e te rm in e th e tr e n d fo r y e a rly d a ta a n d th e n c o n v e rt it to
q u a rte rly data.

C a lc u la tin g tr e n d b y th e m e th o d o f le a s t sq u a re s
Y ear Y e a rly Y e a rly D e v ia tio n s XY X2 T re n d
to tal a v e ra g e f ro m m id v a lu e s
Y year X Yc
1990 140 35 -2 -7 0 4 32
1991 180 45 -1 -4 5 1 44
1992 200 50 0 0 0 56
1993 260 65 +1 +65 1 68
1994 340 85 +2 + 170 4 80
E Y -2 8 0 I X Y = 1 2 0 S X 2= 1 0

T h e e q u a tio n o f th e stra ig h t lin e tr e n d is Y = a+ b X .


A-23

EY 280
a= ------- = ------- = 56
N 5

EXY 120
b = ------- - = ------- = 1 2
EX2 10

12
Q u a rte rly in c re m e n t = ------- = 3
4
C a lc u la tio n o f Q u a rte rly T re n d V alu es. C o n s id e r 1986. T re n d v a lu e fo r
th e m id d le q u a rte r, i.e., h a lf o f 2nd a n d h a lf a n d 3 rd is 32.
3
Q u a rte rly in c re m e n t is 3. So th e tre n d v a lu e o f 2nd q u a rte r is 32 - —
2
3
i.e., 3 0 .5 , a n d fo r 3rd q u a rte r is 3 2 H------ , i.e ., 3 3 .5 . T r e n d v a lu e fo r
2
th e 1st q u a rte r is 3 0 .5 -3 , i.e., 2 7 .5 a n d o f 4 th q u a rte r is 3 3 .5 + 3 , i.e., 36.5.

S im ila rly tr e n d v a lu e s o f th e v a rio u s q u a rte rs o f o th e r y e a rs c a n be


c a lc u la te d . T h e s e v a lu e s are ta b u la te d b e l o w :

T re n d v a lu e s
Y ear 1st Q u a rte r 2 nd Q u a rte r 3rd Q u a rte r 4 th Q u a rte r
1990 27.5 3 0 .5 3 3 .5 3 6 .5
1991 39.5 4 2 .5 4 5 .5 4 8 .5
1992 51.5 54.5 5 7 .5 60.5
1993 63.5 6 6 .5 6 9 .5 7 2 .5
1994 75.5 7 8 .8 8 1 .5 85.5

T h e g iv e n v a lu e s o f th e tim e se rie s w ill n o w b e e x p re s s e d a s p e rc e n ta g e s


o f th e c o rre s p o n d in g tre n d v a lu e s g iv e n ab o v e. T h u s fo r th e 1st q u a rte r
1 9 9 0 , th is p e rc e n ta g e w o u ld b e /27.5 x 100 = 1 0 9 .1 , fo r th e se c o n d

q u a rte r it w o u ld b e / 30.5 x 100 = 131.1 o th e r v a lu e s c a n b e c a lc u la te d


lik e w ise . T h e s e v a lu e s are g iv e n in th e f o llo w in g tab le.

G iv e n Q u a rte rly V a lu e s a s % o f th e tre n d v a lu e s


Y ear l sl Q u a rte r 2nd Q u a rte r 3rd Q u a rte r 4 th Q u a rte r
1990 109.1 131.1 107.5 93.1
1991 86.1 '* 12 2 .4 1 0 9 .9 9 0 .7
A -24

1992 7 7 .7 106.4 9 3 .9 79.3


1993 8 5 .0 114.3 9 7 .8 85.5
1994 10 6 .0 117.6 105.5 9 7 .0
T o ta l 4 6 3 .9 5 9 1 .8 5 1 4 .6 4 4 5 .6
A v e ra g e 9 2 .7 8 118.26 10 2 .9 2 8 9 .1 2
S e a so n a l
In d e x 9 2 .0 7 117.4 102.1 8 8 .4
A d ju ste d
T o ta l o f a v e ra g e s = 9 2 .7 8 + 1 1 8 .2 6 + 1 0 2 .9 2 + 8 9 .1 2 = 4 0 3 .0 8

S in c e th e to ta l is m o re th a n 4 0 0 a n a d ju stm e n t is m a d e by m u ltip ly in g each


400 /
a v e ra g e b y /403.08 a n d fin a l in d ic es a re o b ta in ed .

T h u s th e a d ju ste d se a so n a l in d e x fo r th e firs t q u a rte r w o u ld be

92.78x400
= -------- ---------- =92.07
403.08
S im ila rly o th e r c a lc u la tio n s h a v e b e e d o n e .

I l l u s t r a t i o n 15 : F in d th e se a so n a l in d e x fro m th e fo llo w in g ta b le b y ratio


to m o v in g a v e ra g e m e th o d .____________________________________ ___________
S e a so n s 1970 1971 1972 1973 1974
I Q u a rte r 40 42 41 45 44
II Q u a rte r 35 37 35 36 38
III Q u a rte r 38 39 38 36 38
IV Q u a rte r 40 38 42 41 42

S o lu tio n :
R a tio to m o v in g a v e ra g e m e th o d to d e te rm in e S e a so n a l Index.
Y ear Q uarter Given 4 figure 2, figure Moving Given figures
"
values moving moving average as percentage
totals totals centered o f moving
= (5+8) average =

[ 3/6 x 100 ]

1 2 3 4 5 6 7
1970 1 40
II 35
153 1. '
A-25

C a lc u la tio n o f S e a s o n a l In d e x
Y ear I Q u a rte r II III IV
Q u a rte r Q u a rte r Q u a rte r
1970 - - 9 8 .7 0 1 0 2 .5 6
1971 10 6 .6 0 9 4 .2 7 100.31 9 8 .7 0
1972 107 .5 6 9 0 .9 0 9 6 .2 0 10 4 .6 8
1973 112 .5 0 9 0 .8 6 9 1 .4 2 1 0 3 .8 0
1974 1 1 0 .0 0 9 4 .1 3 - - T o ta l
A-26

T o ta l 4 3 6 .6 6 3 7 0 .1 6 38 6 ,6 3 4 0 9 .7 4 1603.19
A v e ra g e (S .I.) 10 9 .1 6 9 2 .5 4 96 .6 5 102.43 4 0 0 .7 8
A d ju ste d 108.95 9 2 .3 6 9 6 .4 6 102.23 4 0 0 .0 0
S e a so n a l
In d e x
T h e to ta l o f th e a v e ra g e s o f in d ic e s is 4 0 0 .7 8 . T h e re fo re the
,,, 400/
c o rre c tio n fa c to r w o u ld b e /4 0 0 78-

E a c h se a so n a l a v e ra g e w ill b e m u ltip lie d b y th e c o rre c tio n fa c to r to


g e t th e a d ju ste d se aso n a l in d ex . T h u s fo r th e firs t q u a rte r, th e ad ju sted
se a so n a l in d e x w o u ld be.
1 0 9 .1 6 x 4 0 0
= ---------------------- = 108.95
4 0 0 .7 8

9 2 .5 4 x 4 0 0
S e c o n d Q u a rte r = ---------------------- = 92 .3 6
4 0 0 .7 8

96 .6 5 x 4 0 0
T h ir d Q u a rte r = ---------------------- = 9 6 .4 6
4 0 0 .7 8

102.43 x 4 0 0
F o u rth Q u a rte r = ---------------------- = 102.23
4 0 0 .7 8

I l l u s t r a t i o n 16 : C a lc u la te th e se a so n a l in d ic e s b y th e lin k re la tiv e m e th o d
f o r th e fo llo w in g d a t a :

O u tp u t o f R ic e in M illio n T o n n e s
Y ear 1st Q u a rte r 2 nd Q u a rte r 3rd Q u a rte r 4 th Q u a rte r
1995 6 .0 6.5 6 .2 6 .9
1996 6.2 6.8 6.5 6.8
1997 6.5 7 .0 6 .4 6 .2
1998 7.0 7.5 6.8 6.7
1999 7 .2 8 .0 7 .0 7.8

S o lu tio n : C o m p u ta tio n o f S e a s o n a l In d ex b y L in k R e la tiv e M e th o d


Y ear L in k R e la tiv es
1st Q u a rte r 2 nd Q u a rte r* " 3 rd Q u a rte r 4 th Q u a rte r
A-27

1995 108.3 9 5 .4 111.3


1996 8 9 .9 109.7 9 5 .6 10 4 .6
1997 9 5 .6 107.7 9 1 .4 9 6 .5
1998 1 1 2 .9 107.1 9 0 .7 98.5
1999 107,5 111.1 87.5 11 1 .4
T o ta l o f
L in k 4 0 5 .9 5 4 3 .9 4 6 0 .6 5 2 2 .7
re la tiv e s
A rith m e tic
10 1 .4 8 108.78 9 2 .1 2 10 4 .5 4
M ean
C h a in
100 108.78 100.21 1 0 4 .7 6
rela tiv e s
A d ju ste d
C h a in 100 107.2 9 7 .0 5 10 0 .0 2
re la tiv e s
S easo n al
9 8 .9 4 10 6 .0 7 9 6 .0 2 9 8 .9 6
In d ex

L in k re la tiv e fo r a n y q u a rte r

C u rre n t S e a s o n ’s fig u re
= ----------------------------------------- x 100
P re v io u s se a s o n ’s fig u re

L in k re la tiv e fo r 2ad q u a rte r o f firs t y e a r i.e ., 1995

= — x 1 0 0 = 108.3
6.0

L in k re la tiv e fo r 3rd q u a rte r

6.2
= ------ x 1 0 0 = 9 5 .4
6.5

Chain Relatives

C h a in re la tiv e fo r 1st q u a rte r = 1 0 0 (b y a s su m p tio n )

L in k re la tiv e fo r II x 100
C.R. fo r 2nd q u a rte r =
100
A-28

10 8 .7 8 x 100

100
= 10 8 .7 8

L in k re la tiv e fo r III x C .R . fo r II
C .R . fo r 3 rd q u a rte r =
100

9 2 .1 2 x 108.78
= 100.21
100

1 0 4 .5 4 x 100.21
C .R . fo r 4 th q u a rte r = = 104.76
100

T h e c h a in -re la tiv e o f th e firs t q u a rte r, o n th e b a sis o f 4 th q u arter.

L .R . o f th e I s* q u a rte r x C .R . o f th e 4 * q u a rte r 101.48 x 104.76

100 100

= 106.31

D iffe re n c e b e tw e e n c h a in -re la tiv e s o f th e 1st q u a rte r n o w c a lc u la te d an<


p re v io u s ly a s s u m e d = 106.31 - 100 = 6.31.

6.31
D iffe re n c e p e r q u a rte r = -------- = 1 .5 8
4

T h u s a d ju ste d C .R . fo r 1st q u a rte r = 100


F o r 2 nd q u a rte r = 1 0 8 . 7 8 - ( 1 . 5 8 ) = 107.2
F o r 3rd q u a rte r = 100.21 - (2 X 1 .58) = 97.05
F o r 4 * q u a rte r = 1 0 4 . 7 6 - ( 3 X 1 .5 8 )= 100.02
A v e ra g e o f a d ju ste d c h a in re la tiv e s

1 0 0 + 1 0 7 . 2 + 9 7 . 0 5 + 100.02

4 0 4 .2 7
= 101.07
4
A-29

Seasonal In d ice s :
100
Q u a rte r I = ----------- x 100 = 98.94
101.07
107.2
Q u a rte r II = ----------- x 100= 106.07
101.07
97.05
Q u a rte r III = ----------- x 100=96.02
101.07
W % \ ‘i i * '{ r ' »'.'•••»• \ ' ' '{■■■■ :»•

100.02
Q u a rte r IV = ----------- x 100 = 98.96
101.07

THEORY QUESTIONS
1. D e fin e tim e se rie s ? E n u m e ra te th e v a rio u s c o m p o n e n ts o f a tim e
se rie s in detail.
2. W h a t a re th e u se s o f A n a ly s is o f T im e S e rie s ?
3. D e fin e d tre n d . E x p la in th e d iffe re n t m e th o d s o f m e a s u rin g se c u la r
tr e n d in a g iv e n tim e series.
4. W h a t is a se a so n a l v a r ia tio n ? W h a t a re its c a u s e s ?
5. W h a t a re c y c lic v a ria tio n s ? H o w a r e th e y c a u s e d ?
6. W h a t a re irre g u la r v a ria tio n s ? H o w a r e th e y c a u s e d ?
,7. D isc u ss th e m e rits a n d d e m e rits o f d iffe re n t m e th o d s o f m e a s u rin g
T re n d ?
8. D is tin g u is h b e tw e e n tre n d , se a so n a l v a ria tio n s a n d c y c lic a l
flu c tu a tio n s in a tim e se rie s.
9. W h a t a re d iffe re n t m e th o d s o f.e s tim a tin g s e a s o n a l v a ria tio n s ?
(B.Com., Andhra)
10. W h a t d o y o u u n d e rs ta n d b y D e s e a s o n a lis a tio n o f D a ta ? E x p la in b y
m e a n s o f a n illu stratio n .

PROBLEMS
1. P lo t th e f o llo w in g d a ta o n a g ra p h a n d a s c e rta in tre n d b y th e fre e h a n d
m e th o d :
Year 1994 1995 1996 1997 1998 1999
Production 100 120 95 105 108 102
(Million tonnes)
A-30

2. F it a tr e n d lin e to th e fo llo w in g d a ta b y th e fre e -h a n d m eth o d .


Year 1995 1996 1997 1998 1999 2000
Sales 65 95 85 115 110 120
('000 units)

3. D ra w a tr e n d lin e b y th e m e th o d o f se m i-a v e ra g e s fro m th e fo llo w in g


d ata.
Year 1984 1985 1986 1987 1988 1989 1990 1991
Sales
(‘000 200 210 218 192 204 216 224 228
Units)
A lso p r e d ic t th e sa le s fro m th e y e a r 1993 fro m th e g rap h .
(A n s : S a le s f o r 1993 = 2 3 0 ) (B.Com., O.U.1993)

4. F it a tr e n d lin e to th e fo llo w in g d a ta b y th e m e th o d o f se m i-av e ra g es:

Year 1980 1981 1982 1983 1984 1985 1986


Sales of firm X (in 115 112
101 106 114 110 109
thousand units)

5, F in d th e tr e n d v a lu e s b y 3 - y e a rly m o v in g a v e ra g e :
Year 1 99 0 1991 1992 1 993 1994 1 995 1996 1 997 1998

Annual
S a le s (R s. 40 42 40 44 49 46 42 44 44
in '0 0 0 )

(B.Com., O.U.2000)

6, U s in g 3 -y e a r m o v in g av e ra g e , d e te rm in e th e tren d .
Year 1989 1990 1991 1992 1993 1994 1995 1996 1997 1998

P ro­ 21 22 23 25 24 22 25 26 27 26
duction
(B.Com., O.U.1999)

7, C a lc u la te 5 -y e a rly m o v in g a v e ra g e fro m th e f o llo w in g d a ta :


Y ear R a in fa ll Y ear R a in fa ll
( in in c h e s) ( in in c h es)
1978 100 1985 118
1979 94 1986 96
1980 81 1987 101
1981 78 1988 103
A-31

1982 102 1989 91


1983 147 1990 89
1984 158 1991 100
(B.Com., 0 7 0 1992)

8. C a lc u late 5 - y e a rly m o v in g a v e ra g e s o f n u m b e r o f s tu d e n ts in a
c o m m erce co lleg e a s s h o w n in b y th e fo llo w in g fig u re s : _______
Y ear 1966 1967 1968 1969 1970
N o. o f 332 317 357 392 402
S tu d en ts

Y ear 1971 1972 1973 1974 1975~"~]


N o. o f 405 410 427 405 408
S tu d en ts

D ra w a lso a g ra p h to re p re s e n t th e m o v in g av e rag es.

9. C a lc u late th e fo u r y e a rly m o v in g a v e ra g e fro m th e fo llo w in g data.


Y ear P ro d u c tio n Y ear P ro d u c tio n ^ '
1985 672 1990 802 "
1986 679 1991 807 "
1987 690 1992 809
1988 702 1993 816 _
1989 712 1994 821 '
(B.Com., o7u7l998)'

10. W o rk o u t th e c e n te re d 4 - y e a r ly m o v in g a v e ra g e fo r th e follow ing


d a ta :
Y ear T onnage o f Y ear T o n n ag e o f- ' '
g o o d s c a rrie d g o o d s carried
1966 2204 1972 2904 "
1967 2500 1973 3098 "
1968 2360 1974 3172 "
1969 2680 1975 2952 "
1970 2424 1976 3248 "
1971 2634 1977 3172 "
A-32

11. G iv e n b e lo w a re th e fig u re s o f p r o d u c tio n :


Year 1984 1985 1986 1987 1988 1989 1990
Production 77 88 94 85 91 98 90
F it a s tra ig h t lin e b y “ L e a s t sq u a re s” m e th o d , ta b u la te th e tre n d
v a lu e s a n d p lo t o n grap h .
(A n s : Y c = 8 9 + 2 X ) (B.Com. ,o.u.2000)

12. B e lo w a re g iv e n fig u re s o f p ro d u c tio n ( in ‘0 0 0 Q tls ) o f a su g a r


__ fa c to ry . _____ ____________________________ _ _ _ _ _ _
Year 1988 1989 1990 1991 1992 1993 1994
Production 80 90 92 83 94 99 92
a ) F it a stra ig h t lin e tre n d to th e a b o v e d a ta a n d c a lc u la te th e tre n d
* v a lu e s f o r a ll th e y e a rs.
b ) E stim a te th e p ro d u c tio n fo r 1998
(A n s : (a ) : Y c = 9 0 + 2 X (b ) = 10 4 ) (B.Com .o.u., 1998)

13. C o m p u te tre n d b y m e th o d o f le a s t sq u a re s f o r th e fo llo w i n g :


Y ear A s se ts o f R .B .I. ( in c ro re s)
19 7 6 -7 7 830
1977-78 920
1 9 7 8 -7 9 710
1 9 7 9 -8 0 900
1980-81 169C(
(A n s : Y c = 1010+ 17 0 X ) (B.Com., sku )

14. F it a stra ig h t lin e tr e n d b y th e m e th o d o f le a s t sq u a re s to th e


f o llo w in g d a ta . A s s u m in g th a t sa m e ra te o f c h a n g e c o n tin u e s , w h a t

Year 1977 1978 1979 1980 1981 1982 1983 1984


Earnings
(Rs.in 38 40 65 72 69 60 87 95
Lakhs)
(A n s : Y c = 6 5 .7 5 + 7 .3 3 ; 1985 = R s. 9 8 .7 5 L a k h s) (B.Com. Andhra)

15, F it a s tra ig h t lin e tr e n d fo r th e f o l l o w i n g ;-


Y ear R e se rv e s R s. Y ear R e s e rv e s R s.
1 9 7 7 -7 8 6 ,1 2 ,0 0 0 1 9 8 1 -8 2 1 0 ,0 1 ,0 0 0
1 9 7 8 -7 9 7 ,1 9 ,0 0 0 1 9 8 2 -8 3 1 1 ,0 6 ,0 0 0
A-33

1 9 7 9 -8 0 8 ,2 0 ,0 0 0 1 9 8 3 -8 4 1 2 ,3 1 ,0 0 0
1 9 8 0 -8 1 9 ,0 7 ,0 0 0
(A n s : 9 1 3 7 1 4 .2 8 + 1 0 0 4 2 8 .5 7 X ) (B.Com.SKU)

16. F it a tr e n d lin e b y th e m e th o d o f le a s t s q u a re s to th e f o llo w in g d a t a :


Y ear 1970 1971 1972 1973 1974
S a le s ( in to n n e s ) 10 12 18 20 25
(A n s : Y c = 17 + 3 .8 X ) (B .C o m ., K a k a tiy a )

17. B e lo w a r e g iv e n th e fig u re s o f p r o d u c tio n (in th o u s a n d q u in ta ls ) o f a


s u g a r fa c to ry . ____________________________________________________
Year 1985 1987 1988 1989 1990 1991 1994
Production
thousand 154 176 188 170 182 196 180
quintals
—a------------------------- L_--------- ------------ ------------ ------------ ------------ ------------
F it a s tra ig h t lin e b y th e le a s t sq u a re s m e th o d a n d ta b u la te th e tr e n d valu es.
( A n s : 1 7 7 .6 0 7 + 2 .7 5 X )

18. A n n u a l tr e n d o f m ilk c o n s u m p tio n (y ) is 18.6 + 1.8x, c o n v e rt th e


e q u a tio n in to m o n th ly b asis.

(A n s : 1.55 + 0 .0 1 2 5 X ) (B.Com., Kakatiya)

19. Y = 1 1 0+ 2x. S h ift th e o rig in fro m 1 9 8 2 to 1986 (tim e u n it 1 y ea r).


(A n s : Y = 118 + 2 X ) (B.Com., Kakatiya)

20. S h ift th e b a s e y e a r o f th e fo llo w in g tr e n d e q u a tio n f ro m 1 9 9 4 to 1990


a n d re w rite th e e q u a tio n :-
Y c = 4 2 .2 - 1 .5 x ; (1 9 9 4 = 0 )
(A n s: Y = 4 8 . 2 - 1 .5 X ) (B.Com , Kakatiya)

21. U s in g 1 9 9 0 a s th e o rig in , o b ta in th e stra ig h t lin e tr e n d e q u a tio n b y


th e m e th o d o f le a s t s q u a r e s . ___________________________
Year 1985 1986 1987 1989 1990 1992 1993
Sales 700 720 800 760 840 880 900
F in d th e sa le s in th e 1 9 8 8 , 1991
( A n s : Y c = 8 2 9 + 2 5 X ; 1 9 8 8 = 7 7 9 ; 1991 = 8 5 4 )

22. A s s u m in g th a t th e tr e n d is a b s e n t, d e te rm in e i f th e re is a n y
s e a s o n a lity in th e d a ta g iv e n b e lo w .
A-34

Y ear I Q u a rte r II Q u a rte r III Q u a rte r IV Q u a rte r


(J-F -M ) (A -M -J) (J-A -S ) (O -N -D )
1982 3 .7 4.1 3.3 3.5
1983 3 .7 3 .9 3 .6 3 .6
1984 4 .0 4.1 3.3 3.1
1985 3 .3 4 .4 4 .0 4 .0
C o n s tru c t se a so n a l In d ic e s b y A v e ra g e m e th o d fo r th e fo u r Q u arte rs.
( A n s : 9 8 .7 ; 11 0 .8 ; 9 5 .3 ; 9 5 .3 )

23. F ro m th e d a ta g iv e n b e lo w c a lc u la te se a so n a l in d ic e s fo r I, II, III an d


IV q u a rte rs a s s u m in g th e tre n d is absent.
Y ea rs
Q u a rte rs
1976 1977 1978 1979 1980
I 40 42 41 45 44
II 35 37 35 36 38
III 38 39 38 36 38
IV 40 38 40 41 42
(A n s : 1 0 8 .1 6 ; 9 2 .3 5 ; 9 6 .4 3 ; 102 .5 5 )

24. C a lc u la te se a s o n a l in d ic e s b y ra tio to tre n d m e th o d fro m th e


fo llo w in g d ata.
Y ear 1st Q u a rte r 2 nd Q u a rte r 3rd Q u a rte r 4 th Q u a rte r
1991 36 34v 38 32
1992 38 48 52 42
1993 42 56 50 52
1994 56 74 64 62
1995 82 90 88 80
(A n s : 1 0 0 .0 3 ; 1 0 9 .5 5 ; 1 0 3 .1 4 ; 8 7 .2 8 )

25. F in d th e se a s o n a l v a ria tio n b y th e ra tio to tre n d m e th o d fro m th e d a ta


g iv e n b e l o w :
Y ear 1st Q u a rte r 2 nd Q u a rte r 3rd Q u a rte r 4 th Q u a rte r
1996 60 80 72 68
1997 68 104 100 88
1998 80 116 108 96
1999 108 152 136 124
A-35

2000 160 184 172 164


(A n s : 9 2 .0 5 ; 117.36; 10 2 .1 2 ; 8 8 .4 7 )

26. C a lc u la te th e se a so n a l in d ic e s b y th e r a tio to th e m o v in g a v e ra g e
m e th o d fro m th e fo llo w in g d a t a
O u t p u t o f W h e a t in M illio n to n s
Y ear 1st Q u a rte r 2 nd Q u a rte r 3 rd Q u a rte r 4 th Q u a rte r
1994 68 62 61 63
1995 65 58 56 61
1996 68 63 63 67
1997 70 59 56 62
1998 60 55 51 58
(A n s : 107 .0 2 ; 9 6 .4 3 ; 9 4 .4 3 ; 102.12).

27. C a lc u la te se a so n a l in d ic e s b y (A ) th e r a tio to m o v in g a v e ra g e m e th o d
a n d (B ) th e m e th o d o f lin k re la tiv e f r o m th e fo llo w in g data.

W h e a t P ric e s ( in R u p e e s p e r Q u in ta l)
Y ear 1975 1976 1977 1978
Q u a rte r

Q. 75 86 90 100
q 2 60 65 72 78
q 3 54 63 66 72
Q4 59 80 85 93

A ns. ( A ) : 122.37; 9 2 .4 3 ; 8 4 .6 9 ; 1 0 0 .5 1 1
(B ) : 124.19; 9 3 .4 6 ; 8 2 .4 9 ; 9 9 .8 6 J
28. C a lc u la te th e se a so n a l v a r ia tio n in d ic e s b y th e m e th o d o f lin k
re la tiv e s fo r th e fo llo w in g fig u re s :
Y ear
Q u a rte r
1990 1991 1992 1993 1994
I 45 48 49 52 60
II 54 56 63 65 70
III 72 63 70 75 84
IV 60 56 65 72 86
( A n s : 82; 9 8 .4 ; 116.8; 102.6)
A-36

29. A c o m p a n y e s tim a te s its sales for a p a rtic u la r y e a r to be


R s. 2 4 ,0 0 ,0 0 0 . T h e se aso n a l indices fo r sales a re a s fo llo w s :
M o n th S e a so n a l Index M o n th S e a so n a l In d ex
J a n u a ry 75 July 102
F e b ru a ry 80 A u g u st 104
M a rc h 98 S ep tem b er 100
A p ril 128 O cto b er 102
M ay 137 N ovem ber 82
June 119 D ec em b e r 73

U s in g th is in fo rm a tio n , ca lc u la te e stim a te s o f m o n th ly sa le s o f th e
c o m p a n y . A s s u m e th a t th e re is n o trend.

2 4 ,0 0 ,0 0 0
[ H i n t : (i) M o n th ly S ale s = ------------------- = Rs. 2 ,0 0 ,0 0 0
12

S easo n al In d ex
(ii) S e a s o n a l E ffe c t = ---------------------------
100
(iii) T h e e s tim a te d sa le fo r a n y m o n th w o u ld b e m o n th ly sa les
x se a s o n a l e ffe c t ]

30. T h e se a so n a l in d ic e s o f th e v s a le o f re a d y -m a d e g a rm e n ts o f a
p a r tic u la r ty p e in a c e rta in sto re a re g iv e n b e l o w :_____________________
Q u a rte r S e a so n a l In d e x
I J a n u a ry - M a rc h 98
II A p ril - Ju n e 89
III J u ly - S e p te m b e r 82
I V O c to b e r - D e c e m b e r 130

I f th e to ta l sa le s in th e firs t q u a rte r o f a y e a r b e w o rth R s. 10 ,0 0 0 , d e te rm in e


h o w m u c h w o rth o f g a rm e n ts o f th is ty p e sh o u ld b e k e p t in sto c k b y th e
sto re to m e e t th e d e m a n d in e a c h o f th e re m a in in g q u arters.

(A n s : 9 ,0 8 2 ; 8 ,3 6 7 ; 13,265).

10000x89
[ H i n t : S a le s f o r II Q u a rte r = = 9 ,0 8 2 ]
98
CHAPTER-X
INDEX NUMBERS

Introduction :
H isto ric a lly , th e firs t in d f x n u m b e r w a s c o n stru c te d in t h e y e a r 1764
b y a n Ita lia n n a m e d C a rli th e co m p are th e c h a n g e s in th e p ric e f o r th e y e a r
1750 w ith th e p ric e le v e l fo r th e y ea r 1500. T h o u g h o rig in a lly d e v e lo p e d
f o r m e a s u rin g th e e ffe c t o f c h a n g e in p ric e s, th e re is h a rd ly a n y p h e n o m e n a
to d a y w h e re in d e x n u m b e rs a re n o t u sed . N e w sp a p e rs h e a d lin e th e fa c t th a t
p ric e s are g o in g u p o r d o w n , th a t in d u stria l p ro d u c tio n is r is in g o r fa llin g ,
th a t im p o rts a n d e x p o rts a re in c re a sin g o r d e c re a s in g in a p a r tic u la r p e rio d
c o m p a re d to th e p re v io u s p e rio d as d isc lo se d b y in d e x n u m b e rs . In d e x
n u m b e rs a re s u p p o s e d to b e b a ro m e te rs o f e c o n o m ic a c tiv ity , i.e ., i f o n e
d e s ire s to g e t a n id e a a s to w h a t is h a p p e n in g to a n e c o n o m y , h e sh o u ld
lo o k to im p o rta n t in d ic e s lik e th e in d e x n u m b e r o f in d u stria l p ro d u c tio n ,
a g ric u ltu ra l p ro d u c tio n , b u sin e ss activ ity , etc. T h e im p o rta n c e o f in d e x
n u m b e rs in th e re a lm o f q u an tita tiv e te c h n iq u e s c a n n o t b e o v e r
em p h asised .

Meaning and Definition :


In d ex n u m b e rs a re s ta tistic a l dev ices d e s ig n e d to m e a su re th e re la tiv e
c h a n g e in th e le v el o f a p h e n o m e n o n w ith re sp e c t to tim e , g e o g ra p h ic a l
lo c a tio n s o r o th e r c h a ra c te ristic s su ch a s in c o m e , p ro d u c tio n , s a le s etc. T h e
fo llo w in g are so m e o f th e im p o rta n t d e fin itio n s o f in d e x n u m b e rs.

In d e x N u m b e rs h a v e b e e n d e fin e d b y C ro x to n a n d C o w d e n a s
“D e v ic e s fo r m e a s u rin g d iffe re n c e s in th e m a g n itu d e o f a g r o u p o f r e la te d
v a ria b le s” .

A c c o rd in g to H o ra c e S e c rist, “In d ex n u m b e rs are a se rie s o f n u m b e rs


b y w h ic h c h a n g e s in th e m a g n itu d e o f a p h e n o m e n o n a re m e a s u re d fro m
tim e to tim e , p la c e to p la c e .”

It is v ery c le a r fro m th e ab o v e d e fin itio n s th a t a n in d e x n u m b e r is a


sp e c ia lise d av e ra g e d e s ig n e d to m e a su re th e c h a n g e s in a g r o u p o f re la te d
v a ria b le s o v e r a p e rio d o f tim e.

IMPORTANCE
In d e x n u m b e rs a re in d is p e n sa b le to o ls in th e h a n d s o f g o v e rn m e n t
o ffic ia ls, b u sin e ssm e n , e c o n o m is ts, in d u s tria lis ts a n d p ro fe s s io n a l m e n to
B-2

a n a ly se b u s in e s s a n d e c o n o m ic situ a tio n s o f a co u n try . S o m e o f th e


im p o rta n t u s e s o f in d e x n u m b e rs ar e lis te d a s u n d e r :

(i) In d e x n u m b e rs a re th e m o s t p o w e rfu l to o ls o f e c o n o m ic a n d b u sin e ss


an a ly sis. T h e y a r e u s e d to fe e l th e p u ls e o f th e ec o n o m y . T h e y h a v e
c o m e to b e u s e d a s in d ic a to rs o f in fla tio n a ry a n d d e fla tio n a ry
te n d e n c ie s , th a t is w h y th e y a re d e s c rib e d a s “ e c o n o m ic b a ro m e te rs .”

( ii) In d e x n u m b e rs a re h e lp fu l in p la n n in g a n d fo rm u la tio n o f v a rio u s


e c o n o m ic a n d b u sin e ss p o lic ie s.
,
( iii) In d e x n u m b e rs h e lp so c ia l sc ie n tists to stu d y tre n d s a n d te n d e n c ie s o f
im p o rta n t b u s in e s s , e c o n o m ic a n d s o c ia l v a ria b le s o v e r a p e rio d o f
tim e i.e ., im p o rts a n d e x p o rts, a g ric u ltu ra l a n d in d u s tria l p ro d u c tio n ,
p o p u la tio n etc.

(iv ) In d e x n u m b e rs m e a s u re th e p u rc h a s in g p o w e r o f m o n e y a n d h e lp in
f in d in g o u t th e re a l w a g e s o r in c o m e s o f th e p eo p le.

L im i ta ti o n s :
In d e x n u m b e rs, d e s p ite o f its u se fu ln e ss to e c o n o m is ts, b u s in e s s m e n
a n d o th e rs, h a v e f o llo w in g lim ita tio n s :

(i) S in c e in d e x n u m b e rs a re b a s e d o n s a m p le d a ta , th e y o n ly p ro v id e
a p p ro x im a te re su lts w h ic h m a y n o t e x a c tly re p re s e n t th e c h a n g e s in
re la tiv e lev els. (

( ii) W e c a n n o t c a lc u la te a sin g le o r u n iq u e in d e x n u m b e r c o m m o n to all


th e e c o n o m ic a c tiv itie s. F o r e a c h p h e n o m e n o n u n d e r stu d y , w e h a v e
to c o m p u te a sp e c ia l in d e x n u m b e r.

(iii) A s in d e x n u m b e rs a re s p e c ia lis e d a v e ra g e s , th e y a re su b je c t to th o se
lim ita tio n s to w h ic h a n a v e ra g e su ffe rs.

(iv ) T h e re a re m o re c h a n c e s o f e rro rs in c o n s tru c tio n o f in d e x n u m b e rs. It


m a y b e a n y w h e re fro m le v el o f s e le c tio n o f c o m m o d itie s to c h o ic e
o f th e fo rm u la .

(v) In d e x n u m b e rs c a n b e c o n s tru c te d in m a n y d iffe re n t w ay s. D iffe re n t


m e th o d s g iv e d iffe re n t a n sw e rs.
C
B-3

K in d s o f I n d e x N u m b e r s :
O n th e b a s is o f th e s tu d y o f re la tiv e c h a n g e s in d iffe re n t v a ria b le s ,
in d e x n u m b e rs a re b r o a d ly c la s s ifie d in to v a rio u s ty p es.

1. P r ic e I n d e x N u m b e r s : A m o n g a ll th e in d e x n u m b e rs , p ric e in d e x
n u m b e rs a re m o s t c o m m o n ly u s e d in e c o n o m ic a n d b u sin e ss fie ld s to
m e a su re th e re la tiv e p r ic e le v e l c h a n g e s o f c o m m o d itie s a t so m e tim e
o r a t c e rta in p la c e w ith re f e r e n c e to so m e b a s e p e rio d . P ric e in d e x
n u m b e rs a re fu rth e r d iv id e d in to tw o ty p es.

a) W h o le s a le P ric e I n d e x N u m b e rs : T h e g e n e ra l p ric e lev el


c h a n g e s in a c o u n try a r e s tu d ie d b y u s in g w h o le s a le p ric e in d e x
n u m b e rs.

b) R e ta il P ric e In d e x N u m b e r s : R e ta il p ric e in d e x n u m b e rs a re
v e r y u s e fu l to m e a s u r e g e n e r a l c h a n g e s in re ta il p ric e s o f v a rio u s
c o m m o d itie s s u c h a s c o n s u m p tio n g o o d s, b a n k d e p o s its, b o n d s,
s h a re s etc. T h e y a re p o p u la rly k n o w n a s th e “ C o st o f L iv in g
In d e x N u m b e rs ” o r “ C o n s u m e r P ric e In d e x N u m b e rs ” .

2. Q u a n t i t y I n d e x N u m b e r s : T h e s e in d e x n u m b e rs stu d y th e c h a n g e s
in th e v o lu m e o f g o o d s p r o d u c e d , c o n s u m e d a n d d istrib u te d d u rin g a
p a rtic u la r p e r io d o f tim e a s c o m p a re d to its b a s e p e rio d . In d ic e s o f
a g ric u ltu ra l a n d in d u s tria l p ro d u c tio n , im p o rts a n d e x p o rts, etc., a re
e x a m p le s o f q u a n tity in d e x n u m b e rs .

3. V a lu e I n d e x N u m b e r s : T h e s e in d e x n u m b e rs a re m e a n t to m e a su re
th e c h a n g e s in th e to ta l v a l u e o f c o m m o d itie s c o n c e rn e d fo r a p e rio d
o f tim e. H o w e v e r, th e s e a r e n o t c o m m o n ly u se d lik e p ric e a n d
q u a n tity in d e x n u m b e rs .

P R O B L E M S I N T H E C O N S T R U C T I O N O F IN D E X N U M B E R S

B e fo re c o n s tru c tin g in d e x n u m b e rs th e fo llo w in g p re lim in a rie s


sh o u ld b e c a re fu lly c o n s id e re d .

1. T h e p u r p o s e o f t h e i n d e x n u m b e r s : S in ce in d e x n u m b e rs are
c o n s tru c te d f o r a v a r ie ty o f ite m s lik e p ric e s, w ag e s, p ro fits, fo re ig n
tra d e a n d s im ila r o th e r ite m s , w e sh o u ld first d e c id e th e p u rp o se fo r
w h ic h in d e x n u m b e r h a s to b e c o m p u te d . F a ilu re to c le a rly d e fin e tire
p u rp o se o f th e in d e x n u m b e r , w o u ld n o t le ad to a n y u se fu l resu lts.
B-4

2. Selection of the b a s e year : T h e se le c tio n o f a b a s e p e rio d p rim a rily


d e p e n d s o n th e p u rp o se o f th e in d e x n u m b e r. H o w e v e r, th e
fo llo w in g p o in ts a re s ig n ific a n t to se le c t a n a p p ro p ria te b a s e p erio d .

a) B a se y e a r c h o s e n sh o u ld b e a y e a r o f n o rm a l a n d sta b le
e c o n o m ic c o n d itio n s.
b) T h e b a s e y e a r sh o u ld n o t b e to o d is ta n t fro m th e c u rre n t year.
c) W h ile s e le c tin g th e b a s e a d e c is io n h a s to b e m a d e a s to w h e th e r
th e b a s e sh a ll re m a in fix e d o r n o t, i.e., w h e th e r w e h a v e a fix e d
b a s e o r c h a in b a s e in d e x .

3. Selection of the number o f items : E v e ry ite m c a n n o t b e in c lu d e d


w h ile c o n s tru c tin g a n in d e x n u m b e r a n d h e n c e o n e h a s to se le c t a
sa m p le . F o r e x a m p le , w h ile c o n s tru c tin g a p ric e in d e x it is n o t
p o ssib le to in c lu d e e a c h a n d e v e ry c o m m o d ity . W e h a v e to se lec t
c o m m o d itie s in su c h a m a n n e r th a t th e y a r e re p re se n ta tiv e o f th e
ta ste s, h a b its a n d c u s to m s o f th e p e o p le fo r w h o m th e in d e x is m e an t.

4. P r ic e quotations : T h e p ric e s o f c o m m o d itie s v a ry fro m p la c e to


p la ce . It is n o t p o ssib le to o b ta in p ric e q u o ta tio n s fro m a ll p la ce s.
P ric e q u o ta tio n s o f o n ly re p re se n ta tiv e m a rk e ts sh o u ld b e ta k e n in to
acco u n t.

5. Choice of an average : sin c e in d e x n u m b e rs a re s p e c ia lise d


a v e ra g e s , a d e c is io n h a s to b e m a d e a s to w h ic h a v e ra g e sh o u ld b e
u s e d fo r c o n s tru c tin g th e in d ex . A lth o u g h th e re a re a n u m b e r o f
a v e ra g e s , b a s ic a lly a ch o ic e h a s to b e m a d e b e tw e e n a rith m e tic m e a n
a n d g e o m e tric m e a n . T h e o re tic a lly sp e a k in g , g e o m e tric m e a n in th e
b e s t av e rag e.

6. Selection of appropriate weights : T h e te rm ‘w e ig h t’ re fe rs to th e


re la tiv e im p o rta n c e o f th e d iffe re n t ite m s in th e c o n s tru c tio n o f th e
in d e x n u m b e r. A ll ite m s a re n o t o f e q u a l im p o rta n c e a n d h e n c e it is
n e c e s sa ry to d e v ise so m e su ita b le m e th o d w h e re b y th e v a ry in g
im p o rta n c e o f th e d iffe re n t ite m s is ta k e n in to a c c o u n t. T h is is d o n e
b y a llo c a tin g w eig h ts. G e n e ra lly q u a n tity w e ig h ts a re a s sig n e d w h e n
th e ite m s a re v a ry in g in q u a n tity w h ile c o n s u m in g , p u rc h a s in g o r
sellin g . T h e v a lu e w eig h ts a re a s s ig n e d w h e n e x p e n d itu re in c u rre d o n
d iffe re n t ite m s v a rie s.

7. Choice of method : A la rg e n u m b e r o f m e th o d s h av e b e e n d e v ise d


fo r c o n s tru c tin g th e d iffe re n t ty p e s o f in d e x n u m b e rs. T h e c h o ic e o f a
B-5

p a rtic u la r m e th o d w ill d e p e n d u p o n th e fa c to rs su c h a s a v a ila b ility o f


d a ta , p u rp o se fo r w h ic h in d e x n u m b e rs a re re q u ire d a n d re s o u rc e s at
th e d isp o s a l fo r c o n s tru c tin g in d e x n u m b e rs.

METHODS OF CALCULATING INDEX NUMBERS

T h e v a rio u s fo rm u la e u se d fo r c o n s tru c tin g in d e x n u m b e rs a re


c la ssifie d u n d e r tw o h e a d s :

1. Unweighted indices a n d
2. Weighted indices

(1) Unweighted indices : In u n w e ig h te d in d ic e s w e ig h ts , a re n o t


e x p re s s ly a s sig n e d . T h is ty p e m a y b e f u rth e r d iv id e d in to tw o
h ead s.

(A ) S im p le A g g re g a tiv e M e th o d , a n d
(B ) S im p le A v e ra g e o f P ric e R e la tiv e s M e th o d .

(A) Simple Aggregative Method : W h e n th is m e th o d is u s e d to


c o n s tru c t a p ric e in d e x th e to ta l o f c u rre n t y e a r p ric e s f o r th e
v a rio u s c o m m o d itie s in q u e s tio n is d iv id e d b y th e to ta l o f b a s e
y e a r p ric e s a n d th e q u o tie n t is m u ltip lie d b y 100.

T h e fo llo w in g step s a re re q u ire d fo r c o m p u ta tio n :

(i) A d d th e c u rre n t y e a r p ric e s fo r v a rio u s c o m m o d itie s, i.e., o b ta in


SPi-
(ii) A d d th e b a s e y e a r p ric e s f o r th e sa m e c o m m o d itie s, i.e ., o b ta in
£Po
(iii) A p p ly th e fo llo w in g fo rm u la .

IP i
Poi = ----------- x 100
SP0

w h e re , P 0i = P ric e in d e x fo r c u rre n t y e a r o n th e b a s is o f b a s e y e a r
prices.
E P i = T o ta l o f c u rre n t y e a r p ric e s o f v a rio u s c o m m o d itie s.
I P o = T o ta l o f b a s e y e a r p ric e s o f v a rio u s co m m o d itie s.
B-6

I llu s tr a tio n 1 : F ro m th e f o llo w in g d a ta c o n s tru c t a n in d e x fo r 1991


ta k in g 1990 a s b a s e :

P 01= -----------x 1 0 0 = ---------------x 1 0 0 = 117.14


SPo 350

(B ) S im p le A v e r a g e o f P r ic e R e la tiv e s M e th o d : U n d e r th is
m e th o d p ric e re la tiv e s a re c o m p u te d fo r e a c h c o m m o d ity . P ric e
re la tiv e is th e ra tio o f c u r r e n t y e a r p ric e to th e b a s e y e a r price.
E ith e r A rith m e tic M e a n o r G e o m e tric M e a n is used.
(a ) W h e n A r ith m e tic M e a n is u s e d , th e fo llo w in g p ro c e d u re is
a d o p te d to c o m p u te p ric e in d e x .
(i) C a lc u late th e p ric e re la tiv e (P ) fo r e a c h co m m o d ity b y

Pi
u s in g th e f o rm u la ----------- x 100
Po
B-7

(ii) O b ta in th e to ta l o f p ric e re la tiv e s (E P )

IP
(iii) A p p ly th e f o r m u l a : P 0I ~
N

W h e re , E P = S u m o f P ric e re la tiv e s o f a ll th e c o m m o d itie s.

N = N u m b e r o f ite m s o r c o m m o d itie s.

(b ) W h e n G e o m e tr ic M e a n is u s e d , th e fo llo w in g p ro c e d u re is
a d o p te d to co m p u te p ric e in d e x .
(i) C a lc u la te p ric e re la tiv e s f o r e a c h c o m m o d ity

Pi
( i . e . , P = ----------- x 1 0 0 )
Po
(ii) F in d o u t lo g v a lu e s fo r v a rio u s p ric e re la tiv e s (lo g p ) a n d
o b ta in th e to ta l E lo g p.
E lo g p
(iii) A p p ly th e f o rm u la : P0i = A n tilo g -----------
N
W h e re, E lo g p = S u m o f lo g v a lu e s o f p ric e re la tiv e s
N = N u m b e r o f ite m s

I l l u s tr a tio n 2 : C a lc u la te In d e x N u m b e r b y A v e ra g e o f re la tiv e m e th o d
u s in g A rith m e tic M e a n a n d G e o m e tric M e an .

C o m m o d ity A B C D E
P ric e in 1990 6 10 2 12 5
P ric e in 1995 8 15 4 8 5

S o lu tio n : C o m p u ta tio n o f P ric e In d ex

P ric e
P ric e in P ric e in R e la tiv e s
C o m m o d ities Pi lo g p
1990 1995
p = — X too
Po
Po Pi P
A 6 8 133.33 2 .1 2 4 9
B 10 15 15 0 .0 0 2.1 7 6 1
C 2 4 2 0 0 .0 0 2 .3 0 1 0
B-8

D 12 8 6 6 .6 7 1 .8 2 4 0
E 5 5 1 0 0 .0 0 2 .0 0 0 0
X P = 650 S lo g p =
1 0 .4 2 6
(a ) W h e n A rith m e tic M e a n is u s e d :

IP 650
P01= -------- = --------- = 130
N 5

(b ) W h e n G e o m e tric M e a n is u s e d :

E lo g p 10.426
Poi = A n tilo g -------------- = A n tilo g --------------
N 5

= A n tilo g 2 .0 8 5 2 .
= 121.7

(2) Weight indices : W e ig h te d in d e x n u m b e rs a re o f tw o ty p e s :

(A ) W e ig h te d A g g re g a tiv e In d ic e s, a n d
(B ) W e ig h te d A v e ra g e o f R e la tiv e s In d ice s.

(A) Weighted Aggregative Inidces : U n d e r th is ty p e w e ig h ts are


a s s ig n e d to th e v a rio u s ite m s in c lu d e d in th e in d e x . T h e re a re
v a rio u s m e th o d s fo r c o n s tru c tio n o f in d e x n u m b e rs. U n d e r th is
ty p e , th e f o llo w in g a re so m e o f th e im p o rta n t m e th o d s.

(i) Laspeyres Method : U n d e r th is m e th o d th e w e ig h ts a re


d e te rm in e d b y th e q u a n titie s in th e b a s e p e rio d . The
f o llo w in g ste p s a re re q u ire d fo r c o n s tru c tin g in d ex .

(a ) M u ltip ly th e c u rre n t y e a r p ric e s o f v a rio u s c o m m o d itie s


w ith b a s e y e a r w e ig h ts a n d o b ta in £ P iq 0.
(b ) M u ltip ly th e b a s e y e a r p ric e s o f v a rio u s c o m m o d itie s
w ith b a s e y e a r w e ig h ts a n d o b ta in £ P oq0.
(c ) A p p ly th e f o llo w in g f o r m u l a :

£Piq0
Poi = ----------- x 100
2 P 0q 0
B-9

(ii) P a a s c h e ’s M e th o d : U n d e r th is m e th o d th e w e ig h ts a re
d e te rm in e d b y q u a n titie s in th e g iv e n y e a r. T h e f o llo w in g
ste p s a re r e q u ire d fo r c o n s tru c tin g in d e x :
(a ) M u ltip ly c u rre n t y e a r p ric e s o f v a rio u s c o m m o d itie s w ith
c u rre n t y e a r w e ig h ts a n d o b ta in Z P iq i.
(b ) M u ltip ly th e h a s e y e a r p ric e s o f v a r io u s c o m m o d itie s
w ith c u rre n t y e a r w e ig h ts a n d o b ta in X P0qi.
(c ) A p p ly th e f o llo w in g f o r m u l a :

EPjqi
Poi = ----------- x 100
EPoqi

(iii) Fishers I d e a l Index : T h e F is h e r ’s Id e a l I n d e x is g iv e n b y the


fo llo w in g f o r m u l a e :

W h e re , L = L a sp e y re s In d e x , P = P a a s c h e ’s In d e x

(iv ) D o rb is h a n d B o w le y ’s M e th o d :

EP^o EPiqi

SPoqo EP0qi
Poi = ----------------------------------- x 100
2

(v) Marshall - Edgeworth Method :

EPiqo + EP.qi
Poi = --------------------------- x 100
E P 0q 0 + E P 0q i

(v i) Kelly’s Method : U n d e r th is m e th o d a v e ra g e q u a n tity o f tw o o r


m o re y e a rs m a y b e u s e d a s w e ig h ts. F o r eg.

q0+qi EPiq
q = ----------- ; Poi = ------------ x 100
2 £P0q
B-10

I llu s tr a tio n 3 : C o m p u te w e ig h te d p rice in d e x fro m th e fo llo w in g d a ta b y


u s in g

(i) L a s p e y re ’s m e th o d
(ii) P a a s c h e ’s m e th o d
(iii) D o ib is h B o w ie y ’s m e th o d
(iv ) M a rs h a ll-E d g e w o rth m e th o d
(v ) F is h e r’s Id e a l m e th o d

B a se y e a r 1995 C u rre n t y e a r 1998


C o m m o d ity
P ric e Q u a n tity P ric e Q u a n tity
A 3 10 5 20
B 6 12 8 15
C 5 15 6 20
D 4 20 3 30

Solution :
C o n s tru c tio n o f W e ig h te d P ric e In d ice s
Com 1995 1998
m o d it Poqo Poqi
Po qo Pi qi Piqo P iq i
ies
A 3 10 5 20 50 100 30 60
B 6 12 « 15 96 120 72 90
C 5 15 ( 6 20 90 120 75 100
D 4 20 3 30 60 90 80 120
.1! ' t 2 p iq 0 S p iq i Epoqo Spoqi
=296 =430 =257 =370

(i) L a s p e y r e ’s m e th o d

Spiqp
Poi = ----------- x 100
Epoqo

296
Poi = ------- x 100
257

= 115.17


B -ll

(ii) Paasche’s method

Spiqi
Poi = ------------- x 100
Spoqi

430
P o i= ---------x 100
370

= 116.2

(iii) Dorbish and Bowley’s method

Spiqo Epiqi
-----------+ ------------
Epoqo Epoqi
P o i= -------------------------------------- x 100

296 430
--------+ ----------
257 370

Po.= x 100

2.31
x 100
2

115.52

(iv) Marshall-Edgeworth method

E p,qo + E p ,q ,
P 01= ------------------------------- x 100
Epoq0 + Epoqi

296+430
Poi= ------------------ x 100
257 + 370 i . 11 11 1

726
x 100
627

= 115.78
B-12

(v) Fisher’s Ideal method

= V l . 1 5 x 1 .1 6 x 100

= V 1 .3 3 4 x 100

= 1 .1 5 5 x 1 0 0

= 115.5

Illustration 4 :
F o llo w in g a r e th e p ric e a n d q u a n titie s o f c e rta in ite m s. C o n s tru c t K e lle y ’s
in d e x .
Q u a n titie s P ric e (in R s.)
Ite m s
1991 1993 1991 1993
R ic e 20 30 10 20
W heat 40 60 8 10
D al /— 30 80 20 30
O th e rs ( 10 30 10 20

Solution : ________________ C o n s tru c tio n o f K e lly ’s I n d e x


Q u a n titie s P ric e ( in R s.)

Ite m 1991 1993 1991 1993


qo qi Po Pi qo+qi Poq P iq
q-
2
R ic e 20 30 10 20 25 250 500
W heat 40 60 8 10 50 400 500
D al 30 80 20 30 55 1100 1650
O th e rs 10 30 10 20 20 200 400
£poq = 2 P iq =
T o ta l 1950 3050
B-13

E p iq
P 0i = -------------- x 100
Spoq

3050
= ----------- x 100
1950

= 156.41

(B) Weighted Average o f Relatives Indices : (a) When arithmetic


Mean is used, th e fo llo w in g p o in ts m u s t be fo llo w e d :
(i) C a lc u la te p ric e re la tiv e s f o r e a c h c o m m o d ity

(ii) C a lc u la te v a lu e w e ig h ts (V ), V = P 0 x qO a n d o b ta in th e to ta l
IV .
(iii) M u ltip ly e a c h (p ) v a lu e w ith v a lu e w e ig h ts (V ) a n d o b ta in th e
to ta l I P V .
(iv ) A p p ly th e fo llo w in g f o r m u l a : P 0i = I P V / I V

(b) When Geometric Mean is used, th e fo llo w in g p o in ts m u s t be


f o llo w e d :
(i) F in d th e p ric e re la tiv e s f o r e a c h c o m m o d ity (p )
(ii) T a k e lo g v a lu e f o r e a c h p ric e re la tiv e (lo g p )
(iii) C a lc u la te v a lu e w e ig h ts (V ) a n d o b ta in th e to ta l I V .
(iv ) M u ltip ly e a c h lo g p v a lu e w ith v a lu e w e ig h ts (lo g P X V = lo g
P V ) a n d o b ta in th e to ta l I l o g P V .
Ilo g PV
(v ) A p p ly th e f o rm u la : P 0i = A n tilo g ----------------
IV

N o te : I f w e ig h ts a re g iv e n in th e p ro b le m (W ) th e n d o n o t c a lc u la te v a lu e
w eig h ts

I llu s tr a tio n 5 : F ro m th e f o llo w in g d a ta c o m p u te p ric e in d e x b y w e ig h te d


av e ra g e o f p ric e re la tiv e s m e th o d u s in g (i) A rith m e tic M e a n a n d (ii)
G eo m e tric M ean.
C o m m o d ity P ric e in 1995 Q u a n tity in 1995 P ric e in 2 0 0 0
A 35 5 48
B-14

B 25 10 40
C 40 6 50

S o lu tio n : (i) C o n s tru c tio n o f P ric e In d e x B y u s in g w e ig h te d A rith m e tic


M e a n o f P ric e R e la tiv e s.
Com ­ P ric e in Q u a n tity P ric e Pi V (p0qo) PV
P = -----------X 1 0 0
m o d ity 1995 in 1995 in
Po
2000
' Po qo Pi
A 35 5 48 137.1 175 2 3 ,9 9 3
B 25 10 40 160.0 250 4 0 ,0 0 0
C 40 6 50 125.0 240 3 0 ,0 0 0
SV = EPV =
665 93993

Epv 93993
P 01= --------------= --------------- = 14 1 .3 4
Ev 665

(ii) C o n s tru c tio n o f p ric e in d e x B y u s in g w e ig h te d G e o m e tric M e a n


o f P ric e R e la tiv e s.
Com­ Price in Quan­ Price Pi
logp V logPV
modity 1995 po tity in in P = --------X 100 (Poqo)
)■ 1995 2000 Po

qo Pi
A 35 5 48 137.1 2.1370 175 373.98
B 25 10 40 160.0 2.2041 250 551.03
C 40 6 50 125.0 2.0969 240 503.26
EV ZlogPV
=665
1428.27

S lo g P V 1 4 28.27
Poi = A n tilo g ---------------- = A n tilo g ---------------

= A n tilo g 2 .1 4 7 8
B-15

T e s ts o f A d e q u a c y o f In d e x N u m b e r s
T h e f o llo w in g fo u r te sts h a v e b e e n d e v e lo p e d fo r c h o o s in g a su itab le
in d e x :

(i) U n it t e s t : T h is re q u ire s th a t th e in d e x n u m b e rs m u s t b e in d e p e n d e n t
o f th e u n its in w h ich , o r fo r w h ic h , p ric e s a n d q u a n titie s a re q u o te d . A ll
f o rm u la e e x c e p t th e sim p le (u n -w e ig h te d ) a g g re g a tiv e in d e x sa tisfy th is
test.

(ii) T im e r e v e r s a l t e s t : It is o n e w h ic h h e lp s in d e te r m in in g w h e th e r a
g iv e n m e th o d w ill w o rk in b o th w a y s in tim e , f o rw a r d a n d b a c k w a rd .
T h is te s t im p lie s th a t i f th e tim e s u b s c rip t o f a n y in d e x f o rm u la b e
in te r c h a n g e d th e n th e r e s u ltin g in d e x s h o u ld b e th e r e c ip r o c a l o f th e
o r ig in a l in d e x ( o m ittin g th e f a c to r 100 fro m e a c h in d e x ). T h is te s t
w a s p r o p o u n d e d b y P ro f. I rv in g F ish e r.
S y m b o lic a lly ,
Pol x Pio = 1

T h e F is h e r’s Id ea l In d ex sa tisfie s th is te st

iii) F a c t o r r e v e r s a l t e s t : A n o th e r te s t s u g g e s te d b y P ro f. Irv in g F ish e r to


e x a m in e th e c o n s iste n c y o f in d e x n u m b e r f o rm u la is “F a c to r R e v ersa l
T e s t” . A c c o rd in g to th is te s t, th e p ro d u c t o f a p ric e in d e x a n d th e
q u a n tity in d e x sh o u ld be e q u a l to th e
Spiqi
c o rre s p o n d in g v a lu e index , i.e., P 0i x Q 0i = -----------
£ p 0qo
B-16

T h e f a c to r r e v e r s a l te s t is s a tis f ie d o n ly b y th e F is h e r ’s Id e a l In d ex .

(iv ) C i r c u l a r t e s t : T h is te s t is a n e x te n s io n o f th e tim e re v e rs a l te st a n d is
c o n c e r n e d w ith th e m e a s u r e m e n t o f p ric e c h a n g e s o v e r a p e rio d o f
tim e , w h e n it is d e s ira b le to s h ift th e b ase.

O n ly th e s im p le a g g r e g a tiv e m e th o d a n d th e w e ig h te d a g g re g a tiv e
w ith f ix e d w e ig h ts m e e t th is test.

S y m b o lic a ll y :
Poi x P 12 x P20 = 1
T h e s im p le a g g r e g a tiv e p r ic e in d e x sa tisfie s th e c irc u la r T e s t w h ic h is
show n b e lo w :

EPj EP2 EP0


-------- x -------- x --------= 1
EP0 EP, EP2

F ix e d w e ig h ts a g g re g a tiv e m e th o d a lso sa tisfie s th e c irc u la r te st w h ic h


is s h o w n b e lo w ■

SPiq Epz q Ep0 q


------------- x ------------ x --------------= 1
EP0q Epxq Ep, q
B-17

Illustration 6 : From the data given below show that Fisher’s Ideal Index
satisfies both Time Reversal and Factor Reversal Tests :
1982 1983
Commodities
Po qo Pi qi
A 8 50 2 0 60
B 2 15 6 10

C 1 2 0 2 25
D 2 10 5 8

E 1 40 5 30

Solution :
Computations for Time Reversal Test and Factor Reversal Test
Com­ 1982 1983
modi­ Po qo Pi Piqo Poqo Poqi
qi Piqi
ties
A 8 50 2 0 60 1 ,0 0 0 400 1 ,2 0 0 480
B 2 15 6 1 0 90 30 60 2 0

C 1 2 0 2 25 40 2 0 50 25
D 2 1 0 5 8 50 2 0 40 16
E 1 40 5 30 2 0 0 40 150 30
2piq0 2p0q0 Epiqi Spoqi
=1,380 =510 =1,500 =571

Time Reversal T e s t: Time reversal test is satisfied When


Poi x Pio = 1

EPiqi
Poi =
EPoqi

1500 571 510


Po, x Pio — -------- x ------- x -------
571 1380 1380

Hence time reversal test is satisfied,


H

B-18

F a c to r R e v ersa l T e s t :

IP jq i
Factor reversal test is satisfied when P 0i x Q0i =
SPoqo

i.1
/ S q iP 0 E q iP i
and Qoi =
E q0P0 EqoPi

/ 1380 1500 571 1500 1500


Poi x Qoi = A
510 571 510 1380 510

EPxqi \v'w-)' VL: i• 1500


is also equal to
^ .2PPqo 510

Hence factor Reversal test is satisfied

Q u a n tity In d e x N u m b e rs :
Quantity indices are used to measure average changes in quantities.
These are very useful to measure and compare the physical volume of
commodities produced or marketed or distributed in the given year with
reference to any base year. The quantity indices are computed by using
various formulae of price indices by interchanging q to p and p to q.

Thus,
i) Laspevre’s quantity index
Sqipo
Qoi = —------ x 1 0 0

2q0po
B-19

ii) Paasche’s quantity index


Sqipi
Qoi ~ x 100
2q0pi

iii)F isher’s quantity index

Sqipi
x --------- x 10 0

£q0pi

V a lu e In d e x N u m b e r s :
The value of commodity is the product of its price and quantity, the
value index can be obtained by expressing the total value of current year as
percentage of the total value in the base year.

Thus, ik
Spiqi
V = ---------- x 1 0 0

Epoq0

where, V = Value Index


Piqi = Total values of all commodities in the current year.
Poqo= Total values of all commodities in the base year.

T H E C H A IN IN D E X N U M B E R S :

According to Croxton and Cowden, ‘T he Chain Index is one in


which the figures for each year are first expressed as percentages of the
preceding year. These percentages are then chained together by successive
multiplication to form a Chain Index”.

The following steps must be followed for the construction of Chain Index
Numbers.
: '■■■■' ; , 11; : To.,*}©,! ’l '[ : if.u ■;! ;l
(i) Calculate link relatives for each year

Current year price


Link relative = ------------------------------ x 100
Previous year price
B-20

(ii) C h a in In d e x fo r th e c u rre n t y e a r =

C u rre n t y e a r lin k re la tiv e x p re v io u s y e a r c h a in in d ex

100

N o te : T h e firs t y e a r lin k re la tiv e a n d c h a in in d e x w ill b e 100.

F IX E D B A S E D I N D E X N U M B E R S :

U n d e r fix e d b a s e in d e x n u m b e rs th e y e a r o r th e p e rio d o f y e a rs to
w h ic h a ll o th e r p ric e s a re r e la te d to b e k e p t c o n s ta n t fo r all tim es. F o r
in sta n c e i f th e in d ic e s o f 1991, 1992 a n d 1993, a re a ll c a lc u la te d w ith 1980
a s th e b a s e y e a r, su c h in d ic e s w ill b e c a lle d fix e d b a s e in d ices.

C o n v e r s io n o f C h a i n B a s e I n d e x N u m b e r s to F ix e d B a s e I n d e x
N u m b ers :
F ix e d b a s e in d e x (F .B .I) n u m b e rs c a n b e o b ta in e d fro m th e c h a in
b a s e in d e x (C .B .l) n u m b e rs b y u s in g th e fo llo w in g fo rm u la:

C u rre n t Y e a r C .B .l x P re v io u s Y e a r F B I
C u rre n t Y e a r F .B .I = --------------------------------------------------------------------------
100

N o te : T h e F .B .I. fo r th e firs t p e rio d b e in g th e sa m e a s th e C .B .l fo r th e


firs t p e rio d .

D lu s tr a t io n 7 : C o n s tru c t In d e x N u m b e r b y C h a in b a s e m e th o d fo r th e
fo llo w in g d a t a ______ _________ _____________________________________________
Y ear 1992 1993 1994 1995 1996 1997

P ric e o f W h e a t
530 600 770 800 840 900
p e r 100 K g

S o lu tio n :__________C o m p u ta tio n o f C h a in In d ices


Y ear P ric e L in k R e la tiv e s C h a in In d ex

1992 530 1 0 0 .0 0 100.00

600 113.21 xlOO


1993 600 ------- x 100 = 113.21 --------------------- = 113.21
530 100
770 128.33x113.21
1994 770 ------- x 100=128.33 ------------------------ -- 145.28
600 100
B-21

Y ear P ric e L in k R e la tiv e s C h a in In d ex

800 103.9Qx 145.28


1995 800 ------- x 100 = 103.90 -------------------------- = 150.95
770 100

840 105 x 150.95


1996 840 ------- x 100 =105.00 --------------------- = 158.50
800 100

900 107.14x158.50
1997 900 ------- x 100 = 107.14 ---------------------------- 169.82
840 100

I l l u s t r a t i o n 8 : F ro m th e fo llo w in g d a ta o f th e w h o le s a le p ric e s o f w h e a t
f o r th e te n y e a rs c o n s tru c t in d e x n u m b e rs ta k in g (a ) 1 9 7 9 a s b a s e , a n d (b )
b y c h a in b a s e m e th o d .________________________________ ____________________
Y ear P ric e o f W h e a t Y ear P ric e o f W h e a t
(R s. p e r 2 0 k g ) (R s. p e r 2 0 k g )
1979 50 1984 78
1980 60 1985 82
1981 62 1986 84
1982 65 1987 88
1983 70 1988 90

S o lu tio n :
(a ) C o n s tru c tio n o f In d e x N u m b e rs ta k in g 1979 a s b a s e
Year Price of Index Number Year Price of Index Number
Wheat (1979=100) Wheat (1979=100)
78
1979 50 100 1984 78 -----x 100=156
.... „ . ., 50

60 82
1980 60 -----x 100=120 1985 82 -----x 100=164
50 50
62 84
1981 62 -----x 100=124 1986 84 ----- x 100=168
50 50
65 88
1982 65 -----x 100=130 1987 88 ----- x 100=176
50 50

70 1 90
1983 70 ----- x 100=140 1988 90 -----x 100=180
50 50
(b ) C o n s tru c tio n o f C h a in In d ices
Y ear P ric e o f L in k R e la tiv e s C h a in In d ices
w heat (1 9 7 9 = 1 0 0 )

1979 50 10 0 .0 100.0

60 120x100
1980 60 ------ x.100 = 120.0 ------------------------ = 1 2 0 .0
50 100

62 1 0 3 .3 x 1 2 0
1981 62 ------ x 100 = 103.3 ---------------------- = 124.0
60 100

65 1 0 4 .8 x 1 2 4
1982 65 ------- x 100 = 104.8 ---------------------- = 129.95
62 100

70 1 0 7 .7 x 1 2 9 .9 5
1983 70 -------x 100 = 107.7 ------------------------ = 139.96
65 100

78 1 1 1 .4 x 1 3 9 .9 6
1984 78 -------x 1 0 0 = 111.4 ---------------------- = 155.92
70 100
82 105.1 x 155.92
1985 82 ------ x 100 = 105.1 ---------------------- = 163.87
78 100

84 1 0 2 .4 x 1 6 3 .8 7
1986 84 ------ x 100 = 102.4 ------------------------ = 1 6 7 .8 0
82 100

88 1 0 4 .8 x 1 6 7 .8 0
1987 88 ------ x 100 = 104.8 ---------------------- = 1 7 5 .8 5
84 100

90 1 0 2 .3 x 1 7 5 .8 5
1988 90 ------ x 100 = 102.3 ---------------------- = 1 7 9 .8 9
88 100

I llu s tra tio n 9 : (a ) F ro m th e fo llo w in g p ric e s o f th re e g ro u p s o f


c o m m o d itie s f o r th e y e a rs 1973 to 1977, fin d th e c h a in b a s e in d e x n u m b e rs
c h a in e d to 1973 :
G ro u p s 1973 1974 1975 1976 1977
I 4 6 8 10 12
II 16 20 24 30 36
III 8 10 16 20 24

(b ) A ls o f in d fix e d b a s e in d e x n u m b e rs w ith 1973 a s b a s e year.


B-23

S o lu tio n : (a ) C o m p u ta tio n o f C h a in b a s e in d e x n u m b e rs
Relatives based on proceeding year
Group
1973 1974 1975 1976 1977
6 8 10 12
i 100 -----X 100=150 — X 100=133.33 — X 100=125 -----X 100=120
4 6 8 10
20 24 30 36
h 100 -----X 100=125 ----- X 100=120.00 -----X 100=125 — X 100=120
16 20 24 30
10 16 20 24
n 100 — X 100=125 ---- X 100=160.00 ---- X 100=125 ---- X 100=120
8 10 16 20
Total of
300 400 413.33 375 360
L.R.
Aver­
age
100 133.33 137.78 125 120
L.R.
(AM )

1 0 0 X 1 3 3 .3 3 1 3 7 .7 8 X 1 3 3 .3 3 1 2 5 X 1 8 3 .7 0 1 2 0 X 2 2 9 .6 3
Chain
100 100 100 100 100
Indices
= 1 3 3 ,3 3 = 1 8 3 .7 0 = 2 2 9 .6 3 = 2 7 5 .5 6

(b ) C o m p u ta tio n o f fix e d b a s e in d e x n u m b e rs
Price Relatives (1973=100)
Group
1973 1974 1975 1976 1977
6 8 10 12
I 100 -----X 100=150 -----X 100=200 — X 100=250 -----X 100=300
4 4 4 4 ,
20 24 30 36
H 100 ---- X 100=125 -----X 100=150 — X 100=187.5 ---- X 100=225
16 16 16 16
10 16 20 24
n 100 ---- X 100=125 ---- X 100=200 — X 100=250 ---- X 100=300
8 8 8 8
Total of
Relativ 300 400 550 687.5 825
es
Index
No.
(Aver­
100 133.33 183.33 229.17 275
age of
Rela­
tives)

I l l u s tr a tio n 10 : F ro m th e c h a in b a s e in d e x n u m b e rs g iv e n b e lo w fin d
fix e d b a s e in d e x n u m b e rs :
Y ear 1995 1996 1997 1998 1999
C h a in b a s e in d e x n u m b e rs 90 120 130 120 140
B-24

Solution ;______ Computation of Fixed Base Indices


Year Chain base Fixed base
indices indices
1995 90 90.00
120X90
1996 1 2 0 ---------------- = 108.00
10 0

130X108
1997 130 = 140.40
10 0

120X140.40
1998 1 2 0 = 168.48
10 0

140X168.48
1999 140 = 235.87
10 0

BASE SHIFTING, SPLICING AND DEFLATING INDEX


NUMBERS

Base Shifting
Base Shifting means changing the Base year period of a series of
index numbers and finding out the index numbers with new base period.
The shift in Base period is made in the following situations.

(a) To compare two or more series of index numbers with different Base
periods.

(b) When the Base period is too distant from the current year to make
meaningful comparisons.

Original Index Number


Shifted Index Number = -------------------------------------------------- x 100
Index Number of New Base Period

Illustration 11 : Reconstruct the following indices using 1990 as the base.


Year 1986 1987 1988 1989 1990 1991 1992
Index
1 1 0 130 150 175 180 2 0 0 2 2 0
Nos.
B-25

Solution : Index Numbers (Base 1990 = 100)


Year Index No. Index Number (Base 1990=100)
1986 1 1 0 100
— —X 1 1 0 = 61.11
180

1987 130 100


----- X 130 = 72.22
180

1988 150 100


------- X 150 = 83.33
180

1989 175 100


------- X 175 = 97.22
180

1990 180 1 0 0 .0 0

1991 2 0 0 100
-------X 200 = 111.11
180

1992 2 2 0 100
------- X 220 = 122.22
180

100
Shifted Index No. = __________________ x Original Index No.
Index No. of
New Base period

Splicing
The technique of combining two or more overlapping series of index
numbers into one continuous series is known as Splicing. This continuity
of the series of index number is required to facilitate comparisons.

Index number Old index number


of current year of New base year
Spliced Index Number = ---------------------------------------------------------
100

Illustration 12 : Two sets of Indices, one with 1969 as base and the other
with 1977 as base are given below
(a) Year Index Nos. (b) Year Index Nos.
1969 10 0 1977 10 0

1970 1 1 0 1978 105


1971 1 2 0 1979 90
B-26

1972 190 1980 95


1973 300 1981 102
1974 330 1982 110
1975 360 1983 96
1976 390
1977 400

T h e in d e x (a ) w ith 1 9 6 9 b a s e w a s d is c o n tin u e d in 1977. Y o u are


r e q u ire d to s p lic e th e s e c o n d in d e x n u m b e r (b ) w ith 1977 b a s e to th e first
in d e x n u m b e r.

S o lu tio n :_________________ S p lic in g o f In d e x b w ith In d e x a


Y ear In d ex N u m b er In d e x N u m b e r In d e x N u m b e r (b )
(a ) w ith 1969 a s (b ) w ith 1 9 7 7 a s sp lic e d to (a ) w ith 1969
base b a se as base
1969 100
1970 110
1971 120
1972 190
1973 300
1974 330
1975 360
1976 390
400
1977 400 100 100 X ------- = 4 0 0
100

400
1978 105 105 X ------- = 4 2 0
100

400
1979 90 90 X ------- = 360
100

400
1980 95 95 X ------- = 380
100

400
1981 102 102 X ------- = 4 0 8
100

400
1982 110 110 X ------- = 4 4 0
100

400
1983 96 96 X ------- = 384
100
It 11

D e fla tin g :
T h e p ro c e s s o f d e c re a s in g .1 I i hu " " i i l i i i i - ii»t, • ■
a s to a llo w f o r c h a n g e in th e p i n e lnv«*l i- -tii- i t . m ,, ,
te c h n iq u e a se rie s o f m o n e y w a g e s .......... mi* <mi » - . . . . . . i,,, , . t(
c h a n g e s to f in d o u t th e le v e l o f re a l w agm , m i i i n m i . 1 1. . ............ . , ,

M oney w age
R eal w ages = ------------------------- x 100
P ric e in d e x

R e a l w a g e in d e x o r R e a l in c o m e in d e x =

In d e x o f m o n e y w a g e
--------------------------------------- x 100
P ric e in d e x N o .
(o r)
R e a l w a g e o f th e c u rre n t y e a r
= -------------------------------------------------------- x 100
R e a l w a g e o f th e b a s e y e a r

I l l u s t r a t i o n 13 : T h e a n n u a l w a g e s ( in R s .) o f w o r k e r s a r e g i v e n alone
w ith C o n s u m e r P ric e In d ic e s. F in d (i) th e re a l w a g e s a n d ( ii) t h e r e a l w a g e
in d ic es.
Y ear 1990 1991 1992 1993
W ages 1800 2200 3400 3600
C o n s u m e r P ric e In d ic e s 100 170 300 320 ~

S o lu tio n :________C o n s tru c tio n o f re a l w a g e in d ic e s


Y ear W ages P ric e R eal W ages Real Wagelndices~
(R s .) In d e x 1990=100
1800
1990 1800 100 ------------- X 100 = 1800.0 100
100

1991 2200 170 2200 1294.1


------------- X 100 = 1294.1
170 1800 X 100 = 7 1 .8 9

1992 3400 330 3400 1 1 33.3


------------- X 100 = 1133.3
300 1800 X 100 = 6296

1993 3600 320 3600 1125


------------- X 100 = 1125.0
320 1800 X 100 = 6 2 .5 0
B-28

COST OF LIVING INDEX NUMBERS :

C o st o f liv in g in d ic e s m e a su re th e c h a n g e in th e c o st o f liv in g o f
w o rk e rs d u e to c h a n g e in th e re ta il price. A c h a n g e in th e p ric e level
a ffec ts th e c o s t o f liv in g o f d iffe re n t c la sse s o f p e o p le d iffe re n tly . T h e
g e n e ra l in d e x n u m b e r fa ils to re v e a l th is. So th e re is th e n e e d to c o n stru c t
c o n s u m e r p ric e in d e x .

Uses of Cost of Living Indices :


T h e f o llo w in g a re th e u s e s o f c o s t o f liv in g in d ic e s :
(a ) C o st o f liv in g in d ic e s h e lp s u s to d e te rm in e th e e ffe c t o f th e rise a n d
fa ll in th e p ric e s o f d iffe re n t c la sse s o f p e o p le liv in g in d iffe re n t
g e o g ra p h ic a l re g io n s.
(b ) T h e y a re u s e d w id e ly in w a g e n e g o tia tio n s a n d w a g e c o n tra cts.
(c ) G o v e rn m e n t c a n m a k e u s e o f th e s e in d ic e s fo r w a g e p o lic y , p ric e
p o lic y , r e n t c o n tro l, ta x a tio n a n d g e n e ra l e c o n o m ic p o lic ie s.
(d ) T h e c e n tra l a n d s ta te g o v e rn m e n ts, m a n y b ig in d u stria l a n d b u sin e ss
u n its u s e th e c o s t o f liv in g in d e x n u m b e rs to re g u la te th e d e a rn e ss
a llo w a n c e a n d g r a n t o f b o n u s to th e e m p lo y e e s to m e e t th e ris e in th e
c o s t o f liv in g d u e to ris e in th e p rices.

Construction of Consumer Price Index :


T h e fo llo w in g p o in ts a re to b e c o n s id e re d w h ile c o n s tru c tin g th e co st
o f liv in g in d e x n u m b e rs.
(i) S co p e a n d C o v e ra g e : T h e firs t ste p in th e c o n s tru c tio n o f c o st o f
liv in g in d e x is to d e te rm in e th e c lass o f p e o p le f o r w h o m th e in d e x is
b e in g c o n s tru c te d a n d th e g e o g ra p h ic a l a r e a to b e co v e red .
(ii) F a m ily B u d g e t E n q u iry : T h is is c o n d u c te d b y s e le c tin g a n a d e q u a te
n u m b e r o f re p re s e n ta tiv e fa m ilie s fro m th e c la ss o f p e o p le f o r w h o m
th e in d e x is b e in g d e sig n e d . T h e p u rp o se o f c o n d u c tin g th is e n q u iry
is to d e te rm in e th e a m o u n t, th a t a n a v e ra g e fa m ily sp e n d s o n
d iffe re n t ite m s o f c o n s u m p tio n su c h a s fo o d , c lo th in g , fu e l, h o u se
r e n t a n d m is c e lla n e o u s.
(iii) S e le c tio n o f b a s e p e rio d
(iv ) C o lle c tio n o f p ric e q u o ta tio n s

Methods of Construction of cost of living Indices :


T h e in d e x m a y b e c o n s tru c te d b y a p p ly in g a n y o f th e fo llo w in g
m eth o d s.
It

(1) Aggregate Expenditure Method or Aggi ■ . m . ii<,, j

£Piqo
Consumer Price Index ------------ x 100
SPoqo
T h is is in fa c t L a sp e y re s m e th o d d is c u s s e d p rev io u sly

(2) Family Budget Method or the Method of weighted Kt-lulivr*


EPV
Consumer Price Index = ---------
IV
■' <• V- ' v '!• I ;'A V, . ■••aV i t „ ’{ V i; f Y

Pi
Where P = ------- x 100 for each item
Po

V = V alu e w eig h ts, i.e ., P 0qo-

Illustration 14 : Construct the consumer price index number for 1983 on


the basis of 1982 from the following data using

(i) the aggregate expenditure method, and


(ii) the family budget method__________
Commodities Quantity Units Price in Price in
Consumed in . 1982 1983
1982 (Rs.) (Rs.)
A 6 quintal Quintal 11.50 12.00
B 6 quintal Quintal 10.00 16.00
C 1 Kg Kg 12.00 18.00
D 6 Kg Kg 16.00 20.00
E 4 Kg Kg 4.00 3.00
F 1 Kg Kg 40.00 30.00

(i) Calculation of consumer price index number by the Aggregate


expenditure method.
Com­ Quanti­ Units Prices in Prices in Pi9o Poqo
modities ties 1982 P0 1983 p.
Consumed
in 1982 q0 i
A 6 Quintal Quintal 11.50 12.00 72 69
B-30

B 6 Quintal 10 .0 0 16 0 0 96 60
C IK g Kg 12 .0 0 18.00 18 12

D 6 Kg Kg 16.00 2 0 .0 0 12 0 96
E 4 Kg Kg 4.00 3.00 1 2 16
F 1 Kg Kg 40.00 30.00 30 40
£Piqo IPoq>
= 348 = 293

SPiqo 348
Consumer Price index = ----------- x l0 0 = -------- x 100= 118.77
iPoqo 293
(ii) Construction of consumer price Index Number by the Family
_____ Budget Method. ______ ______ _________ _______________
Price Price Pi
Quantity ----- x 100
Com­ in in Poqo
Consumed Units Po PV
modities 1982 1983 V
qo P
Po Pi
A 6 Quintal Quintal 11.50 12.00 104.35 69 7200.15
B 6 Quintal Quintal 10.00 16.00 160.00 60 9600.00
C 1 Kg Kg 12.00 18.00 150.00 12 1800.00
D 6 Kg Kg 16.00 20.00 125.00 96 12000.00
E 4 Kg Kg 4.00 3.00 75.00 16 1200.00
F 1 Kg Kg 40.00 30.00 75.00 40 3000.00
ZV = ZV =
293 34,800.15

EPV 34,800.15
Consumer Price Index ---------- = ------------- = 118.77
EV 293

Illustration 15 : Construct the cost of living index number from the table
given below :_________ __________________________________________
Group Index for 1982 Expenditure
1. Food 550 46%
2. Clothing 2 2 0 7%
3. Lighting 215 1 0 %

4. House Rent 275 25%


5. Miscellaneous 150 1 2 %
B-31

S o lu tio n :
Construction of cost of Living Index Number
Group Index Number I Expenditure W IW
Food 550 46 25,300
Clothing 2 2 0 7 1,540
Lighting 215 1 0 2,150
House Rent 275 25 6,875
Miscellaneous 150 1 2 1,800
IW = 10 0 £ IW = 37,665

XIW 37,665
Cost of Living Index = --------- = ----------- =376.65
EW 100

THEORY QUESTIONS

1. Explain the term ‘Index Number. Why Index Numbers are called
economic baromters ?

2. Explain the significance of index numbers ? Mention their


limitations ?

3. Write a brief notes on (a) Price Index Number (b) Value Index
Number and (c) Quantity Index Number.

4. Explain the problems involved in the construction of index numbers.


(B.Com., Kakatiya)

5. What are Index Numbers ? How are they constructed ?

6 . What do you mean by tests of consistency for an index number ?


What tests, is a good index number expected to satisfy ?

7. Distinguish between Time reversal test and factor reversal test.


(B.Com., O.U)

8 . Distinguish between ‘Fixed’ and ‘Chain base indices.

9. What is meant (i) Base shifting, (ii) Splicing and deflating of index
numbers ? Explain and illustrate.

10. What do you understand by cost of living number ? What are its
uses ? Discuss the methods used for the computation.
B.-32

PROBLEMS
1. Compute a price index for the following by
(a) Simple aggregative method and
(b) Average of price relative method by using (i) arithmetic mean
and (ii) geometric mean

Commodity A B C D E F
Price in 1990 (Rs.) 2 0 30 10 25 40 50
Price in 1995 (Rs.) 25 30 15 35 40 55
(Ans : (a) PM= 114.29 (b) (i) 120.83; (ii) 119.3)

2. Construct Index numbers from the following data using


(a) Simple aggregative Method.
(b) Average of price relative method by using (i) arithmetic mean
and (ii) geometric mean.
Commodities A B C D E
Price in 1996 (Rs.) 50 60 10 15 25
Price in 1998 (Rs.) 75 60 1 2 18 35
(Ans : (a) P„r=125 (b) (i) PM = 126; (ii) 124.7)

3. Compute price indices for 1999 and 1997 taking 1992 as base year
from the data given below :____________________________________
Commodities
Year
A B C D E F
1999 90 55 70 25 130 70
1997 80 40 60 2 0 1 2 0 50
1992 60 30 40 10 80 40
(Ans : Price index for 1997 = 142.3
Price index for 1999 = 169.2 )

4. Calculate Index number by Average of relative method using (a)


Arithmetic Mean and (b) Geometric Mean.___________ ___________
Commodities A B C D E
Price in 1995 5 1 2 2 1 0 6

Price in 1997 5 8 4 15 8

(Ans: (a) 130 (b) 121.7)


u n

5. From the following details, construe i .in !Mill-s t>u ■nun i .


as the base by the price relative method ii.miir i .u .mil.... ...
(b) geometric mean for averaging relat 1 ves
Commodities A B (' D
Prices (1995) 2 0 1 0 St) n
Prices (2000) 18 1 2 is 40
(Ans : (a) 130 (b) 126.9)

6 . Calculate Price Index numbers by :


(a) Laspeyres method, (b) Paasche’s method and
(c) Fisher’s Ideal method_____________ ____________
1980 1981
Commodities
Price Q ty Price Qty
A 2 0 8 40 6

B 50 1 0 60 5
C 40 15 50 10

D 2 0 2 0 2 0 15
(Ans : (a) 124.69 (b) 125.23 (c) 124.97)

7. Compute weighted price index from the following data by using (i)
Laspeyre’s method (ii) Paasche’s method and (iii) Fisher’s Ideal
Method.
1982 1983
Items
Price Qty Price Qty
A 10 1 2 1 2 15
B 7 15 5 2 0

C 5 24 9 2 0

D 16 5 14 5
(Ans : (i) 118.82; (ii) 112.76; (iii) 115.75)

8 . Construct Fisher’s Ideal Index Number from the following data:


Base Year Current Year
Price per Total Price per Total
Items unit expen­ unit expen­
(Rs.) diture (Rs.) diture
• (Rs.) (Rs.)
A 2 40 5 75
B-34

B 4 16 8 40
C 1 10 2 24
D 5 25 10 60
(AnS : 219.12) (B.Com., Andhra)

Hint: Quantity should be ascertained by dividing expenditure by price.

9. C o n s tru c t F is h e r ’s Id e a l In d ex N u m b e r a n d sh o w h o w it sa tisfie s th e
tim e r e v e rs a l anc fa c to r re v e rsa l t e s t s :
B a se y e a r 1981 C u rre n t y e a r 1988
C o m m o d itie s
P ric e Q ty P ric e Q ty
P 10 100 20 140

Q 8 150 8 200
R 12 120 18 160
S 20 80 30 80
T 16 160 24 200
(AnS : 148.84) (B.Com., Osmania)

10. C o m p u te F is h e r ’s Id ea l In d e x fro m th e d a ta g iv e n b e lo w a n d p ro v e
th a t it sa tis fie s tim e re v e rsa l a n d fa c to r re v e rsa l te sts :
1986 1988
C o m m o d itie s
P ric e Q ty P ric e Q ty
A 4 8 5 8
B 6 9 8 11
C 3 5 4 6
D 7 3 9 4
E 4 2 6 2
(A n s : 1 3 1 .5 6 ) (B.Com., Kakatiya)

11. C a lc u la te fro m th e fo llo w in g d a ta th e F is h e r’s Id e a l In d ex a n d sh o w


h o w it sa tis fie s T im e R e v e rsa l T e st.
P ric e Q u a n tity
Ite m s
1990 1991 1990 1991
A 8 20 50 60
B 2 6 15 10
C 1 2 20 25
Ii r.

D 2

E 1

(Ans: 266.61) <m „„ , m,

12. Compute weighted index numbers for the given d.ii.i .......... ........
Bowley’s method, (b) Marshall Edgewortli method. h i i ■i i , -
method.
Comm Base year Current ycai
odities Price Qty Price Qty
A 2 2 0 4 45
B 4 24 5 30
C 6 30 8 40
D 8 40 1 0 60
(Ans : (a ) 133.20; (b ) 133.46; ( c ) 133.46) (B.Com.,Kakatiya)

13. Construct index numbers of price from the following data by


applying:
(i) Bowley’s Method (ii) Marshall - Edgeworth Method
(iii) Kelly’s Method__________________ ____________________
Comm 1986 1987
odities Price Qty Price Q ty ’
A 5 1 0 6 5
B 2 8 4 6

C 2 19 2 13
D 4 14 5 10

(A n s: (i) 125.6; (ii) 125.48; (c) 125.48)

14. From the following data compute price index by weighted average of
price relatives method using (I) Arithmetic Mean and (II) Geometric
Mean.
Commodities A B C D E
Price in 1990 30 28 40 25 35
Quantity in 1990 1 3 6 10 5
Price in 1995 40 42 50 40 48
(Ans: (1) 141.97; (II) 143.2)
B-36

15. From the following data compute price index by using weighted
average of price Relatives Method using (a) Arithmetic Mean and (b)
Geometric Mean
Commodity Base year price Base year Current year
(Rs.) Quantity price (Rs.)
X 2 2 4
Y 3 2 0 4
Z 4 5 6

(A n s: (a) = 140.47; (b) = 139.8)

16. The following table gives the average prices of five commodities for
the years 1975 and 1980.______________________________________
Commodity Weight Price in Rs.
1975 1980
A 5 3.75 4.80
B 7 2.50 4.00
C 6 4.00 5.25
D 3 2.80 4.20
E 4 3.00 4.80

Calculate the price index number for 1980 with 1975 as the base year
using:
(a) Simple average of price relatives (Ans : 145.85)
(b) Weighted average of Price relatives (Ans : 145.5)

17. From the following data compute quantity indices by using (i)
Laspeyer’s Method, (ii) Paasche’s Method and (iii) Fisher’s Ideal
Method.
Base year 1999 Current year 2000
Items
Price Qty Price Qty
A 4 40 5 50
B 1 0 30 1 2 40
C 3 80 4 10 0

D 5 50 8 50
E 8 40 10 50
(Ans : ( i) = 132.28; (ii) = 130.97; (c) = 131.62)
B-37

18. Compute by suitable method the index number of quantity from the
data given b elo w :_________________ ____ ______ ______________
1998 1999
Commodities
Price Total Value Price Total Value
X 8 80 1 0 1 1 0

Y 10 90 1 2 108

o
Z 16 256 340
(A n s: Qoi = 105.64)
Hint: Since we are given, the value and the price we can obtain quantity
figure by dividing value by price for each commodity. We can then
apply Fisher’s method for finding out quantity index.

19. Compute value index number from the following data


1995 2 0 0 0
Commodities
Price Quantity Price Quantity
P 4 1 0 6 15
Q 10 1 0 8 15
R 8 4 6 10

S 5 15 8 2 0

(Ans: 174.08)

20. Show with the help of the following data that simple aggregative
index satisfies the circular Test.
Prices of Articles
Year
P Q R S T
1994 6 8 1 2 1 0 15
1992 4 6 1 2 1 0 8
1990 10 8 5 4 3

IP , IP 2 IP 0
H int: P0 1 X P 1 2 X P 20= 1 i.e., ------- x ------- x — = 1
IP 0 IP , IP 2
21. From the following data, show how fixed weighted Aggregative
Method satisfies the circular Test ?
Price (in Rs.)
Commodities
1990 1994 1998 Weights
A 2 0 30 40 2 1
B-38

B 30 40 50 23
C 2 0 30 40 1 2

D 1 0 2 0 30 2 1

E 40 50 60 23
Z P ]q S P 2q Z P 0q
H int: P 0i x P i2 x P 20 = 1 i.e., ----------- x ----------- x ----------- = 1
Z P 0q X P ,q Z P 2q

22. From the following data compute chain Index Numbers.


Month July August Sept. Oct. Nov. Dec.
Share
1 1 12.5 10 .8 15 14 16
Price
(Ans : 100; 113.63; 98.17; 136.33; 127.23; 145.39)

23. Construct index number by Chain base method for the following
data.
Year 1990 1991 1992 1993 1994 1995
Price 2 0 30 35 25 28 40
(Ans. 1 0 0 ; 150; 175; 124.98; 139.98; 199.96)

24, Construct Chain index numbers from the link relatives given below.
Year 1992 1993 1994 1995 1996
Link
10 0 1 2 0 103.33 104.84 107.69
Index
(Ans : 100; 120; 124; 130; 140)

25. From the following prices of three groups of commodities for the
years 1994 to 1998. Calculate fixed base and chain base index
numbers with 1994 as base y e a r:_______________________________
Group 1994 1995 1996 1997 1998
X 5 6 9 10 1 2

Y 8 10 15 2 0 25
Z 16 2 0 24 30 36
(Ans: (a) Fixed index numbers : 1 0 0 ; 123.33; 172.5; 212.5; 259.16
(b) Chain index numbers : 1 0 0 ; 123.33; 172.67; 212.63; 258.70)
26 From the chain base index immlwii, jin. it ... n ,..i i ....
index numbers.
Year 1994 19(95 1 our,

Index No. 90 1 1 0 1 1 •>

(Ans : 90; 99; 113.85; 136.62; 177.61)

27. Prepare fixed base index numbers from the cli.im lM .f m.i. , .......u . -
given b elo w : _________ _________ ______
Year 1991 1992 1993 1994 1905
Index
92 1 0 2 104 98 103 10 1
No.
(Ans : 92; 93.84; 97.59; 95.64; 98.51; 99.50)

28. From the following Chain Base Index Numbers calculate Fixed Base
Index Num bers:
1991 1992 1993 1994 1995 1996
80 95 1 0 2 98 105 10 0

(Ans. 76; 77.52; 75.97; 79.77; 79.77)

29. An index is at 100 in 1991. It rises 4% in 1992, falls 6% in 1993,


falls 4% in 1994 and rises 3% in 1995. Calculate index numbers for
the five years with 1993 as base.

(Ans : 102.04; 106.12; 100; 95.92; 98.98)

30. The following are price index numbers (Base 1990=100),


Year 1990 1991 1992 1993 1994 1995 1996 1997
Index
100 105 110 125 135 180 195 205
No.

Shift the base to 1993 and obtain new index numbers

(Ans. 80; 84; 8 8 ; 100; 108; 144; 156; 164)

31. In the following series of index numbers shift the base from 1992 to
1994 and recast the index numbers.
Year 1992 1993 1994 1995 1996
Index No. 10 0 108 1 2 0 150 2 1 0

(A ns: 83.33; 90; 100; 125; 175)


32. The following are the two sets of indices ‘x ’ with 1985 and ‘y’ with
1988 as base periods. The index numbers ‘x ’ with 1985 as base
discontinued in 1988. Splice the index ‘y ’ with ‘x \

Year Index V Year Index V


1985 10 0 1988 10 0

1986 240 1989 105


1987 250 1990 1 2 0

1988 260 1991 90


1992 1 1 0

1993 1 2 0

(Ans : 260; 273; 312; 234; 286; 312)

33. Splice the following two index number series, continuing series A
forward and Series B backwards.
Year 1988 1989 1990 1991 1992 1993
Series A 10 0 1 2 0 150
Series B 10 0 1 1 0 1 2 0 150

(Ans : (A - 150; 165; 180; 225) (Forward)


(B - 6 6 .6 6 ; 80; 100) (Backward))

34. The following data relate to the income of the people and General
Index Numbers of prices of a certain region. Calculate Index
Numbers of Real Income with 1988 as base :

Year Income Price Index


1988 800 10 0

1989 819 105


1990 825 1 1 0

1991 876 1 2 0

1992 920 125


1993 938 140
1994 924 140
(Ans : 100; 97.5; 93.75; 91.25; 92; 83.75; 82.5)
B-41

35. Following table shows the annual wage and price index of a labour.
Prepare the real wage index for the labour._____________ __________
Year Wages Price Year Wages Price
(Rs.) Index 1 ' i:1 (Rs.) Index
1987 2 0 0 10 0 1971 360 300
1988 240 160 1972 370 320
1989 350 280 1973 375 330
1990 360 290
(Ans : 100; 75; 62.5; 62; 60; 57.8, 56.8)

36. Calculate the index Number using the Aggregate Expenditure


Method for the year 1983 with 1973 as base year, from following
d a ta :
Commodity Quantity in Price per Unit in Price per Unit in
Units in 1973 1973 (Rs.) 1983
A 1 0 0 8 .0 1 2 .0 0

B 25 6 .0 0 7.50
C 1 0 5.00 5.25
D 2 0 48.00 52.30
E 65 15.00 16.50
F 30 19.00 27.00
(Ans : Consumer price Index = 124.6)

37. Calculate the index number using both the Aggregate Expenditure
Method and Family Budget Method for the year 1973 with 1960 as
the base year from the following data.___________________________
Commodity Quantity in Price per Unit in Price per Unit in
Units in 1960 1960 (Rs.) 1973
A 10 0 8 .0 0 1 2 .0 0

B 25 6 .0 0 7.50
C 1 0 5.00 5.25
D 2 0 48.00 52.00
E 25 15.00 16.50
F 30 9.00 27.00
(Ans: 142.1) (B.Com., Bangalore)
B-42

38. Calculate the Index Number, using family budget method for the year
_____ 1980 with 1970 as base year from the following d ata:_____________
Commodity Quantity Price 1970 Price per unit
Consumed 1970 (R s) 1980 (Rs.)
A 10 0 8 1 2

B 25 6 7.50
C 1 0 5 5.25
D 2 0 48 52
E 95 15 16.50
(Ans: 119.57)

39. An enquiry into the budget of middle class families in a town


revealed the following information.______________ _______ _______
Food Fuel Clothing Rent Misc.
Expenses o n :
35% 15% 2 0 % 2 0 % 1 0 %

Prices in 1980 (Rs.) 80 30 45 60 25


Prices in 1988 (Rs.) 150 55 75 130 50
Constmet cost of living index number by using family budget method.
(Ans: 189.79)

40. An enquiry into the budgets of middle class families in a certain city
gave the following information :________________ _______________
Food Fuel Clothing Rent Misc.
Expenses o n :
35% 1 0 % 2 0 % 15% 2 0 %

Prices in 1980 (Rs.) 150 25 75 30 40


Prices in 1988 (Rs.) 145 23 65 30 45
What is the cost of living index number 1981 as compared with that of
1980 ?
(Ans: 97.87) (ic w.A)

41. Calculate the cost of living Index Number


Items Weights Index Number
Food 8.5 140
Rent 2 .0 175
Clothing 3.5 160
Fuel & Light 1 .0 135
| M im I’ l l i i n r i i l l ' I 11

(Ah'. I ‘ f• I

42 C o n s t Jill I i i' I III 11 1 II1 |' null MUml mi 11 II


lll‘ lllh U .-ii'h i• 1'i ii c Relatives
A 33 140
II id 120
c 12 130
1) 1 110
(A ns I I I '> i (B.Com., O.U)

43. lii tin- tniiM m .in.il ..I ,i i i-ii.m i nl Living Index Number, the
followmi’. j'.iiH111 in.i.■ - 11111111u•i •. were found. Calculate the cost of
I ,i vi iif. Iin I* , in .... . i i.. iim ii|- (i ) Weighted arithmetic mean, and (ii)

QfOtip Index Numbers Weights


Fowl 350 5
Fuel A li)’,liimi' 2 0 0 1

Clot 1ii ii)'. 240 1

House Ri-nl 160 1

M im I’lliuieous 250 2

(Ana: (i)2S3; (il)2754) (B.Com., Bombay)

EIW Elog 1 W
Hi n t: Formulae (0 - (ii) Antilog
EW EW

44 A textile worker in the city of Bombay earns Rs.350 per month. The
cost of living for a particular month is given as 136. Using the
fol lowing data find the amounts he spent on house rent and clothing.
Group Expenditure Group Index
Food 140 180
Clothing ? 150
House Rent ? 10 0

Fuel & lighting 56 1 1 0

Miscellaneous 63 80
(Ans : He spent Rs.42 on Clothing and
Rs.49 on House Rent)
J

B-44

45. An enquiry into the budgets of the middle class families of a certaii
city revealed that on an average the percentage expenses in th
different groups were :

Food 45, Rent 15, Clothing 12, Fuel and Light 8 , Miscellaneous 2J
The group indices for the current year as compared with a fixed bas
period were respectively 410, 150, 343, 248 and 285. Calculate th
cost of Living Index Number for the current year.

Mr. X was getting Rs.240 in the base period and 430 in the curr : r
( year. State how much he ought to have received as extra allows Jur
to maintain his former standard of living ?
(Ans: Rs. 350 more to maintain the base year’s Standard of
living)
I
1
I '1

j.

1
J
1

You might also like